ARROZ CON MANGO

Dear friends, these are remembered/repeated questions (RQs) and answers I COPIED and PASTED from different discussions on Facebook. I feel sorry because I couldn’t organize the file the way I wanted but I hope it helps. Probably you’ll find some wrong answers in this file, but PLEASE … DO NOT CRITICIZE! Find out the right answer, learn it, share it, PASS your test and BE HAPPY J I wish you all the best GOD BLESS YOU! PAITO

1. All of the following are adverse effects of opioids except? diarrhea and somnolence 2. Advantage of osteogenesis distraction is? less relapse, large movements 3. An investigation that is not accurate but consistent is: reliability 4. Remineralized enamel is rough and cavitation? Dark hard and opaque 5. Characteristics of a child with autism - repetitive action, sensitive to light and noise 6. S,z,che sounds : Teeth barely touching – True 7. Something about bio-transformation, more polar and less lipid soluble? - True 8. How much of he population has herpes? 80% - (65-90% worldwide; 80-85% USA) More than 3.7 billion people under the age of 50 – or 67% of the population – are infected with herpes simplex virus type 1 (HSV-1), according to WHO's first global estimates of HSV-1 infection published today in the journal PLOS ONE. 9. Steps of plaque formation: pellicle, biofilm, materia alba, plaque 10. Dose of hydrocortisone taken per year that will indicate have adrenal insufficiency and need supplement dose for surgery - 20 mg 2 weeks for 2 years 11. Rpd clasp breakage due to what? Work hardening 12. Most impacted tooth? Third molar not in options - Maxillary canine 13. Least common survival of cancer – white female 14. Aspirin mode of action - inhibit irreversible platelet aggregation thromboxane a2 15. Myasthenia gravis, what is contraindicated? erythromycin and aminoglycosides 16. Treatment for ANUG - Debridement and mouth rinse with h2O2 and if systemic involvement then antibiotics 17. Which type of periodontitis needs antibiotics – LAP 18. Antibiotic used for periodontitis - Tetra and Pen V, also metro 19. Asthma physiology definition - Wheezing on expiration 20. Ranitidine definition - Selective H2 (Histamine) receptor antagonist, these receptors are present in lining. Thereby blocking the receptors and prevents release of Gastric acid. 21. Which is prevalent: 1 wall defect, 2 wall defect, 3 wall defect 22. Penumbra definition - Penumbra is lack of sharpness of the film. 23. Which is more hydrophillic pvs or polyether (this question was asked to me twice during day1) - Polyether (but also hydrocolloids) 24. Many questions on study model all leading to a common answer that was cross sectional study model (they tend to repeat the questions in different formations during day 1). 25. Face division vertically and horizontally - vertical 3 and horizontal 5 26. Disease more common in men (hemophilia) 27. One q on relation between incisal guidace is equal and condylar guidance - When adjusting the condylar guidance for protrusive relationship, the incisal guide pin on the articulator should be raised out of contact with the incisal guided table. 28. Many qs on study model leading to a common answer that was clinical trail (mostly they were on effectively of practices, drugs, etc). 29. Which has poorest prognosis and the answer was PLGA tumor. They had confused it with adenoid cystic and mucoepi - False - Pleomorphous Low Grade (PLGA) has good prognosis after surgical excision (Mosby) – I think answer is

30. A lot of questions on anti cancer drugs so study it carefully… really sorry that i don’t remember them as I was poor in that topic - Dihydrofolate reductase by methotrexate, amoxicillin inhibits renal tubular secretion of methotrexate. 31. What comes before plaque or pellicle or biofilm 32. Deepest part of occlusal rest for rpd - marginal ridge 33. Simple questions in oral patho about cleidocranial as to what it affects (clavicles) 34. Many many many questions in endo with positive or negative findings in relation to percussion, palpation, night pain and then diagnosis of the combinations were asked. 35. Simple questions in relation to pulpectomy and pulpotomy: pulpotomy - vital / pulpectomy - non vital 36. Questions on modellling technique in pt mngmt (pt made to observe his cousins or friends behaviour to improve his own) 37. Disto buccal extension of mandi cd lateral limit influenced by masseter or ramus 38. Key feature of the custom tray during final impression is its under extension? - The custom tray for a final mandibular or maxillary complete denture impression should have a spacer w/ stops to ensure the tray will be seated in proper relationship to the arch, and to ensure adequate room for the impression material. Other features: trimmed 2 mm short of the mucosal reflection and frenae. The primary indicator of the accuracy of border molding is the stability and lack is displacement of the tray in the mouth. 39. Class ii and iii drug: S II: amphetamines, morphine, cocaine, pentobarbital, oxycodone, methadone, codeine, and Percocet (acetaminophen + oxycodone). (Must have a written prescription and cannot be refilled) S III: Tylenol 3, Vicodin - must have a written prescription, refills do not need new prescription and may be called into the pharmacy.

40. Absolute contra of papoose straps emergency situation or a co-operative patient 41. Hemorrhage common complication of maxi extractions and to my surprise the same question with mandi extractions was asked but without the option of dry socket, infection was the only sane option so i went for it. 42. A lot of questions on 3/4th and 7/8 crowns but they were basic ones and i could derive the answer by using the facts given in dental decks.

43. A lot of questions on ethical principals of a dentist (guys for these questions please ref to the ethical principal details mentioned on the ada website…. If you do it from there you will be able to nail each and every one of: http://www.ada.org/~/media/ADA/Member%20Center/FIles/2016_ADA_Code_Of_Ethics. pdf?la=en 44. What sound is affected if incisors are placed too far superiorly and ant: f and v 45. Best (conservative) method to close a 1 mm non patho diastema in a young adult is: composite 46. If a patient doesn’t have 3rd molars how many furcations will he have in mxi arch (to make the calculation simple there are three furcations for maxi teeth mesial distal and Buccal rest is simple calculation) - 16 - maxillary molars have 3 furcations (4 molars * 3 furcations = 12), 1st pm is bifurcated, that means it has 2 furcations (2*2=4). We have 16 furcations. 2nd premolars, central incisors, lateral incisors and canines do not have furcations. 47. Most common site for graft iliac – true 48. Most common site for mandi fracture – condyle 49. Complication with distraction osteogenesis - nerve damage 50. Implant analogue used to replicate the position of implant on the implant platform (confusing options were if its used for impressions? Nope that’s impression copings – An implant analogue is an implant replica and is inserted into the set impression material before the cast is poured. Impression coping is attached directly to the implant fixture head. Impression copings facilitate transfer of the intraoral location of the implant to the same position on the laboratory cast. Impression copings can be either screwed into the implant body or screwed or snapped onto an implant abutment. 51. Simple questions on implants like distance from adjacent tooth and the implant 1.5mm 52. Distance from the ant loop and the implant which is 5mm 53. During IANB bleeding is caused due to puncture into which artery – inferior alveolar artery 54. Two questions on ideal resto material for a class v lesion but this patient was young and had an active caries condition (I was shocked to see this question repeat twice with minor formation changes) - According to DD: glass ionomer is indicated in high caries risk and CLASS V where esthetics is a concern, so I would go with GIC. 55. Two questions on the incision for int bevel in which direction is it directed - the internal bevel incision (reverse bevel) starts at a distance from the and is aimed at the bone crest. 56. Which of the following cements should not be used with all ceramic restorations due to reports of ceramic fractures? 1. Gic 2. Resin luting cement 3. Zinc phosphate cement 4. Resin modified GIC 57. Same went for gingivectomy (same q on where is the incision directed) - above . 58. Why do we remove bone during modified widman flap - to achieve a good tissue adaptation to the neck of the teeth. 59. Which type of comp are used for two cases posterior comp and for repairing small occlusal defects (micro, mid, macro, etc ) - Microhybrids - they have particular size small enough to polish to a shine similar to microfills but large enough to be highly filled, thus achieving higher strength. Particle size small enough to polish and high strength. Retain good properties of hybrids (strength), with improved handling. Polishability almost equal to microfills. Universal use—anterior and posterior restorations. - Microfills: high polishability but low fracture toughness (esthetic areas, anterior restorations), lower elastic modulus—better in class V situations. - Hybrids: not high polish but improved physical properties compared to microfills., Universal use—anterior and posterior restorations. - Macro filled composites: First-generation” restorative composites. Poor physical and mechanical properties. Poor esthetics.

60. Why don’t we do posterior comp - because of its low wear resistance we don’t do posterior composite in cases of . 61. Dentist does a comp but shade is too light what is the most conservative mngmt which acc to my opnion was apply tint and not redo the entire restoration – tint 62. Simple q like chisel cuts - used mainly to cut enamel 63. A lot of questions on mngmt of hypertension like wha drug for mild cases, what drug for sever cases or htn emergency: - Mild hypertension: CHLOROTHIAZIDE (thiazides), diuretics, beta-blockers such as PROPRANOLOL, alpha1 blockers such as ATENOLOL, centrally acting adrenergic drugs such as METHYLDOPA or CLONIDINE, angiotensin converting enzyme inhibitors such as CAPTOPRIL, LISINOPRIL. - Severe hypertension: GUANETHIDINE and ganglionic blocking agents. 64. What not to give in COPD emergency mnmgt which acc to my opinion was only oxygen (other options had a potent bronchodilator along whith o2 which is the way to go) – True, inhalation of 100% oxygen is contraindicated in a patient w/ COPD 65. Montelukast and zakirlucast what type of drugs in relation to their effect on leukotriene - Block leukotriene (cys-LT1) receptors 66. Simple calculations in LA (based upon normal values and how much to give i the give case, how much epi is present in x carpule ). 67. Effect of age on biotransformation of the drug (this was a bouncer for me)- W/ age -> slower rate of biotransformation and reduced rates of elimination - Children will inactivate and eliminate medications faster than adults. In the elderly there may be a diminished dose requirement for many medications because of age-related decreases in liver mass, hepatic enzyme activity, and hepatic blood flow. 68. If one increases the h2o to powder ratio what does it do with relation to hardness and expansion (this is for gypsum) - decreased setting expansion and decreased strength (increased setting time) 69. What property of a cement is not affected by water to powder ratio options were solubility, consistency, thickness, etc 70. What would one prefer distal rest or mesial rest…. always go for mesial rest - mesial to edentulous area.

71. Sulcular depth req for lingual bar - depth = 7-8 mm minimum At least 4 mm below the gingival margins 72. Most rigid type of maxi major connector - anterior-posterior palatal strap 73. Question on migrane with how the TRIPTANS work - Triptans are agonists at serotonin 5- HT(1b/1d) receptors - serotonin 5-HT agonist antimigraine drugs (Mosby) 74. Pt with symptoms of headache fatigue fever and vision loss… the only sane option was temporal arteritis – True: weight loss, polymyalgia, rheumatic, fever, decreased vision, jaw claudication. (Mosby) 75. Supernumerary teeth with intestinal polps and osteomas (Gardners syndrome) - True 76. Unstimulated flow from submandibular salivary gland - 0.1ml/minute 77. Ant flabby tissue under maxi cd and mandi ant natural teeth combination syndrome or kellys syndrome – True 78. Pka with rate of onset, lipid solubility with bioavailability of a drug (two questions on that) 79. Sign of kidney failure which will levels will shoot the earliest blood: uric acid, creatinine, creatinine phosphokinase, urea, etc - creatinine 80. Q on when to treat patient with respect to dialysis: one day after his dialysis. 81. Commonly used nsaid in kids is: ibuprofen 82. Acetaminophen acts on temp centre in hypothalamus thus causing its anti pyretic effect, options were supposed to confuse us to choose prostaglandins. Hypothalamus -> temperature regulation center. 83. Then the same old same old question was asked as to if a patient has basal cel carcinoma and you are about to break the news to him and he says that just tell me straight do i have cancer or not, first response should be should i cal someone to be with you, the other option which made sense was that prognosis of basal cell is good but this shoudnt be the anser in my opnio as the patient is in a grumpy mood and you need to calm him down first before going on to discuss the details with him). 84. Advantage of led light cure with respect to haolge - energy efficient, lightweight, lifetime bulb. 85. What is the first stage of HIV infection, i went for asymptomatic phase as that’s the first stage followed by acute infection, aids related complex which as hairy , , and other stuff and lastly full blown AIDS. 86. Dementia early sign short term loss or long term loss… i went with short term as one of my family member suffered from it so i remembered it crystal clear. 87. Autism patient will have problem with listening and speaking there was a q on that too - Delay in several skill areas as talking, listening, plating with peers, and attention span. 88. Denture considerations in patients with diabetes like use porcelain teeth, arch shape should be narrow, imp with non pressure tech rest of the points are given in decks… the question was on arch shape 89. One opg with a radio lucency extending from post mand to ant, the sane option was OKC and the confusing options were ameloblastoma, but since ameloblastoma spreads labiolingually rather than ant post - OKC - antero-posterior direction without bony expansion and often associated with impacted tooth. 90. One more pano with radiolucency seen with unerrupted tooth and the only sane option was - It contains a crown of an unerupted tooth (DD) 91. Cause of ORN which had two conf options like bisphos and 42 gy , i went with bisphos as 42 gy is susceptible and more than 60 gy means he has high chances but bisphos is like the principal cause that’s y it was called BRONJ but now they have changed it to MRONJ which is medically induced orn – FALSE: (ORN) is related to head and neck radiation and BRONJ is bisphosphonate-related - related to intravenous and oral bisphosphonates therapy. If the case is about ORN the answer is not bisphosphonates. If the question is about BRONJ or MRONJ the answer should be bisphosphonates. 92. Base value for good patient relation is: communication should be good 93. Same follow up on this question was active listening is done by active eye contact. 94. Never judge a patient …this was the q and a never what …. (and the only sane option was) judge 95. Plz do all the interferences and wht cusps occlude with what during maximum intcusptn… there is a nice video on youtube called fence diagram video which i had done during part one that helped me a lot. 96. Very little qs on ortho mostly with cephalometrics angles like sna snb and stuff bc they were kept in store for day two 97. Dist between casette/grid and the collimator for a lateral ceph 15cm, 60 cm, 5 inch, 6 inch (as per i remember these were the options, but i just randomly gussed this one ).

98. One pano of a kid and dental age was asked 99. Supernumerary teeth occurs due to problem in what stage of tooth development ans was: initiation 100. Basic questions on space maintainers like loss of uni first primary molar what does one give – Band and loop 101. Class v cavity punch the hole on rubber dam more bucally - True 102. Same for gold restoration use 212 ivory clamp - clamp #212 for CLASS V facial preparation 103. Same for teeth who bear clamps the hole should be bigger than the rest - hole punched larger than usual and slightly to the facial of the other holes in the arch. - BB 104. Intrusion of primary maxi central 5mm inside what to do splint it or leave and follow up and allow to re-erupt. 105. Most common cause of amalgam failure imp prep or moisture contamination 106. Bur to polish porcelain is diamond 107. Question on what type of shouldn’t be removed during prep which is eburnated dentin - True - we don’t remove sclerotic = eburnated dentin 108. One question which i don’t remember very clearly but it was on the fact that never do pulpo on closed apex – True, pulpotomy is indicated on undeveloped primary roots. 109. One thing we cant see clearly on 2d image is missed canal or extra canal which requires CBCT - Dental cone beam computed tomography (CT) … but the way this was put was confusing but once i read it carefully i could break it down . 110. Le forte 1 was with max sinus 111. Plz do all the elevators carefully, i got a question saying which of these can’t be used for mandi pm and the only sane option was 151 – False: #151 is for mandibular pm, we can’t use #23 because it is for mandibular molars. 112. You need to adjust the denture near bucca frenum as the denture keeps on falling when the patient smiles… this is due to which muscle – Because it’s maxillary it should be orbicularis. Buccal frenum -> triangularis (mandibular), Labial frenum -> orbicularis (mandibular and maxillary). 113. For implant the instrument specification is: low speed and high torque 114. One question on bundling with the same scenario as dentist charging a procedure as one whe and the insurance company charging it as two different procedures - I think it’s unbundling -> separating of a dental procedure into component parts. 115. is seen on: Attached gingiva 116. You did a prep with high speed and diamond bur, tooth is sensitive, what is it about bur and handpiece that it caused sensitivity? A) Desiccation b) traumatized dentin c) Heat 117. What is lithium used for? Psychotic stage of bipolar 118. 8yrs old girl with ant , max laterals have not erupted yet. When and how do you treat crossbite? I put immediate tx and some appliance with springs 119. Same girl, supposedly there’s a supernumerary not erupted next to 6, what can happen when u extract it? A-necrosis of 6 B- necrosis of 7 C-necrosis of 8 D-7 won’t erupt 120. Benzo overdose? Flumazanil 121. Flumazenil what for? Reverse benzodiazepines 122. Tooth most with dry socket. – mandibular 3rd molar 123. Pvs and latex interaction - sulfur in the latex retards the setting of addition silicone materials. 124. What does major connector does? Rigidity and Stability 125. Width of palatal strap - I put 8mm not sure, (single palatal strap its width is more than 8mm) 126. How do u compensate protrusion in articulator something like that. I put slide forward 5-8mm 127. Lesion hard and painful near lower pm. Options fibroma, neurofibroma, traumatic neuroma. They didn’t say that pt had denture. (Traumatic neuroma = painful / Neurofibroma = asymptomatic) 128. Lesion on tongue: , Ectopic thyroid gland, Giant ossifying fibroma (Pyogenic granuloma = 75% on gingiva / Giant ossifying fibroma = exclusive to gingiva / Ectopic thyroid gland = base of tongue) - it's provably pyogenic granuloma because pyogenic granulomas are also found on the tongue, BUT! Keep in mind that neurilemoma (schwannoma) is an encapsulated mass that presents as an asymptomatic lump and the tongue is the most common location. 129. Hemangioma on tongue – hamartoma 130. Pt with aids what do u ask? Cd4 or viral count – T-cell counts are most important for staging the disease and guiding dental treatment. (Kaplan Cases) 131. New pt upset, crossing arms, looking at floor. What do u ask after introduction -What brings u here today? 132. Pointer in facebow, what for – designed to mark anterior reference point (infraorbital notch) and can be locked in position with a clamp. It is present in the arbitrary facebow. 133. Saturation -Chroma –hue –value 134. What albuterol causes -risk of caries -dry mouth (chronic use of albuterol is known to cause xerostomia). 135. What not to give in asthmatic attack? We give: 1st oxygen w/ beta2-adrenergic agonist (albuterol, terbutaline). If resistant to beta agonists => Theophylline. We can also give corticosteroids. Epi is only for severe asthmatic attack. 136. Heart rate in child - 110 137. Max dose of Tylenol – 4g (4000 mg per 24 hours) 138. Tylenol kids: 10-15 mg/Kg dose every 4-6h 139. Kid is 65 pounds, how many carpules of lido? 1-3 140. ANB of 5.1 what skeletal class is - Class II (ANB >4 = Class II) 141. SNA of 87 and SNB of 81 what it means - Skeletal class II – prognathic maxilla 142. Pt wants dentures. For tx, pt had 2 and 3, 22, 24, 24, 25, 27, 32. 18 What is unnaceptable do for tx plan option. There were different types of options with implants, fpd, etc. I put extraction of all teeth 143. Tylenol which schedule - Sch 3 if with codeine (Tylenol 3) 144. Need of sinus lift for ext of ant upper teeth? False! 145. What pain med for patient with apnea - Nevertheless, patients with OSA who undergo surgical procedures should receive regional analgesia and non-opioid agents (e.g., NSAIDs) if there are no contraindications for their use. 146. Disadvantage of NO2 - It is not a complete pain reliever (a local anesthetic is still required), nausea is the most common patient complaint, and diffusion hypoxia can occur. 147. What could cause unilateral class II molar? Early extraction of A (max 2nd molar) Early unilateral loss of a deciduous second molar is likely to cause the developing dentition to be crowded as the first permanent molar will drift or tip mesially. 148. 0.1% chlorhexidine gel. Where do u use it? Interproximal caries Root Erosion Occlusal amalgam with minor open margin 149. Goal of GTR? - Coronal movement of PDL - regenerate the periodontum, promoting growth of endothelial cells, osteblasts and cementoblasts (NOT EPITHELIAL CELLS) 150. What no to do on D of 3rd molar is there’s not enough keratinized tissue? I put Distal wedge 151. What anxiolytic for pregnant and breastfeeding – Promethazine 152. DMFT for what study? Community trials (epidemiological) 153. Who has more diabetes? Black males 154. Perio doesn’t depend on nutritional - T 155. In class III kennedy where do u get support and retention from? Entirely tooth supported (abutments) – Rests and bases for support 156. Pt with porcelain and amalgams, what fluoride to give? Neutral NaF 157. Most common anomaly? DI (after cleft ) 158. First pass metabolism – Liver (enteral – oral) 159. Teeth joined by dentin and ? Fusion 160. What pain med for liver toxicity? Oxycodone 161. What pain for pt with bleeding problems? Acetaminophen 162. Ginseng not with? Aspirin 163. Advantage of IV – Titration 164. Reverse epinephrine by – Phentolamine (selective alpha blocker phentalomine, phenoxybenzamine, prazosin) 165. Epi works on which receptor: all alpha and beta 166. First side effect of beta blocker - weakness or drowsiness (The most common adverse side effects of beta blockers are WEAKNESS & DROWSINESS) - BB 167. Which Anesthesia without epi – Mepivacaine 168. Allergic to both ester and amide what to give – Diphenhydramine (Benadril) 169. EMLA composed of which two type of anesthesia - Lido 2.5 % and 2.5% prilocaine 170. Which Anesthesia not with anemia – prilocaine 171. Anesthesia not vasodilator – Cocaine 172. Why anesthesia not work in inflammation - Less free base 173. First nerve affect by LA - Small unmyelinated (pain and temperature) 174. Which trimester contraindicate the No2 -1st 175. Most Caries where? Max 1st molar (according to Kaplan) 176. Overdose of LA what to give – diazepam (for toxicity) 177. Antagonist of diazepam – flumazenil 178. Problem in liver which benzo to give - LOT- lorazepam, oxazepam n temazepam 179. Buspirone which receptor works in – Serotonin - BUSPIRONE (BUSPAR) - a minor tranquilizer orally administered anxiolytic (anti-anxiety agent) whose mechanism works by DIMINISHING SEROTONERGIC ACTIVITY. - BB 180. Best anti epileptic for absence seizure – Ethosuxemide 181. LA contra with which drug: MOA - LA with epi contra with MAO inhibitors, also with TCA – Epi contraindicated w/ Levodopa. 182. Omeprazol used for (GERD or zollinger)? – GERD 183. Remember all the antibiotics effect on protein synthesis - Clinda, macrolides, tetracyclines, aminoglycosides, chloramphenicol 184. Most common side effect of clindamycine is - Psudomembranous colitis 185. Mech of action of nystatin - inhibit ergosterol 186. Which drug increase the effect of amoxicillin – Probenecid 187. Drug not with milk – Tetracycline 188. All of these are antipsychotic exept (lorazepam) - T 189. Drug not with cimetidine – Terfenadine (Seldane) (dangerous drug interaction) – Cimetidine: a potent inhibitor of hepatic drug-metabolizing enzymes. 190. Drugs known to interact with SELDANE are ketoconazole (NIZORAL), erythromycin (E-MYCIN), nefazodone (SERZONE), itraconazole (SPORANOX), clarithromycin (BIAXIN), mibefradil (POSICOR) as well as grapefruit juice. 191. Eps (Extrapyramidal syndrome) with which drug except: clozapine - Effectively treats Schizophrenia and more effective & less toxic than the older drugs. - BB 192. Aspirin cuz all except (hyperpnea / tacypenia / hyperkalemia / hyperthermia) – It causes hypokalemia, not hyperkalemia. 193. Tx of motion sickness – scopolamine 194. Tx of myasthenia gravis – pyridostigmine (edrophonium just for diagnosis) - Neostigmine or Pyridostigmine - are cholinesterase inhibitors used to reverse the NMJ blockade (paralysis) caused by NMJ blockers. - BB 195. Edrophonuim all true except (cause dry mouth) – Edrophonium is an indirect- acting cholinergic agonist (cholinomimetic). Drug of choice to "diagnose" myasthenia gravis because of its rapid onset of action and reversibility. 196. Which drug is safe in Myesthenia Gravis (a) Penicillin (b) Erythromycin (c) Clarithromycin (d) IIDR 197. Tx of xerostomia due to radiation – Pilocarpine 198. Overdose of organophosphate cholinesterase inhibitor – Pralidoxime 199. Dont use cortisone in (all the answer were contraindicate so I picked all) 200. Mech of action of diltiazem - Calcium channel blockers useful as anti-anginal agents to treat chronic stable angina pectoris by blocking calcium entry through the membranous calcium ion channels of cardiac and vascular smooth muscle. - Calcium channel blockers that prevent angina attacks by dilating coronary blood vessels to improve blood flow to the heart muscle. - Calcium channel blockers decrease oxygen demand by reducing afterload by reducing peripheral resistance via vasodilation. 201. Contra for use digoxin – Diuretics (will inc digitalis toxicity), also contraindicated in ventricular fibrillation & ventricular tachycardia. 202. Mech of action of heparin - potentiation of Antithrombin III, thus inactivating thrombin. This prevents the formation of fibrin. – BB 203. The most important anticoagulant effect of heparin is to interfere with the convertion of: 1. PTA t PTC 2. PTC to factor VII 3. Fibrinogen to fibrin – (ASDA) 4. Prothrombin to thrombin 5. Proaccelerin to accelerin. 204. Aldosterone antagonist – Spironolactone – Spironolactone a pharmacologic antagonist of aldosterone in the collecting tubule. Spironolactone competes with aldosterone receptor sites in the renal tubules causing increased secretion of Na+, CI, and H20, while conserving K+. - BB 205. Cyp34A u should know how drug effect on it, from dd only - Inhibitors are antifungals, increase triazolam serum concentration. Cytochrome P450 3A4 (abbreviated CYP3A4, is an important enzyme in the body, mainly found in the liver and in the intestine. It oxidizes small foreign organic molecules (xenobiotics), such as toxins or drugs, so that they can be removed from the body. – Wiki 206. Which drugs affect cytochrome P450 metabolism of other drugs? erythromycin lowers cytochrome P450 metabolism of other drugs, Macrolid ABs, antifungual, cimetidine 207. Tx of glaucoma by: betaxolol – Pilocarpine, Betaxolol, Latanoprost, Bimatoprost 208. Overdose of lead tx by - EDTA by chelation (Mosby) 209. Modafinil decreases GABA to improve what – glutamate (Modafinil decreases GABA and increases glutamate, dopamine, and orexin)

210. Overdose of morphine tx by – Naloxone (Opioid adverse effects are reversed & recovery hastened by administration of Naloxone (Narcan) an opioid antagonist.) 211. Only opioid transmucosal is – Fentanyl 212. Side effects of opioid - sedation, dizziness, nausea, vomiting, constipation, physical dependence, tolerance, and respiratory depression. 213. Common Side Effects OF OPIOIDS: sedation and drowsiness (by depressing the conscious centers of the brain), dizziness, & nausea. The MOST common side effect of the narcotic (opiate) analgesics is NAUSEA. Narcotic analgesics DO NOT cause peptic ulcers (steroids) or insomnia. - BB 214. Light pass through to through? (craze line / crack) 215. Not in the first line of tx of periodontists? (antibiotic/surgery) 216. Not feature of modify Widman flap? (displace/no reduce of osseous defect) 217. Most common cause of amalgam failure is? contamination 218. Doing FMX and u charged the patient for each one is: unbundling 219. If u reject null hypothesis and p = 0.08 (type 1 error) 220. Wheelchair (I picked something with sliding) - Important points: 1. Two people required for shifting patient. 2. Do not pull patient from behind chair. 3. Lock chair wheels before shifting. - Sliding board is the best 221. Child with furcation involve in tooth number E best treatment is (extraction)? – According to Mosby à if it is a restorable 2nd primary molar and there is no root resorption, tx of choice is pulpectomy. We need to know the child’s age too. If it is a 1st molar w/ furcation involvement then we talk about extraction. 222. Best test to test tooth with crown (cold test) 223. Worst media to save avulsion tooth (water) 224. Union between two teeth by the mean of cementum is (concrescense) 225. Stimulated salivary flow (1ml/min)-1L in 24h 226. Unstimulated (0.1ml/min) no 0.2 or 0.3 227. Trapezoidal mouth and shovel incisor? Apert - Oral manifestations of Apert syndrome includes trapezoidal shaped mouth, shovel shaped incisors, Byzantine arch shaped palate, severely delayed eruption, severe crowding and ectopic eruption. - BB 228. Baby bottle decay affect mostly (max ant) 229. Most common cause of sealant failure - Contamination with saliva 230. Most beneficiated tooth with sealants? Maxillary 1st molar 231. NaF for how many minutes should be applied – 4 minutes 232. Most common congenital missing primary tooth – primary maxillary lateral incisors 233. Missing permanent - Most common congenitally missing tooth is the mandibular second premolar (excluding 3rd molars), followed by the lateral incisor, followed by the maxillary second premolar (Mosby) 234. Most common primary ankylosed tooth? Primary 2nd mandibular molar – now you know why J 235. The smallest primary tooth is - Primary mandibular lateral incisor 236. Mesial cusp ridge is longer than distal one in - Maxillary 1st pm, also in primary max canine. 237. Which stage abnormality may cause peg shape lateral – bell stage (morphodifferentiation) 238. Two canals mostly in which pm - maxillary 1st pm 239. EDTA I picked can remove THIN calcification not any calcification (Thin layer of calcification) 240. Question about pulp diagnosis (pain on percussion) not respond to thermal (I picked d) A: abscess with irreversible B:reverible pulpitis C:periodontitis D: non of above 241. No generation after endo for: dentin 242. Remineralized enamel is harder and darker – T 243. 245 longer than 330 – T 244. Change amalgam to composite (I picked veracity) - T 245. Class 2 caries triangle and the apex to the pulp – T 246. Cusp reduction resistance form – T 247. Most lab complain from: under prepared – T 248. Composite and bleaching wait 1 week before composite – T 249. Ameloblastoma from okc (light microscope) 250. Which of these will not cause SICKLE CELL anemia crisis – NO2 251. Stridor (larangyospasm) 252. Seizure due to (hypo-Na) - hypoNAtremia 253. Initiator of light cure?? 1/Camphoroquinone 2/ benzyl peroxide 254. Albuterol side effect? tremor, anxiety, headache, muscle cramp, DRY MOUTH, tachycardia. 255. Albuterol side effects except? a. excess salivation b. tachycardia c. diarrhea. d. CNS stimulation. 256. Ledronate use in treat which disease - Pagets, osteoporosis 257. N2o side effect: Nausea and vomiting 258. Pt on Biophosphonate what to do? endo 259. With the increase in age, keratinization of the gingiva: decreases 260. Width of attached gingiva with age – increases 261. 40 years male how to correct cross bite ? Surgery 262. Not feature of modify Widman flap (displace/no reduce of osseous defect) – bony defects can be curetted (Oxford) 263. Most common respiratory problem in dental clinic (hyperventilation / no asthma in the choices) 264. Target in x Ray? anode and tungsten 265. Dementia? Short term memory loss 266. Amantadine: antiviral (influenza A) and antiparkinsonian drug. 267. ADHD? Which drug – Amphetamine (Adderall) and methylphenidate (Ritalin) 268. Large composit and acceptable appearance what to do? Tint 269. More affect perio? Smoking 270. Distobuccal complete denture? Masseter 271. 0.01? Type 1 error 272. Least Ab use? Chronic perio 273. Lisinopril moa? Inhibition of the Angiotensin-Converting Enzyme. Angiotensin- Converting Enzyme (ACE) Inhibitor: interfere with the conversion of Angiotensin I (weak vasoconstrictor) into Angiotensin II (highly effective vasoconstrictor that simulates the release of Aldosterone) by inhibiting the Angiotensin-Converting Enzyme. - BB 274. What will not set off an event in a child with sickle disease: Trauma, Cold, Infection, Nitrous oxide 275. Will have wavelength ? HUE 276. Pt mouth breather? Open bite 277. Most tooth affect perio? Max molar (Max 1st molar – most difficult) 278. Distance btw Implants? 3mm 279. Down syndrom = 280. ANUG treatment – debridement, hydrogen peroxide (or warm saline) rinses, and antibiotic therapy (penicillin V) ONLY if there is systemic involvement (i.e. fever, malaise, lymphadenopathy). Patients with HIV-associated ANUG require gentle debridement and antimicrobial rinses. 281. Max sinus x Ray? Waters, Ct, Both in op 282. Lefort 1? Max sinus involvement 283. Atenolol which receptor? Atenolol (Tenormin) - competitive b1 cardioselective antagonist that blocks b1 receptors to treat hypertension, chronic angina pectoris, or after a heart attack (MI recovery). 284. Flouride which ion? OH- ion (hydroxil ion) 285. The color of gingiva is due to: A. Capillaries B. Thickness of epithelium C. Thickness of keratinization and pigmentation D. All of the above 286. While u taking pano u patient move? Horizontal overlap 287. : Is permeable 288. Least test for recent Truma ? EPT 289. Most common cause of xerostomia? Medication 290. Dentogingival unit comprises of: and junctional epithelium 291. Free gingival groove represents: Histologic depth of 292. Which type of cells are most numerous in gingiva? Keratinocytes 293. Gingiva is supplied by: Supraperiosteal vessels 294. The junctional epithelium is attached to the tooth by: Basal lamina 295. Gingiva is attached by: Junctional epithelium 296. The length of junctional epithelium is: 0.25-1.35 mm 297. Least width of attached gingiva is found on the facial aspect of: First premolar 298. Normal consistency of gingiva is: A. Soft B. Hard C. Firm D. Firm and leathery 299. Gingiva in children: Less keratinized, less stippled 300. Eulanin fibers are found in: A. Gingiva B. Cementum C. Alveolar D. Periodontal ligament 301. The function of Langerhan's cells is: Antigen presentation to lymphocytes 302. Langerhan's cells are absent in: Junctional epithelium 303. Which periodontal fibers are consistent and are reconstructed even after the destruction of the alveolar bone? Transeptal 304. In periodontal ligament, there is: type I collagen 305. Periodontal ligament is: narrower on mesial surface 306. The periodontal ligament: contains epithelial cells of Malassez 307. PDL is thinnest at: Middle 308. What type of fibers are principal fibers of PDL? Collagenous 309. Periodontal fibers which are most resistant to forces along the long axis are: Oblique 310. The thixotropic theory claims that: The periodontal ligament has the rheologic behavior of a thixotropic gel 311. Alveolar bone is: Compact bone 312. Anatomic form of roots of teeth is determined by: Hertwig's root sheath 313. “Indifferent fibers" are: Collagen fibers 314. With aging, changes in periodontal ligament are: A. Decrease in number of cells and increase in collagen fibers B. Increase in number of cells and decrease in collagen fibers C. Increase in number of elastic fibers D. Hyalinization changes 315. Feature of aging is: Scalloping of cementum and alveolar bone surface 316. Unattached gingiva: A. is interdental gingiva B. is below mucogingival fold C. cannot be separated by probe D. is marginal gingiva 317. A black line on the gingiva which follows the contour of the margin is due to: A. lead B. Argyria C. Iron D. Mercury 318. Basal lamina consists of: Type IV collagen fibers 319. Cementum found on the cervical third of the root is: Acellular extrinsic fiber cementum 320. The cell that is present in stratum spinosum and stratum basale is: Melanocytes 321. What make penicillin allergic - Beta lactame ring 322. Space loss after loss of which primary tooth - Mand 2nd molar 323. Composite discolored but intact what to do - Redo or polish (go w/ Tint if in options) 324. What happen if temp of developing solution is too high - Dark 325. When you put occlusal rest set of direct reatiner mesial or distal to edentulous area? mesial 326. Inc water powder ratio in gypsum does what: Decreased setting expansion, Decreased strength, increase working/setting time (retards the time) 327. Which bur is not good for porcelain – Carbide 328. Advantage of implant over fix partial denture 329. Best way to dec fear of child – TSD 330. Best way to dec fear of child ..I said sit down to child's level 331. Dentist ask a child u want me to help you to sit on the chair ...how to define this situation: one option was perceived helplessness 332. Tooth most involved in VRF - Mand 1st molar 333. Advantage of stainless steel over ni titanium – strength 334. Ledge what to do - bypass it and continue 335. Collagenase and elastase by which bacteria - Porphyromonas gingivalis 336. LOW WEAR RESISTANCE is the property of filled resins that is primarily to blame for the failure of Class II composite restorations – T 337. Common reason for failure of composite in posterior - Saliva contamination or occlussal wear (recurrent caries according to other books) 338. Anterior teeth heavily damaged what do you do - PFM or all ceramic crown 339. What can't be used as retainers in FPD: inlay 340. Primary tooth with shortest occlusal table - Upper primary first molar 341. Pt came back after a month with discolored margins what could be the reason – microleakage 342. Pt came back after 3 day with discolored margins on veneer what could be the reason - Amine or micro-leakage or bacteria 343. Function of post - Retain core 344. Why you record protusive relationship - to adjust condylar guidance A protrusive record registers the anterior-inferior condyle path at one particular point in the translatory movement of the condyles. - Mosby 345. A dentist is preparing all maxillary anterior teeth for metal-ceramic crowns. Which of the following procedures is necessary to preserve and restore anterior guidance? A. Protrusive record B. Template for provisional restorations C. Custom incisal guide table D. Interocclusal record in centric relation - Anterior guidance must be preserved by means of construction of a custom incisal guide table, especially when restorative procedures change the surfaces of anterior teeth that guide the in excursive (lateral, protrusive) movements. - Mosby 346. Where to place retentive arm - Retentive terminal alone below ht of contour at junction of middle and gingival third (FB group discussion) / Gingival one third of crown in measured undercut (Mosby) - The reciprocal clasp should contact the tooth on or above the height of contour of the tooth (Middle one third of crown). 347. Reciprocal arm what for – Stabilization (resistance of horizontal forces), reciprocation, and auxiliary indirect retention (bracing). Placed on Suprabulge area. 348. Reciprocal anchorage? - Elastics to close diastema? No - Reciprocal tooth movement—two equal anchorage value teeth or groups of teeth (units) are moved against each other and move the same amount toward or away from each other. - Reinforced anchorage—adding additional teeth to a unit to distribute the force over a greater area and slowing the movement of the anchor unit. Another method for reinforcing anchorage would be extraoral force, such as with headgear, with interarch elastics, or by using an implant. 349. Behaviour modifiaction definition? Behavior modification is a treatment approach, based on the principles of operant conditioning, that replaces undesirable behaviors with more desirable ones through positive or negative reinforcement. 350. Child lives in fluoridated area he had a lot of plaque what do you suggest - More systemic fluoride or topical fluoride or fissure sealants. 351. Topical antifungal options fluconazole clotrimazole miconazole griseofulvin 352. Antifungal that can be topical and systemic. fluconazole clotrimazole miconazole, griseofulvin - Miconazole Therapeutic Uses: is an azole antifungal drug available for topical and systemic administration - BB 353. Does premedications required options were cardiac stent murmurs pt had knee replacement within 2years or congestive heart failure or recent MI 354. Bacteria present in gingiva in ANUG when tissue is not necrosis - Spirochete or P.interdemdia (P intermedia only, necrosis assoc with spirochete- DD). 355. Pt is having asymptomatic brown macules on buccal mucosa - Increase melanin or melanocyte proliferation. - “Melanotic macules: These lesions can also occur within the oral cavity, commonly gingiva, buccal mucosa or palate. The cause increased melanin production with NO increase in melanocyte number.” 356. Least resistant to fracture - Leiutic or feldspathic 357. How morphine cause nausea - Centrally acting right ? – True 358. Probing depth is 5 and prob pass 2 mm apical from CNJ how much is the attachment loss – 2 mm 359. Which structure is damaged during free gingival flap surgery taken from palate: Greater palatine artery or nerve 360. The reason of high pressure in pregnant women – Pre-eclampsia 361. After IAN block patient gets infection of which space – Ptergomandibular (masticatory) or messeter or lateral pharyngeal or medial ptergoid 362. A lot of translucency on pontics gingival 3rd what is the reason- wrong shade selection or wrong metal thickness or inadequate preparation 363. Excessive translucency of the gingival 1/3 of an anterior metal ceramic pontic is caused by: 1. Inadequate framework design 2. Error in shape selection 3. Poor ridge contour. 4. The palladium content in the gold alloy. 364. On x-ray of max tooth RL between the margin of a crown and tooth on distal proximal surface. The reason of RL can be all, except: resin, cervical burn, cervical caries. (Other options I do not remember). 365. Preloading of implant whats the major concern – Torque 366. Pt wit gag needs to take x ray? How to manage that - Systematic desensitization or graded exposure 367. Folic acid intake in pat for? – no answer options - Folic acid is commonly prescribed for patients with sickle cell anemia to prevent development of megaloblastic anemia - Folic acid to prevent neural tube defects. 368. Abscess can be released from perio pocket or not – YES 369. What do you say if patient ask about instrument sterilization - its according to universal precautions 370. Gtr best for - GTR best for 3 wall defect and also class 2 furcation 371. Does anyone know from Class 1 till 4 furcation the treatment plan? Class 1 furcation - good prognosis Class 2 furcation – GTR Class 3 furcation ....in maxilla - Root amputation ....in mandible - Hemisection Class 4 furcation – Extract 372. Pain medication for alchoholic - Nsaid or oxycodone 373. Most common emergency in dental clinic - syncope 374. Most common resp emergency in clinic - hyperventilation 375. Function of major connector: Stability or rigidity. 376. Can we give lorazepam in liver disase? - LOT - lorazepam, oxazepam, Temazepam - good when liver problems. 377. How to determine periodontal success - No bleeding on probing or establish clinical attachment? - The BEST CRITERION to evaluate the success of SRP is NO BLEEDING ON PROBING (since BOP indicates active inflammatory ). Amount of attachment loss is the most important factor in the determination of a prognosis of a tooth with periodontal disease (more accurate than probing depth, tooth mobility, and presence of furcation involvement). à Long term prognosis = plaque - Attachment loss à Prognosis of tooth w/ periodontal disease - Bleeding on probing à periodontal success - Plaque à Long term prognosis 378. TWO MOST CRITICAL FACTORS to determine the prognosis of a periodontally involved tooth are MOBILITY & ATTACHMENT LOSS (the most critical). 379. Who review patient on maintenance after referral with periodontal treatment: Dentist or hygienist or dental assistance. 380. Drug for neuogenic and manic disoder.. Tegretol (carbamazepine) or lithium 381. Viral load of HIV 10000 do you treat the patient or referral to specialist – refer 382. Abscess can be relieved thru perio pocket.T/F 383. Feeling of been in the doom - Panic attack or anxiety or simple fear 384. Tongue blade appliance is used in? – anterior cross bite 385. Pt recive blow to eye orbital floor less common to fracture t/False 386. Pt who took treatment for hep A before 3 years so he still contagious or NO 387. What drug (pain) you give to pt with renal failure: Tylenol nsaid codien morphine 388. Teeth with necrotic pulp and perio damage. Only RCT or perio treatment too? Only RCT 389. At least 4mm of gutta-percha MUST remain to preserve the apical seal. 390. Face bow transfers relation of arches? In centric relation, In centric occlusion 391. Most common site(s) for contact – Side of tongue, , Gingiva, All 392. Best radiograph for implant? CT scan. PA. MRI 393. How to treat oropharyngeal candidiasis in HIV patient - Topical or systemic, (fluconazole). Esophageal and pharyngeal candidiasis is related with AIDS. à topical antifungal (clotrimazole or nystatin). – Kaplan Cases 394. You have HIV+ pt you can do all of the followin except a) treath with metronidozole b) free gingival graft c)prophylaxix to treat candidiasis “Data from prospective controlled trials indicate that fluconazole can reduce the risk of mucosal disease (i.e., oropharyngeal, esophageal, and vulvovaginal) in patients with advanced HIV. However, routine primary prophylaxis is not recommended because mucosal disease is associated with very low attributable morbidity and mortality and, moreover, acute therapy is highly effective. Primary antifungal prophylaxis can lead to infections caused by drug-resistant Candida species and introduce significant drug-drug interactions. In addition long-term oral prophylaxis is expensive. Therefore, routine primary prophylaxis is not recommended (AIII).” 395. Permanent tooth with largest occlusal table – maxillary first molar. 396. Epithelium comes from the donor site – T 397. Dexterity- Something to do with flossing -> 5 brush and 8 floss. If lack of dexterity can’t do interproximal floss – T 398. Perio v/s endo abscess- pulp testing, lat percussion 399. Opaque porcelain function mask dark oxidized color – T 400. Methotrexate- anti cancer drug folic acid inhibitor – T 401. What is battery? Treatment without informed consent 402. Autonomy - Let the patient decide. 403. Ectodermal dysplasia oligodontia, sparse hair - or oligodontia, depressed bridge of nose, lack of sweat glands, and the child appears much older than he or she is. (DD) 404. Diabetes commonly found in which ethnicity? Black male 405. Periodontis most Common? Black male 406. What causes cervical discoloration of PFM copper, ag. Plaque 407. Flame shaped radiolucency above an unerupted third molar – pericoronaritis 408. Safe drug in pregnancy acetaminophen 409. Safe drug in breastfeeding promethazine 410. Prolonged use of Aspirin can cause metabolic acidosis 411. Fusion, Gemination - Fusion less number and gemination extra number or one big crown, Fusion 2 buds fuse together and form one tooth.. from total no of teeth, there is decrease in number. 412. Failure of which stage of tooth development affect Number of teeth – initiation 413. Size – morphodifferentiation (bell stage) 414. Which disease lead to MI - thrombosis(I pick atherosclerosis,but I think it is thrombosis, other two wasn’t relate - (MI usually caused by thrombus formation) 415. Acute adrenal insufficiency : a. gingival hyperplasia b. cardiovascular collapse c. hypotension d. ketoacidosis - ASDA 416. Low contrast- kvp (Long scale, low contrast, hight kvp) 417. Which doesn’t recur- AOT 418. Antibiotic treatment- LAP -> Tetra 419. Seizures grand mal phenytoin (and Carbamazepine – Tegretol)- 420. Petit mal – ethosuximide - Ethosuximide in the treatment of absence (petit mal) seizures. 421. Overdose cocaine mydriasis 422. Overdose opioids miosis 423. Short clinical crown- what to do- read from prostho decks, proxy grooves if lack F-L resistance (buccal – retention / proximal – resistance) 424. Pedunculated leision – papilloma 425. Dry socket sedative = dressing 426. Benzoyl peroxide decomposition by tertiary amine in chemically activated resin self cured - a chemical activator like dimethyl-p-toluidine (a tertiary amine) is added to the monomer (MMA) to decompose the benzoyl peroxide initiator into free radicals. 427. Excess bilirubin in blood? kernicterus 428. Epi reversal? Phentolamine – Phentolamine (OraVerse) is used in dentistry to reverse soft tissue anesthesia more quickly after procedures involving a local anesthetic with a vasoconstrictor. Alpha adrenoceptor blockers, such as phentolamine, inhibit the vasoconstrictor effect of epinephrine but not the vasodilator effect of epinephrine. The administration of alpha blockers results in epinephrine reversal. (Mosby) 429. Pt shows reaction to the LA vasoconstrictor so to recover the reactions u will administer: Alpha 1 antagonist - the ZOSINSSSS 430. Which of the following drugs is most effective as an antidepressant? a. Diazepam b. Reserpine c. Amitriptyline 431. Tricyclic antidepressants have a prominent side effect that most nearly resembles the usual pharmacological action of a. Codeine b. Atropine ans c. Ephedrine 432. If you have two distribution that are asymmetrical that’s means a) normal B) skewed is the ans c) bimodal 433. Meds not given to a pt who has epinephrine - MOA and Tricyclic antidepressant (Also Levodopa) 434. Complication of temporal arteritis - blindness 435. Gtr best for – 3 narrow wall defect 436. If the patient has maxillary protrusion and we need to pull the maxillary backward which we will use a) straight pull gear b) cervical pull gear c) face bow d) reverse bull gear Cervical-pull headgear à consists of a cervical neck strap (as anchorage) and a standard facebow inserting into the headgear tube of the maxillary first molar attachments. The objectives of treatment with these types of headgear are to restrict anterior growth of the maxilla and to distalize and erupt maxillary molars. A MAJOR DISADVANTAGE OF TREATMENT USING CERVICAL HEADGEAR IS POSSIBLE EXTRUSION OF THE MAXILLARY MOLARS. 437. What make the reaction on the one that has monomer and activator that was using it for prepaing the secondary impression on the study cast on the lap . its activated by A) exothermic reaction b) MMA with the monomer 438. Gtr-- involves coronal movement of pdl 439. Antidepressants serotonin / SSRI-fluoxitene zoloft,both- SNRI TCA-->Amitryptilline 440. Pain medication for alchoholic – OXYCODONE 441. Fetal alcohol syndrome - Mid face deficiencies, cleft lip (Individuals with fetal alcohol syndrome may have cleft lip with or without cleft palate) 442. Antibioticis of maythenia gravis – penicillin safe (erythro increases weakness) 443. If no enough space what tooth pop out of arch maxi and mand. – First premolars 444. B blockers - Glucagon 445. Acetamenophen - N Acetyl 446. Xylitol is best used anticariogenic when it's a: mouthwash, tablet, chewing gum, varnish 447. Xylitol gum used for :- - DECREDSED SALIVATION (xerostomy) - PREVENT CARIES 448. FDA. determines which drugs are to be sold by prescription only: Schedulle 2 and 3 must have a written prescription. (The only difference is that class 2 cannot be refilled whereas class 3 can be refilled even over the phone.) 449. Contraindication of RCT - vertical root fracture (also Uncontrolled DM, Recent MI, Leukemia) 450. Resistance for short crown proximal grooves 451. Laser in periodontal diseases use for sulcular debridment 452. Meperidine? sintetic opioid, narcotic, less potent than morphine, more than codiene, shorter duration of action, interaction with MOA can cause convulsions. 453. Which study can show incidence – descriptive (cross sectional shows prevalence Cohort is for incidence) 454. Theory of stress and how affect immunity – Stress induce cortisol and reduce immunity 455. Objective fear after previous painful stimulus (own experience) Objective fear is one that you experience directly. If, for example, you had a painful dental visit while having a tooth extracted; the next time you needed a tooth extracted you would be fearful because of this prior experience. If you told your friend about the bad experience and he needed a tooth extracted, and he had fear, this would be subjective fear. 456. Thyroid crisis symptoms hyperT HR increased (High Bp, high body temperature, persistant sweating tachycardia, high fever) 457. Hypoxia sign – cyanosis (Cyanosis and increase in pulse rate too - DD) 458. Signs of oxygen want 1. Cyanosis 2. Pulse rate decrease 3. Tachycardia 459. Antibiotic in gingiva tetracyclines 460. Wheelchair Q – sliding 461. How base metal prevent corrosion – chromium 462. What is the most common psych disorder? Depression 463. Leakage in amalgam decrease with age 464. Systemic desensitization - hierarchy of slowly increasing anxiety stimulus 465. 16 kg anasthesia calculation 70,4 mg 466. Most common emergency in dental clinic - asthma or syncope 467. Most common respiratory emergency it is hyperventilation (no asthma in options) 468. Most common respiratory emergency it is Asthma...I was so happy no hyperventilation in the options 469. Primary stress bearing areas in dentures - max - primary ridge ...secondary rugae mand - primary buccal shelf - and also primary if good ridge 470. Primary areas of support in max and mandibular. Thought we’re maxillary ridge and buccal shelf for mand but options were not together: For mandibular complete denture, the residual ridges if large and broad, are also considered primary support areas. 471. In USA most dental pay is - out of pocket self pay 472. Which is more important? Chroma, value (value is VIP and choosing process is HUE first then VALUE then CHROMA) 473. Which show saturation of color? – chroma 474. Radiation water lysis - Hydrolysis of water 475. Facebow transfer - hinge axis 476. Arcon vs non-acron - Arcon resembles tmj 477. Dental lamina appears at which week 6 th week 478. Distolingual extension of mand, denture which muscle sup constrictor 479. Incisal edge of anterior teeth touch where? 480. Bur for burnishing porcelain = diamond 481. Sodium hypochloride does all except (Chelation) 482. Impresion material moisture tolerant – Polysulfides (because they are hydrophobic). 483. Polyvinyl siloxanes - excellent DIMENSIONAL STABILITY and very low permanent deformation.

484. Behcet's syndrome associated with aphthous ulcers 485. High school children have marginal 486. Ginseng contraindicated with salicyclic acid 487. Erosion – GERD 488. Initiation: supernumerary/missing teeth 489. Bell - 2 types : morphodifferentiation: shape!!! histodifferetioation: d.imperfecta a.imperfecta 490. Apposition: enamel hypoPLASIASSS 491. Moisture contamination in amalgam a) Increases delayed expansion b) Something related to amalgam strength DELAYED EXPANSION of amalgam restorations is associated with insufficient trituration & condensation, and amalgam contamination by moisture during trituration and condensation (MAIN cause of failures). - BB 492. Central giant cell granuloma is seen with pts with condition of /? Hyperparathyroidism - Brown tumor of hyperparathyroidism masquerading as central giant cell granuloma in a renal transplant recipient. 493. Which cement causes irritation to pulp – zinc phosphate 494. Sialilitithiasis is found whew = warton duct ( was as an oprtion) - calcified mass or sialolith forms within a salivary gland, usually in the duct of the (also termed "Wharton's duct"). 495. Kid came to clinic touched everything and last he came to dentist and allowed to keep probe in the mouth what the behavior? – Desensitization 496. Digoxin – it is used for congestive heart disease, atrial fibrillation and when you drink in conjunction with diurectic can increase the toxicity of digoxin 497. Contraindication for nitrous oxide (nasal congestion) 498. Best place for implant ant mand 499. Solution to keep an avulsed tooth hanks worst -> water 500. Aplastic anemia -> chloramphenicol 501. Pseudomembranous colitis (clindamycin) 502. Therapeutic effect (safety) 503. No to do w asthm - Give Oxygen?? – We can give oxygen in asthma (acute attack - give oxygen and albuterol / severe asthma ONLY – epinephrine) 0.3mg OF 1:1000 EPINEPHRINE SHOULD BE ADMINISTERED SUBCUTANEOUSLY 504. No contraindication w asthma - Nitrious oxide 505. Gingival graft contraindications (pocket below alveolar crest) – A FGG is used to increase the zone of attached gingiva and possibility of gaining root coverage. NOT used with DEEP WIDE RECESSIONS. 506. Gingival graft contraindicated when: a) pocket is below the alveolar crest b) pocket is below free gingival groove c) excessive keratinized tissue 507. Which LA is good without vasoconstrictor? Mepi 508. Bipolar disorder lithium (LITHIUM - current drug of choice to treat the MANIC PHASE of bipolar disorder) 509. On which receptors Epi works a1 a2 b1 b2. I choose a1 but not sure – We know it is all but it affects beta receptors predominantly. 510. Morphine overdose = naloxone 511. Porcelain porosity = Inadequate condensation 512. Unbundling - when doctor describes the whole treatment in different parts, separate charge code instead combine. 513. Most commonly used TCA Amitryptyline – AMITRIPTYLINE (ELAVlL) - the most widely used TRICYCLIC ANTI-DEPRESSANT to treat unipolar disorder (depression). 514. Cocaine produces vomiting by = activating CTZ in brain 515. Ques about H1 and H2 receptor --> H1 smooth muscle, H2 gastrointestinal 516. Down Syndrome – macroglossia 517. Ectodermal dysplasia= scarce hair 518. Patient smokes pipe and has red bumps on palate= Nicotine stomatitis (hard palate) 519. Arch discrepancy after loss of which tooth = Mand 2 nd molar - The premature loss of the mandibular primary canine reflects insufficient arch size in the anterior region. 520. Which is NOT used to inhibit salivary secretion = Pilocarpine 521. Osteogenesis imperfecta with = DI 522. Rapport active listening 523. Caries not depend on quantity of carbs 524. Radiograph id tip of the nose, external auditory meatus 525. Cavernous thrombosis infection via anterior triangle 526. Tooth mostly involved in perio relapse Max 2nd molar – Furcation involvement of maxillary 2nd molar has the poorest prognosis following therapy / Trifurcation on maxillary 1st molar are the most difficult of all to root plane (DD) 527. Warfarin test – INR/PT 528. Why you do not use fovea as indicator - it blocks minor salivary glands 529. Sausage like appearance on radiograph or sialodenitis (I pick the first one, not sure) – : within the gland sausage-link appearance. Sialodochitis (also termed ductal ), is inflammation of the duct system of a salivary gland. 530. Antiretraction valves prevent cross contamination (between patients) 531. Hepatitis A positive can checked by a) HBcg b) HBSg group c) no treatment d) alkaline triphostate - (Hep A infection is identified by HAV-specific antibodies (IgM if acute, IgG if past disease) 532. Composite and bleaching wait 1 week before composite 533. Which is a characteristic of a gold inlay? A: Axial walls converge toward the pulpal floor 534. Ortho treatment before veneer placement 535. By having excess amount of monomer in acrylic can create excessive amounts of what: shrinkage 536. Dementia pt, which one the most challenging? 1-ext 2-denture is the ans 3-srp 4-amalgam 537. If you have 2 proximal cavities a) you will fill smaller, prepare larger first ans b) you will fill larger and prepare the smaller first C) you will fill smaller and prepare the smaller first d) you will fill the larger and prepare the larger first 538. Pin retained - Which statement is not correct? increasing in number of pin strengthen amalgam 539. Best biopsy for small bean sized white lesion? a) Incisional b) excisional is the ans c) brush 540. Steroid dose need medical consultant 20 mg for 2 weeks 2 year 541. Osteosarcoma = PDL widening 542. Patient presents with blow to face and horizontal fracture of a previously endodontically treated molar. The fracture is 2mm from the anatomical crown. What should the dentist do? a. Treat root canal with Calcium Hydroxide b. Place temporary crown and revisit in 3 months c. Immobilize d. Ext ans 543. Pt with MOD done 1 month ago, complained it hurt when he bits A) polymerization shrinkage B) leakage c ) cracked tooth is the ans 544. Most common Impacted tooth - Mandi 3m , maxi 3m, max canine 545. Ludwig angina spaces – Submand, Sublingual, Submental 546. Xerostomia due to medications 547. Most common cyst = periapical - RADICULAR CYST (APICAL PERIODONTAL CYST OR ) - MOST COMMON - BB 548. Warthin tumor which gland affects – parotid 549. Sulfur granules and lumpy jaw = actinomycosis 550. Most common infectious stage in syphilis – secondary 551. Hand and foot lesion which virus – coxsackievirus 552. : involves the hard tissues that are derived from both epithelia (enamel) and mesenchymal (dentin and cementum). The teeth in a region or quadrant of the maxilla or mandible are affected to the extent that they exhibit short roots, open apical foramina, and enlarged pulp chambers – ghost teeth. The cause is unknown and because of the poor quality of the affected teeth, their removal is usually indicated. 553. Which of these has pain same to tooth pain (hsv/VZV/max sinusitis) - (vzv) can mimic tooth ache. 554. How to differentiate ANUG from primary herpetic gingivostomatits - Intact interdental papillae in herpes 555. Question about lesion in eye has a name subnour something like this i picked phemphigus but it is mmp - The most common and feared diagnosis associated with symblepharon is Pemphigoid (MMP) 556. Cobblestone appearance – Papillary hyperplasia 557. Which taste bud affected in - Filliform 558. Multiple myeloma starts = bone pain 559. Not vasodilaltor = cocaine 560. Not a(alpha) blocker = metoprolol 561. No nitrious oxide = in 1 st trimester 562. Cooling while implant placement - The bone is prepared with special metallic burs driven at slow speeds with copious water- cooling 563. In implant preparation, which of the following can be used? A) hydroxyapatite irrigation b) High Speed Hand Piece c) Low torque d)Air Coolant 564. Crevicular fluid cells PMNs 565. Opioid= mu receptors 566. Full dentures clicking = increased VDO 567. Carcinoma vs carcinoma in situ = no invasion 568. Fungal infection = nystatin 569. Systemic fungal = fluconazole 570. = carcinoma in situ – Like "leukoplakia", ERYTHROPLAKIA has no histologic connotation, but most erythroplakias are histologically diagnosed as severe epithelial dysplasia, carcinoma in situ, or invasive squamous cell carcinoma. - DD 571. Open bite= LeFort 1 572. Most allergic metal = nickel 573. FRANKFORT-HORIZONTAL PLANE - constructed by drawing a line connecting PORION & ORBITALE. 574. Labiodental sounds and what do they determine? Labiodental sound (fricative)- f,v,ph, formed by maxillary incisor contacting the wet/dry line of mandibular lip. This sounds help determining the position of incisal edge of maxillary anterior teeth. 575. Open apex tx: Vital apexogenesis / Non vital apexification 576. Mouthguard= MPDS 577. Polyether= sticks to teeth 578. RPD connector fracture = do soldering 579. Traumatic neuroma= mental nerve region 580. Primary mand 2 nd resembles - permanent mandibular first molar 581. Access opening for mand molar – Trapezoid 582. MWF = reduce pocket LINING 583. No gingivoectomy = with thin gingiva (An adequate amount of attached gingiva must be present before a gingivectomy is done, otherwise the result will be an area with minimal or no attached gingiva.) - DD 584. Drug testing = clinical trials 585. Where you give GA? 2 year old kid needs lot of restoration 586. Fearful patient how you respond? intoduce tools or TSD 587. Introduce instruments and tools = Desensitization 588. Smokeless tobacco - – verrucous carcinoma (Associated with smokeless tobacco habit).- DD 589. Fracture w paresthesia= angle of mandible 590. Minimum amount gutta percha left in canal after placement of post? 2 or 3 or 5 mm (IT’S 4mm). 591. Buccal-lingual bone width necessary for 4 mm implant. NO 6mm in options! I took 7mm. 592. Battery - Treatment without consent form 593. Pt with dialysis, when to do treatment? 1 day after dialysis 594. Harder area to floss - Mesial maxillary 1 pm 595. If open bite 8 mm, which kind of treatment do you do? Surgery, LeFort 1 596. Amalgam failure - THE CONTAMINATION OF THE AMALGAM BY MOISTURE DURING TRITURATION AND CONDENSATION IS UNQUEATIONABLE THE PRINCIPAL CAUSE OF FAILURES (DD). 597. Question about pigmentation in options was Neurofibromatosis and Preuz- Jeghers. Pay attention to cafe-au-late spots! Both have pigmentation, but only Neurof. has cafe-au late! – Café-au-lait found in VON RECKLINGHAUSEN'S DISEASE (NEUROFIBROMATOSIS) and Albright's Syndrome (McCune-Albright Syndrome) 598. Osteoradionecrosis? More in mandible/ more in maxilla/ 42 gy? According to dd its more than 40 gry and more in mandible (controversial) 599. How treat root caries? Composite, amalgam, GIC 600. Best filling for class 5 – GIC 601. Mucocele where most common? Upper lip, lower lip, side of tongue 602. Ginco Biloba contraindication? all anticoagulants 603. Cleft lip which embryo week? It was 6-9 the only reasonable option I took cleft lip 6 weeks - 7 weeks cleft palate 8-12 weeks - decks 604. Bleeding few days after extraction? fibrinolysis. 605. Transillumination: craze - full tooth illuminate / crack - light stopped - not full tooth / sialolith - in children 606. When do you use Laser or Electrocoagulation surgery intraorally? Only logical option was capillary bleeding! 607. Motion sickness. Treatment: scopolamine 608. When is gingiva inflammation least expected? I was torn between Xerostomia and Lack of keratinized gingiva. I chose lack of keratinized gingiva, although in Dental School we learned, that when keratinized gingiva is missing, gingivitis is programmed! But, Xerostomia sounded worse, so I took lack of keratinized gingiva! 609. Forceps for upper PM - 150 and 150A 610. What is not true about Xerostomia? Few very obvious true options, I chose it is extremely difficult to treat it with Saliva substitutes and inducers. 611. Q. about V-form caries, tip towards the DEJ (interproximal/smooth surface) 612. Burn mouth syndrome due to: candida 613. Warthin tumor – 2nd most benign salivary gland tumor more in and more common reason is smoking. 614. Q. about Pemphigoid - I think, there was an option with basement membrane and hemidesmosomes 615. All are advantages of per oral sedatives in the dental clinic except? There was an option about GI absorption, and well controlled effect, that I chose. 616. Best topical Antibiotic agent? Options were H2O2, CHX, Ethanol and something else. I chose CHX 617. 3 Qs about TAD (=Temporary Anchorage Device). Where it gets its stability from? stability from cortical bone and increase stability if placed perpendicular. – The primary stability of miniscrews is a result of mechanical interlocking of the threads with cortical bone. Placement of miniscrews perpendicular to cortical bone to take advantage of biological and biomechanical stability when applying heavy orthopedic forces. 618. Tx for Status Epilepticus. NO Diazepam in options!!! There was Midazolam in the options, and I chose that! Phenytoin could be an option too. 619. 2 y.o. uncooperative child. What to do to examine? GA, Papoose board, Restrict by dental assistant, Restrict by parent. 620. Fluoride function on Enamel - reduces solubility 621. Most secure indicator for future caries? Bacteria, something else, salivary flow (the more the better) 622. How to avoid bites? increase the horizontal overlap in post teeth 623. Xerostomia causing drugs work on which receptor: Parasympathetic cholinergic 624. Drug conjugation. Add molecule to drug (to make it ionized so less fat soluble and more water soluble) 625. Wrought wire can “do something” around 0,2 mm to the abutment tooth if minimal wire length is 4, 6 or 8 mm? I took 4mm! I don’t have any idea what they were talking about 626. Whats the most favorable taper of abutment to make RPD in part of inch: 0.02- 0.01 - 0.03 - 0.039 627. Most common to see in a teenage girl with buccal erupted canine? Anterior deep bite, recession, 2 others – “…The former displayed significantly higher frequencies at adolescence of ectopic eruption (mostly maxillary canines), anterior cross-bite, extreme maxillary overjet, deep bite, and crowding.” 628. Abutment height in implant restorations selected according to? Only logical option was according to implant lenght! I immediately thought about the abutment- implant ratio! 629. Cocain effect? contraction of dilator pupillae, contraction of sphincter pupillae, 2 other stupid options. 630. Used as topical LA? Cocain, Benzo, Lido, Bupivacaine - Lidocaine is a local anesthetic drug used topically in dentistry - BB. 631. Face rash, defect on heart, kidney and blood vessels? 632. Cause for ? HIV, EBV, HSV, Candida (don’t confuse w/ hairy tongue) 633. Dry socket tx. - sedative, dressing no antibiotic, no curettage 634. Most difficult to maintain space? In a 9 y.o missing permanent MX M1 - 5 y.o missing primary MD M1 - 6 y.o missing primary MD M2, one more I don't remember. 635. After RCT on a tooth with sinus tract, what is the tx for the sinus tract itself. Funny options! We all know, that NO therapy is needed, right?! ;) 636. Disease with excessive bone production, but less bone resorption? Paget disease was the only logic option 637. Mouth breather have? Open bite, long face, deep bite etc. Open bite and long face 638. Typical face feature for muscle weakness? Same like mouth breather. They could be the best couple - long face, open bite 639. Kid with ADHD what tx.? Methylphenidate , Amphetamines 640. Incision for palate tori. Y incision – a double “Y” incision should be made over the midline of the torus. (DD) 641. Pain drug for the night? naproxen - 8 hour relief 642. Pt had tooth extraction want to sleep at night whst u give ? naproxen 643. Side effect of opioids, all except question. Side effects of opioids are sedation, dizziness, nausea, vomiting, constipation, physical dependence, tolerance, and respiratory depression. NO gastric ulcers, NO insomnia, NO somnolence, NO diarrhea. 644. Lateral positioned flap - the option I chose was to maintain or improve keratinized gingiva 645. Most common cause of amalgam failure à unquestionable moisture (DD) 646. Q. about rejecting null hypothesis. Type 1 error 647. What is the strongest point of statistics? 648. Selectivity / Sensitivity in clinical trials 649. Q. on clediocranial dysplasia – clavicle, supernumerary 650. In USA most dental payment – cash out of pocket 651. Restauration in anterior teeth done weeks ago, filling too light. What is the most conservative tx. Keyword here is conservative!!! - Tint 652. Which shows saturation of color? chroma 653. most seen in? male african american 654. Face-bow – hinge 655. In English the s/ch/sh sounds, how is anterior teeth position - very little distance betw. anterior teeth. 656. Major connector function - stability rigidity 657. Why do we do apexification? - non vital tooth, apical barrier for rct after 3 months if calcified (“induce root development” is apexogenesis for vital tooth, apexification for non vital for apical barrier!) 658. Most common seizure in children - febrile 659. Tx. for atypicaly erupting permanent tooth, with gingival margin above CEJ. Gingivectomy, Apical positioned flap, others (Apical positioned flap, to not waste keratinized gingiva) 660. Best graft material - autograft 661. Where do you get the most amount of graft material from? illiac 662. Pka has effect on - onset 663. All can be Dif. Diagnosis of Leukoplakia except. - I think I chose Verrucous leukoplakia, just because they look so different – is often mistaken for leukoplakia BUT appears early in life. Focal (frictional) keratosis: common white lesion caused by chronic friction on the mucosa. Differentiated from idiopathic leukoplakia because cause is known. disappears with stretching. (DD) 664. Collimation purpose - Reduce size of beam 665. Calculation of incidence index - Out of 1000 patients —-> 200 diseased patients last year, 300 this year. 300-200=100, 100/1000= 0,1 666. Kid with bad OH what ortho tx? Fixed, removable, no tx 667. Veneer facial reduction – 0.5mm 668. If a Porcelain to metal fused anterior crown too opaque, what”s the reason? less 2nd plane reduction 669. Perio-Endo lesion tx. – first endo 670. MRI how does that work?- Clinical magnetic resonance imaging (clinical MRI) is an imaging technique used in radiology to form pictures of the anatomy and the physiological processes of the body in both health and disease. MRI scanners use strong magnetic fields, radio waves, and field gradients to generate images of the organs in the body. MRI does not involve x-rays, which distinguishes it from computed tomography (CT or CAT). 671. Drug against kidney failure, what's the best to analyze and prove its action? Creatinine test 672. If lower complete denture overextended buccodistaly what structure impaired? masseter 673. In upper complete denture overextended buccodistaly what structure impaired? coronoid 674. Pseudomembranous colitis - Clindamycin 675. Patient has a palatal torus that extends beyond posterior palatal seal into the soft palatal area, and in need of complete denture. What to do? Remove it 676. Ignoring the pt bad behavior – extinguishing 677. Which tooth more prone to caries? Maxillary 1st molar (Kaplan) 678. High kV? Long wave lenght low energy; Short wave length high energy; 2 more combinations 679. Dexterity – floss (5 brush 8 floss) 680. Q. about a kid with Autism. Repetitive and Sensitive to light and high volume 681. Q. about ACE-blockers - It actually directly blocks the Angiotensin-Converting- Enzyme, not the Angiotensin - blocks the enzyme which converts angiotensin I to angiotensin II. 682. Actinomycosis – sulfur granules 683. Vertical root fracture most commonly seen? Mandibular molars 684. Crown to root fracture most commonly seen? Maxillary anterior 685. Perforation of anterior maxillary incisors during RCT access in general most common on which surface? Mesial - Maxillary Anterior teeth: Mesial perforation due to distal axial inclination. Mand. Ant: LABIAL perforation due to lingual inclination 686. Cast impression least acurate? Reversible Hydrocolloids, Irreversible Hydrocolloids, Polyether, one more 687. Q on Chronic Heart Failure. PHARMACOLOGIC THERAPY: DIURETICS (used to control fluid retention), ACE INHIBITORS (interfere w/ the renin-angiotensin system, are required of all patients w/ cardiac failure unless contraindicated), VASODILATORS (including hydralazine and nitrates, are used when the use of ACE inhibitors is not possible), BETA BLOCKERS (should be used in patients w/left ventricular dysfunction, unless contraindicated), DIGITALIS can improve symptoms and exercise tolerance by increasing cardiac contractility, other medications include oxygen and morphine. ASPIRIN, NSAIDs, and CALCIUM CHANNEL BLOCKERS SHOULD BE AVOIDED. Patient treatment and dental management considerations: prolonged rest, administration of oxygen, digitalis (patients are prone to nausea and vomiting), diuretics/vasodilators (patients are prone to orthostatic hypotention; avoid excessive epinephrine), dicumarol (patients may have bleeding problem). The beneficial effects of digitalis in congestive heart failure results in part from the fact that digitalis causes a decrease in end-diastolic volume, an increase in stroke volume and cardiac output, a decrease in central venous pressure, and a decrease in rate of the heart where tachycardia exists. Digitalis glycosides generally decrease edema, decrease heart size, decrease heart rate, and decrease residual diastolic volume. 688. Neuropraxia - I chose the option about damaged axon, but intact epineurium – FALSE -> Neuropraxia is the mildest form of neural injury. Local axon demylineation occurs at the site of injury. However, the axon, the endoneurium, the perineurium, and the epineurium are INTACT. 689. If a patient has had a car accident and an assumption of broken cervical spine, which xray is contraindicated? Waters, SMV, Pano, CT 690. Q. about Sterilization. 691. Pat. With xerostomia. All is true for the indications to manufacture a complete denture with METAL basis except? More acuracity, More wetability, Low cost, More durability 692. Pat. comes back after two weeks with discolored margins. Cause? amine 693. Qs about preparation before grafting. I chose the option to remove all granular tissue completely. 694. Carbamacepin used in? They give you very confusing options, that look similar!!! I chose nerve pain. 695. Which postion is patient related? VDO, CR, VPR, ICP 696. What will most likely cause the odontoblastic processe to retract in the dentinal tubules? Desiccation? (or Depth to which dentinal tubules are cut?) 697. All is correct about sclerotic dentin except? I was torn between extremely low permeability or better bond with restorative materials compared to normal dentin.. 698. Which study doesn’t show cause and effect. - cross sectional 699. Two questions on Incisal guidance. 700. What is true about remineralized enamel. - more resistant to future acid 701. Sialolithiasis. – submandibular gland 702. Patient complains of pain due to oral mucositis after radiotherapy. The pain is best treated with? a) Nystatin b) Benzyl hydrochloride (BENZYDAMINE HYDROCHLORIDE) c) Topical corticosteroid d) Morphine 703. Characteristic feature of AML? Auer rods 704. Pindborg tumor or CEOT - Liesegang rings / rx: snow appearance 705. Elective RCT contraindication. – recent MI 706. Gagging patient, what's the tx in a long term. Desensitization 707. Exhaling wheezing. - Asthma 708. Adverse effect of codiene. Miosis, nausea 709. Culture sensitivity test. - It was also about an infection that couldn’t be treated with particular AB 710. Not easy Qs. about pontic design. 711. Action of beta blocker on smooth muscle? vasoconstriction 712. What is true about if a general dentist decides to perform a treatment which is normally done by specialist.- No maleficiency in options! There was something like, “He has to perform as good as a specialist, if he does the tx” 713. Q. about orthostatic hypotension. - Patients taking diuretics/vasodilators are prone to orthostatic hypotension, also avoid excessive EPI. - A fainting spell that occurs due to a rapid fall in BP when moving from the supine to the upright position when getting out of the dental chair. The symptoms are similar to simple fainting, but the condition is related to positioning. Amyl Nitrite (Nitrites): Adverse Effects à orthostatic hypotension & headache. 714. Pain killer in pt who has just withdrawn alcohol? Hydrocodone (opioid) 715. Alcoholic pt pain med? Put naproxen. (if not in option hydrocodone) 716. Side effect of erythromycin? Gastrointestinal disturbances – GI tract upset is the most common side effect of the erythromycins (take with food). 717. Risk of extraction of upper Molar? Palatal root goes to sinus 718. Risk of extraction of #32? Damage to L nerve (IAN before Lingual) 719. What’s unethical? Charge more to difficult pts, Raise fees to particular group of insurance. 720. Radiopacity in lower molar area, no teeth. I put idiopathic schlerosis 721. Pt with bleeding and skin lesions? What condition – Leukemia 722. 2 yrs old intrudes incisor all the way in but doesn’t touch perm. Vital. What to do? Ortho extrusion then splint? Leave it 723. 14 is intruded in middle age pt, has distal caries, how to level plane of occlusion? Extract, Intrusion, Crown 724. Mode: The most frequent measurement in a set of data. 725. What type of study is if they look for prevalence? Cross Sectional 726. Socioeconomic influences the risk of developing perio and caries T 727. Due to this there’s risk of perio disease at ALL ages? F 728. What is that a drug has the strongest effect? Potency – Efficacy 729. Treatment of LAP - Put SRP and abx 730. Tx of perio with calculus on smoker. Put: debridement, oh, tobacco cessation advice. 731. When referring a mesialized 32 with irreversible pulpitis with curved roots for RCT, what’s the least important: curve roots, Canal calcification, Difficulty with anesthesia, Inclination of tooth (chose this) 732. U see nasty white lesion on floor of mouth, suspect of except: Scc, Verrucous carcinoma, Leukoplakia, Nicotinic stomatitis (chose this) 733. Same lesion what do u do first to dx? Cytology Excision Biopsy Nothing Wait a month. 734. Glass fiber post compared to custom made post: Aesthetic. Tooth conservation. Less chances of fracture. 735. Patient has tissue on palate due to denture, how to treat? Laser Excision Nothing Not wear denture for a while 736. Funtion of the internal part of an implant? they didn’t mention hex i chose antirotation 737. Short molar how to get extra retention? Groove on buccal 738. When doing protrusive on articulator, do you raise the pin? YES 739. What resto gives more retention for a short crown? Mod onlay, Veneer with grooves, 3/4 with proximal grooves (chose this, not sure) 740. Non-nitrate vasodilator for angina? Dipyridemole, nitroglycerin, lisinopril, isoflorophtate 741. What are filters for? Filtration reduces patient dose, contrast, & film density. Absorbs not useful xray (long length), reducing the patient dosage. 742. IAN anesthesia not well, what accessory nerve might be causing pain? Mylohyoideo nerve 743. U did resto, patient comes back after 6 months with pain to sweet and bitting? Reversible pulpitis 744. Lower molar, no pain. Has radiolucency on all apex of roots, why? Spontaneous necrosis, Vertical fracture, horizontal fracture 745. Target = tungsten? T 746. What happens if you increase more liquid to ZOE? more soluble? 747. Area most susceptible of caries: pits and fissure, above the contact, cervical of contact 748. ANGINA: NITROGLYCERIN, sometimes propranolol, calcium channel blockers, such as verapamil. Angina is primarily treated with nitroglycerin. Drugs that can be used in the prevention and treatment of angina pectoris: propranolol, nitroglycerin, isosorbide dinitrate, pentaerythritol tetranitrate - DD 749. Selective serotonin re-uptake inhibitor drug with the longest half-life (SSRI) – Fluoxetine (Prozac) 750. Advantage of glass fiber posts - Similar modulus of elasticity as dentin 751. What is the indication for an apicoectomy? a. Failed root canal that cannot be reinstrumented b. Root canal that is difficult to perform 752. Example of someone who comes into the office and has a belligerent attitude and says that every dentist she has seen has been incompetent and then 30 minutes into the appointment she starts praising you and saying that you are the best dentist she has had, this person’s personality is best described as: a. Narcissistic b. Borderline c. Schizoid 753. Halogen light and LED light which is true: a. LED light does not polymerize unless has champhoroquinone initiator b. LED light cannot be battery operated c. LED light bulb has a short life span d. LED light emits wavelength 350-370 754. Amantadine (Symmetrel)- anti-viral agent that enters the CNS to treat Parkinson's disease by potentiating dopaminergic responses. 755. Osteoporosis: Thin traveculae 756. Which of the following describes growth after menarche: a. Increases b. Stays constant c. Stops (Mosby: general body tissues including muscles and bone accelerate growth at the same time reproductive tissues proliferate.) 757. Implant is successful? a. 1mm bone loss in year one, 0.2 every year after b. 1 mm bone loss year one, 0.02mm every year after – Mosby and DD say 0.02 mm, no misprints! 758. When analyzing the arch distance which space do they analyze? 1- mesial of canine to canine 2- mesial of first premolar to first premolar 3- mesial of first molar to first molar ( I think the answer is 3) – Mosby: distal from second primary molar to distal of second primary molar on other side 759. Which one of the tooth is most prone to cracked tooth o syndrome? 1-Maxillary premolar 2- mandibular premolar 3-maxillar first molar 4-mandibular second molar - (mandibular first molars followed by 2nd mandibular molar and maxillary premolars) 760. Pt comes to you with complete denture that is loose, she wants a new one, and there is redness under the upper denture with papillary hyperplasia? What is your immediate management ? Excision of lesion, Tissue conditioner on the same denture. Make impression and fabrication a new. 761. The roubela causing mental retardation is caused by? 1- genetically 2- acquired 3- chromosal 762. Permanent max lat inc calcification starts when - 10-12 months (max centrals 3-4 months) 763. Primary principle of behavior modification a) stimulus cause behavior b) behavior has consequences 764. If pt was addicted to alcohol and had a treatment for his addiction, and for 18 months successfully is not drinking. He is considered to be completely recovered from addiction. T/F 765. Which tooth has most common crown-to-root fracture: max anterior incisors, mand ant incisors, max molars, mand molars 766. Most common with chronic periodontitis: Hispanic male, Hispanic female, Black male, Black female 767. Prevalence of cells in gingival sulcus: neutrophils, macrophages, plasma cells 768. Mild facets on occlusal surfaces in primary dentition indicate: normal function, indicate early , parafunction habits, occlusal interferences 769. NUG clinically resembles: primary herpetic gingivitis 770. Actinomycosis of Jaw commonly leads to: diffuse sclerosing osteomyelitis, or ruptures through facial neck soft tissues via multiple drain sinuses 771. Disadvantage of partial thickness flap is: unable to increase zone keratinised gingiva, or dissection from vascular supply source - A or missed option, i have disadvantage in my notes lack of surgical flexibility (limited visibilty and access) from old rq 772. Class II drugs: Percocet, Vicodin, Tylenol 3 773. Studies proved that caries can be arrested by placement of: Ca hydroxide, GIC, or sealing well the margins of restorations – On the basis of the studies cited in this review, one can state that there is substantial evidence that the removal of all infected dentin in deep carious lesions is not required for successful caries treatment— provided that the restoration can seal the lesion from the oral environment effectively. 774. RCT was done one year ago, periapical RL became larger since then, all can explain this, except: Apical scar, proximity to incisive canal, different angulation of x- ray... 775. As for ectodermal dysplasia is the lack of development of zygoma, for cleidocranial dysplasia it is: clavicle 776. Cleidocranial à supernumerary 777. develops because of: stone, or trauma to the duct – Mucocele = trauma 778. Min count of granulocytes for elective surgery: 1,000mm3, 15,000, 100,000, 10,000... 779. In children and adolescents most common cause of xerostomia? 1) sjron, 2)salivary gland obstruction 3) nocturnal mouth breathing 780. Varicosity under tongue: because of untreated HTN (there was not age in options) 781. Pt has vasovagal syncope, you do all, except: A. Administer oxygen B. Place in Tredenburg position C. Give epinephrine D. Apply spirits of ammonia E. Maintain airflow 782. Minimum of hydrocortisone for the Pt to be considered suppressed if taken daily: 20 mg before 2 weeks for 2 years, 10 mg before 2 weeks for 2 years, 100 mg same.. 783. In modified Widman flap it is: full-thickness flap, or partial flap Modified Widman Flap - The modified Widman procedure is a replaced mucoperiosteal (full thickness) flap procedure. 784. Inferior alveolar artery is a branch from: external carotid artery, or internal 785. Mandibular tori are to be removed before constructing the denture. T/F if interfere 786. You need to put in surgery consent about the risk of damaging lingual nerve. T/F 787. Side effects of albumin are all, except: tachycardia, stimulation of CNS, insomnia, increased salivation, diarrhea. (people say it is “atropine” written wrong, so answer here is salivation). 788. The Q about relation of consistency and frequency of maintenance perio visits and patient compliance and long-term warranty 789. A patient's compliance with schedule maintenance visits has no effect on the long-term retention of periodontally treated. Frequency of maintenance visits has no correlation with the development of periodontal pockets and gingivitis. 1. Both statements are false. (ANS) 2. Both statements are true. 3. The fist sta. is true, the second is false. 4. The first sta. us false, the second is true. 790. With the damage of oculomotor nerve where eye will move: (there were 2 directions in every option, like upward and medial, or downward and lateral...) downward and outward, ptosis – Oculomotor controls superior rectus (elevation and adduction), medial rectus (adduction), inferior rectus (depression and adduction) and inferior oblique (elevation and abduction) – FA 791. With the trauma of CN VI nerve in which direction difficult to stare: (one option), downward, upward, lateral, medial 792. MOA of sulfanilamide (sulfonamide): bacteriostatic, inhibit protein synthesis, COMPETE with PABA to inhibit PABAs actions, which prevents bacterial folic acid synthesis to inhibit cellular growth. 793. Pt on sulfonamides, what will be the most likely side effect that being a dentist you can say? ? – EM is a type of allergic hypersensitivity reaction in response to medications, infections, or illness. Medications associated with erythema multiforme include sulfonamides, penicillins, barbiturates , & phenytoin. 794. It is the preferred to quite smoking on the day when full extractions series scheduled. T/F 795. To be able to burnish a margin of a gold restoration which one should be exceed? a) Modulus of elasticity b) Yield strength 796. What impression material has a natural affinity to water? (no hydrochloride in options) Polyether (hydrophilic) 797. Initiate Chantix week before planning on quitting smoking. T/F - Varenicline (trade name Chantix and Champix), is a prescription medication used to treat nicotine addiction. Begin CHANTIX dosing one week before this date. 798. In anterior mandibular teeth there is some crowding. The gingival recession can develop because of all, except: frenum pull, asthma inhaler use, plaque, not enough ... 799. Know immunosupressive drugs: (will give you 4 of them, you rule out one) Also know their side effects more than candida. – I answered Candida but I did not know what was this drug about...Guys, memorize this mycophenolate drug is immunosuppressant! 800. Mycophenolate (CellCept) is used with other medications to help prevent transplant organ rejection (attack of the transplanted organ by the immune system of the person receiving the organ) in people who have received kidney, heart, and liver transplants. 801. Q just before this one asking all are immunosuppressive except one? Methylphenidate – (treatment of attention deficit hyperactivity disorder (ADHD) and narcolepsy), among the more serious adverse reactions are nervousness, insomnia , and anorexia. 802. What can decrease the effect of Warfarin: liver insufficiency, stimulation of hepatic microsomal metabolism - an increased response to warfarin would be expected in patients with liver impairment. 803. Pt with bizarre behavior and confusion, you administer: insulin, epinephrine, glucose 804. The reason to mounting study models is to observe: vertical dimension of occlusion, vertical dimension of rest, interdental occlusion, size and location of anterior teeth. 805. After polishing which appears first on teeth: material alba, biofilm, pellicle 806. Deepest part of occlusal rest is in: central fossa, marginal ridge – The deepest part of an occlusal rest preparation should be inside the lowered marginal ridge. The marginal ridge is lowered to provide bulk and to accommodate the origin of the occlusal rest with the least occlusal interference. – McCracken

807. Which agency initiated Hazzard communication program: CDC, OSHA, EPA 808. Hazzard communication program is about: protecting from and managing blood-born and infectious exposure, protecting from chemical exposures 809. Primary teeth shed prematurely in: cyclic neutropenia? - Yes, true. Perio disease may result in loosening of teeth and early tooth loss in young children. 810. There was x-ray with upper canine with still forming apex, looked basically normal in my opinion, but had in canal some obliterated oval structures...asked it is due to: AI, DI, DD, regional odontogenic dysplasia. Had permanent max canine still forming apex, yes with some premolars and 2nd molars still forming apex too. Those obliterated structures I noticed in both upper canines, but Q asked only about left one. It was regular width canal with pulpal stones. Tooth also was showing half out and the rest under gingiva still. I googled, found some research article where it saying if pulpal stones are seen in several teeth in young permanent teeth the condition is systemic and characteristic to DD – TYPE II (coronal dysplasia): color of primary teeth is opalescent (amber-colored) color of permanent teeth is normal, coronal pulps of permanent teeth are usually enlarged (”thistle tube”) and may contain pulp stones (DD) 811. If there is narrow attached gingiva next to maxillary second molar what you cannot do: apically displaced flap, gingivectomy, distal wedge – Distal wedge procedures, frequently performed after wisdom teeth are extracted, because the bone fill is usually poor, leaving a periodontal defect. Only if sufficient space exists distal to the last molar, a band of attached gingiva may be present. In such a case, a distal wedge operation can be performed. (DD). 812. 14 yo has a good oral hygiene, lives in community fluoridated area. For prevention of future caries what should be instructed: better oral hygiene, placing sealants in all 4 1st molars, fluoride gel placement by pt daily, fluoride rinse daily after brushing. 813. Also she has a little brown discoloration in her mand 1st molar occlusal pit and fissure, with an explorer catching on it, you should: place sealant over, preventive resin resto, amalgam, no treatment at this time. 814. Biological width: junctional epi and connective tissue attachment within sulcus, junctional and connective tissue attachment 815. Side effects of nitroglycerin: (know more, can be an except Q) – hypotension w/ reflex tachycardia, syncope, headache, flushed skin, dizziness, paradoxical bradycardia (google books) – MOSBY: Headache, syncope, tachycardia, tolerance, methemoglobinemia. 816. Side effect of nitroglycerine a. nausea, headache b. lightheadness, respiratory depression 817. Most common post-extraction complication is (did not mention any jaw): dry socket, infection, hemorrhage, fracture 818. Least reoccurrence after removal: ameloblastoma, adenomatoid odontogenic tumor 819. Only microscopy can determine the diagnosis: ameloblastic fibroma, fibro- , KOT, radicular cyst - - follicular & dentigerous cysts that contain keratinizing material, and differs from other odontogenic cysts due to their microscopic appearance & clinical behavior. 820. Pt with chipped veneer but would like to keep it, for fixing chipped part with composite you do: micro-etch, etch, silane, bonding 821. Most common location of intraoral melanoma: palate and gingiva, palate and lip... 822. In osteoporosis you expect to see: brown tumors, thin trabecular, decreased calcification of cortical bone 823. What is not an advantage of LED cure in comparison to halogen: lifetime bulb, energy efficient, weight, curing depth 824. First sign of HIV: asymptomatic, hairy tongue, opportunistic manifestations 825. What change in white blood cells you see in cellulitis: neutropenia, neutrophilia, lymphocytosis... 826. Type of external resorption in pulpal necrosis: inflammatory, replacement 827. Nitrous oxide is contraindicated in asthma pts. T/F NO can be used in anxious pts. T/F 828. Which is safe in all trimesters of pregnancy: ibuprofen, aspirin, some opioid, acetaminophen+codeine (Tylenol 3) 829. How to differentiate incipient white caries on smooth surface with remineralized caries: transillumination, enamel illumination, with using explorer on surface, with better lighting and air drying 830. Studies showed the the effect of leukotriens in: asthma, ulcers 831. Implant analog is what for: to take an impression with it, to pour impression with it 832. All influence duration of anesthesia, Except one: (I think ASDA q) 833. Which muscle to take considerations in shaping the lingual border of denture: mylohyoid, genioglossus, geniohyoid 834. If posterior palatal seal is too deep it causes: unseating of denture, gagging 835. Soccer player wakes up with sore and stiffness of the jaw: myofacial syndrome, osteoarthritis 836. Helix built in uprighting molar serves for: increase force, increase range, increase body translation - in dd ortho card 14 mention helices are incorporated to increase the action range and flexibility , so its b 837. Posture has an influence on: (different intraoral findings in options) - If I remember it correctly , the options were vdo, vertical dimension at rest... 838. Veneer with discolored margin after 3 weeks, reason: amines, insufficient resin 839. FDA is conducting a clinical trial about a new drug on animals and human. What is the phase 3 of this study? a. to see if the drug is cancerous on animal or not b. to find the effective dose of the drug c. to find the MOA of drug d. …..--> there was another option which I think it was the correct answer but I don’t remember it phase 1 - Safety and dosage phase 2 - Evaluate/find effectiveness phase 3 - to CONFIRM the effectiveness, to find common side effects phase 4 - to find the RARE side effects 840. Which one in wax try in for complete denture? a. Facebow record b. Esthetic 841. Early primary teeth lost a. papillon-lefevre syndrome - T (autosomal recessive, deficiency in cathepsin C, primary teeth lost by 4 years old, permanent teeth lost by 14 years old). 842. Most common in mandibule premolar region a. CCOT b. OKC c. OM d. fibrous dysplasia e. 843. Submandibular space drainage through which muscle a. platysma b. masseter c. median pterygoid 844. When draining purulent exudate from an abscess of the using an intraoral approach, the buccinator muscle is most likely to be incised. - DD 845. Which oral pigmented lesion resolve spontaneously? a. varix b. freckle c. melanotic macule 846. Which is correct about conjugation? a. adding a molecule to the drug b. making the drug more lipid soluble c. increase in the effect of drug 847. Which of these cognitive behavior decrease in a normal process of aging: a. learning b. attention c. reaction 848. Which one does NOT show the dispersing of date a. variance b. median c. standard error 849. DMF shows? a. caries b.oral hygiene 850. Dmf index = measures how permanent dentition is affect by caries 851. Fail-safe mechanism part of NO machine does not let the increase of NO flew by what percent a.20 b.50 c.70 d.90 852. Pappoos board isn’t used in which situation: Treatment of a 14 years old cooperative patient 853. Where do progenitor cells for new attachment come from? a) alveolar ridge b) PDL c) cementum d) connective tissue of gingiva 854. What does bimaxillary protrusion mean? A protrusive dentoalveolar position of maxillary and mandibular arches that produces a convex facial profile. 855. Pt doesn’t have upper 3rd molars. How many furcations do we have in upper jaw? a. 12 b. 14 c. 8 is the ans now if 6 is in there is 6?? The answer should be 16! first maxillary premolars are BIFURCATED! Maxillary molars are TRIFURCATED! That means we have (4 maxillary molars * 3 furcations) + (2 first maxillary premolars * 2 furcations) = 16 FURCATIONS!!! 856. Retentive clasp fracture? a. Work hardening b. Crystal formation in clasp c. Low modulus of elasticity d. High elongation 857. Mild wear facet in primary dentition? a. No treatment needed b. It is bcs of parafunction 858. Which one has least effect on DURATION of local anesthesia a. Systemic absorption of drug b. Bind to protein in tissue (Absorption = bioavailability, Duration = protein binding) 859. Missing more than two teeth but not all them – Oligodontia 860. What impression material has a natural affinity to water? a) Polyether b) Additional silicon c) Condensing silicon d) Poly sulfide 861. What do you see in pathology of osteoporosis - Thin trabecula 862. Not a symptom for dentin dysplasia (enamel loss) 863. Most common supernumerary tooth – Mesiodens 864. Most common variable tooth is - Max Lat 865. Most common tooth affected by dens in dente – Max lateral 866. Most common type of tooth loss in bulimic pt - erosion, lingual aspect 867. Another question about tooth loss was unclear it was about tooth loss due to tooth influx – 868. Patient who has medical history but is not debilitating but will require medical management and dental modifications – 1-ASA 3 2- ASA2 869. Failure in morphodifferentiation results in . a.size and shape abnormality ,b. peg lateral ,c. d. All is the ans 870. Bald tongue and dysphagia is in a risk of developing which cancer - Plummer vinson syndrome expressing as SCC (Oral Symptoms: angular stomatitis, smooth, red, painful tongue with atrophy of the papillae). 871. Problem in CL channel which disease - Cystic fibrosis 872. Crowe’s sign - Neurofibromatosis type 1 (von Recklinghausen disease of skin) à autosomal dominant disease, due to a mutation of the tumor suppressor gene NF1, six or more café-au-lait macules greater than 1.5 cm – these are usually smooth- surfaced (“Coast of California”), two or more neurofibromas OR one plexiform neurofibroma (pathognomonic for the condition), axillary freckling (called Crowe’s sign), iris hamartomas (called Lish nodules). 873. Uniform widen in pdl and bilateral resorb of angel what is the disease: No scleroderma in options but “SYSTEMIC SCLEROSIS” (its is also known as SCLERODERMA) 874. Pt has PDL space widening with radiolucency at the angle of the mandible (a) multiple sclerosis (b) osteosarcoma (c) fibrous dysplasia (d) other options – Now you know the missing option. 875. Oral sign with achondroplasia - Normal tooth, crowding and class 3 – DD à The teeth are of normal size but there is limited space within the maxillary and mandibular arches for them to erupt into, which causes overcrowding and subsequent malocclusion. 876. Dental problem with decreased Alk pho – Hypophosphatasia = Enlarged pulp and incomplete roots. DD à CHILDHOOD: THE MOST COMMON FEAUTRE IS PREMATURE LOSS OF PRIMARY TEETH WITHOUT INFLAMMATORY RESPONSE. The premature loss of teeth in children and adults is usually characteristic. Radiographically, the teeth display enlarged pulp chambers and pulp canals, deficient root development as well as alveolar bone loss. Patients w/ Paget disease have high levels of serum alkaline phosphatase. 877. X linked associated with thin hair and - Ectodermal dysplasia 878. What does the hyperbaric chamber help with I put angiogenesis 879. Time for surgical hand washing is 3 min 880. Which drug gives tardive dyskansia is phenothiazines - Tardive dyskinesia-a serious, irreversible neurological disorder that can appear at any age. It is a side effect of antipsychotic/neuroleptic drugs (i.e. phenothiazine). 881. What condition of the tongue involves the foliate papilla is - lingual tonsil hyperplasia 882. Supernumerary teeth with cranial bossing which syndrome - Cleidocranial dysplasia – CD: the most distinctive features include delayed tooth eruption and supernumerary teeth, hypoplastic or aplastic clavicles, cranial bossing, and hypertelorism. (Mosby) 883. Warm bone sign of which disease – Paget (Paget's bone is also hypervascular and may feel warm to palpation.) 884. Early shedding of primary teeth and delay of perm teeth – 885. Dry and rough hair and enlarge tongue and max over grow associated with – Hypothyroidism 886. Bells palsy which nerve – Facial (VII) 887. Delay healing associated with all except (Cushing /Addison/-DM/vit c increase) 888. IgM heterohybridomas diagnosis for - Infectious mononucleosis by EBV 889. Punch out appearance and M spike - Multiple myeloma 890. Medicines in MM - Cyclophosphamide (alkylating agent, immunosuppressant) is used in DD, bisphosphonates 891. Wide pdl and paresthesia and tooth loss with – Osteosarcoma 892. Sharp pain in throat increase with chewing (Unilateral pain in throat worsen by chewing) – Glossopharyngeal neuralgia 893. Meds not given to pt on anti cholinergic tx - opioids 894. Most common non odonto cyst is – Nasopalatine Duct Cyst (Incisive Canal Cyst)- a "heart-shaped" radiolucency in the midline of the hard palate. It is the most common non-odontogenic/developmental fissural cyst. 895. Pear shape cyst is – Globulomaxillary - -an inverted "pear- shaped" radiolucency in bone between the roots of the maxillary lateral & canine (often causes the roots of the involved teeth to diverge). - Tear shape its lateral periodontal cyst. 896. Calcified flax cerbi with which syndrome - Nevoid basal cell syndr / Gorlin syndr 897. Most common cyst – Radicular 898. Only way to differentiate bw granuloma and radicular cyst – Histology 899. Tennis racket and honeycomb appearance which cancer - Odontogenic myxoma 900. Periapical cemental dysp more in – middle age black women/mandible anterior 901. Complex odontoma more in – posterior mandible 902. Most common burn in mouth due to – aspirin 903. Geographic necrosis with kidney problem what is the diagnosis – Wegener’s granulomatosis (Oral lesions present as strawberry gingivitis.) 904. Most common gland affected by salivary gland tumor is –parotid 905. Question about necrotizing silometaplasia – NECROTIZING SIALOMETAPLASIA: deep-seated palatal ulcer with clinical and histologic features mimicking those of a malignant neoplasm. Recognized lesion of the minor salivary glands, characterized by necrosis of the glandular parenchyma w/ associated squamous metaplasia and hyperplasia of the ductal epithelium. The initiating event of necrotizing sialometaplasia is believed to be related to ischemia, secondary to alteration of local blood supply. Both, clinically and histologically, the lesion may simulate a malignancy and, in the past, the condition has been misdiagnosed as a squamous cell carcinoma or . FOLLOWING BIOPSY AND THE ESTABLISHMENT OF THE DIAGNOSIS, FURTHER TREATMENT GENERALLY IS NOT RECOMMENDED SINCE HEALING USUALLY OCCURS WITHING 6-10 WEEKS. 906. Porcelain has tooth matching color by – glaze firing 907. Dentist choice by metamerism - Different colors under different light sources 908. Lab adds stains in the inside of the porcelain – to decrease value 909. Lab glazes and polishes the porcelain in the end of the design - More compatible to the gingiva; In dd card 135, said the glaze firing is the last firing and it produce smooth and translucent surface. During glazing: Surface layers of porcelain melt slightly, coalescing the particles and filling in surface defects (Mosby page 361) 910. Meds not given to one on BDZ – antifungal agents 911. Meds contraindicated in pt on barbiturates – phenothiazines, alcohol, antihypertensive agents, and antihistamines (Kaplan) 912. Chlorothiazide – electrolyte test because it causes hypokalemia 913. Except sweat changes, pt will also show what changes in cystic fibrosis: lung, saliva, urine and some other options - CF results in several symptoms (the most important symptom affects the digestive tract and lungs). 914. Medicines contraindicated in a cystic fibrosis pt – beta 2 antagonists, NO2 (General anesthesia is avoided too in DD, they will have COPD, like chronic bronchitis , sinusitis , so beta 2 agonist will help) 915. COPD pt, what is contraindicated - 100% o2 therapy, nitrous oxide, If patient with copd is taking theophylline should not prescribed erythromycin, it will lead to toxicity, card 75 surgery, General anesthesia is contraindicated and avoid certain antibiotic barbiturate, narcotic, antihistamin and anticolinergic. (USC pt management manual book). 916. Max amount of N2O that can you flow into the tube at a time - 70% adult / 50% kids 917. Tetraycline works by: no 30s or protein synthesis in option, but there was options of interference with collagenase, 50s unit, plasma proteins and dna gyrase 918. Pt on your chair, presents with insulin shock, what will you do next? Orange juice, glucose, epinephrine, O2, etc - The treatment of choice for hypoglycemia in an unconscious diabetic patient: EMS should be contacted. Then 1mg of glucagon can be injected IM, or 50 ml of 50% glucose (dextrose) solution can be given by rapid IV infusion. If conscious, oral glucose (orange juice). - DD 919. Pt on hep B meds, what will you do? Call physician, order regular CBC etc. blood reports, were some fancy names of tests later. 920. Mandi tori removal, most frequent complication? Options were loss of cortical border bone, injury to mental n, lingual n, or IAN injury 921. During maxillary tori removal, you accidentally perforate a part of the palate with the tori, what structure will you see? Nasal cavity, inferior concha, and one more irrelevant option 922. During 3M removal, which mand part is most likely to get fractured? Lingual plate 923. Closed mandi fracture, which is most likely to interfere with the closed reduction? Pull of muscle, 3 different combinations of muscles in 3 options, one option was improper access 924. Intermaxillary fixation indicated in all the following except? Know all the situations when it is needed: Typical indications for its use are minimally displaced fractures, deep bite cases, stabilization of fracture during open reduction and internal fixation, orthognathic surgeries and in tumor resection surgeries

925. Intermaxillary fixation is released earlieast in which of the following? Options with different mandibular feacture sites – The IMF wires are usually removed in 3 weeks and jaw exercises encouraged. Immobilization beyond 3 weeks in condylar fractures can result in ankylosis of temporomandibular joints. The intermaxillary wires may be reapplied for another week if occlusion is not good. Also, a simple, nondisplaced, greenstick (incomplete fracture) in a healthy child would certainly require less intermaxillar fixation time than multiple, grossly comminuted, compound mandibular fractures in an older unhealthy patient. 926. Best radiograph to view zygoma? No submentovertex option, waters, PA, CT, MRI 927. Pt with a nodule on the middle of the neck, what is most likely dx? Thyroglossal duct cyst probably, don’t rmbr other options 928. Bluish lesion on lateral surface of tongue for 5yr painless what is it varicosities or hemangioma – 929. Warty lesion—papilloma 930. Maxillary incisor 4 teeth rpd, what should we achieve? --- Mac anterior teeth contacting on protrusion only, at CR, balanced occlusion, canine guided occlusion 931. Benzoyl peroxide decomposition by tertiary amine in chemically activated resin – T 932. Abx in cellulitis with draining fistula yes or no 933. Antibiotic for sinusitis – Augmentin 934. Complication of temporal artritis – Blindness 935. Where do we use 10% chx varnish? P&f caries prevention, white smooth surface caries prevention, secondary caries prevention 936. Parkinson's disease, except --- is progressive, always require medication, associated with intentional tremor, associated with dementia 937. Amount of epithelial regeneration everyday is 0.5-1 mm – T 938. Chronic periodontitis, class 2 diabetes mellitus-- black males 939. Gtr best for: horizontal augmentation, class 2 furcation, one wall defect, class 3 furcation 940. Gtr-- involves coronal movement of pdl – T 941. Reattachment concept – Reunion of tissue to the rooth 942. Radiographic appearance of pericornitis – flame shape 943. Ameloblastoma, Benign, Localy aggressive, Reverse polarization, Rl post mn, Extreme facial deformity, teeth vital, painless, honey comb or soap bubble appearance 944. Basal cell carcinoma Most common skin cancer - Upper lip or lateral nose, Best prognosis, Sun exp area 945. Value negative calculation - Positive predictive value is the probability that subjects with a positive screening test truly have the disease. Negative predictive value is the probability that subjects with a negative screening test truly don't have the disease. Predict value positive: TP/(TP+FN)*100 Predict value negative: TN/(TN+FP)*100 946. Which of the following drugs is associated with the reaction of hepatitis? A. Valproic acid B. Quinidine C. Isoniazid D. Ethosuximide 947. Which of the following drugs is associated with the reaction of Stevens-Johnson syndrome? A. Valproic acid B. Quinidine C. Isoniazid D. Ethosuximide 948. Which of the following drugs is associated with the reaction of Tendon dyfunction? A. Digitalis B. Niacin C. Tetracycline D. Fluoroquinolones 949. Which of the following is considered a class IA Sodium Channel blocker? A. Propafenone B. Disopyramide C. Aminodarone D. Quinidine (Supraventricular tachyarrhythmias) 950. When part of body is thick which of the following applies? a. X rays penetrate more and the object appears more radiopaque b. X rays penetrate more and the object appears more radioluscent c. Change of developing cancer in that body part is less d. Chance of cancerous change is more – (if “penetrate” hear means “absorption”, because if the substance is thicker, it will absorb more photons and the resultant imaging will be white. 951. Rate of implant success after 10 years? a. 90 b. 95 c. 80 d. 85 952. Which results in necrotic pulp a. Inflammatory resorption b. Replacement resorption c. External resorption 953. Over the counter bleaching is with 25 % carbamide peroxide and the most common side effect is tooth sensitivity a. Both statements are true b. First statement is true, second is false c. First statement is false, second is true d. Both statements are false 954. While performing a tooth preparation, removal of one of the cusps and replacement with restorative material corresponds to the idea of: a. Resistance form b. Retention form c. Outline form d. Convenience form 955. A patient who had a deep MOD composite placed in one of the lower molars complains about pain and sensitivity, dentist replaces the occlusal of the restoration with a new composite and pain is gone what is an explanation of why the pain occurred? a. Voids in the previous restoration b. Leakage c. Fracture of the tooth – The reason for post operative sensitivity is polymerization shrinkage causing gaps, which could result in rapid movement of dentinal fluid and this sensitivity. - BB 956. Recurrent caries incidence for a class II composite is highest where? a. Gingival floor 957. DO composite does NOT contain which of the following? A. Axiopulpal line angle b. Axio gingival line angle c. Mesiofacial line angle d. Axio distal line angle 958. Which one of the following has the highest difference of coefficient of thermal expansion with the actual tooth? 1- ceramics 2- porcelain 3-polymers 4-polemr associated with resin. – Wax (250-400) 959. When there is minimum attached gingival, what happens? 1- most likely you get gingival recession 960. When you have a horizontal rot fracture how do u take and x-ray? 1-one xray from angulated vertical angle 2-one x-ray from horizontal angulated angle 3-3-multiple x-rays from different horizontal angles 4-multiple x-rays from different vertical angle (One at 0 degrees, then one at + and – 15 degrees – Mosby) 961. When do u remove alveolar proper? 1-ostectomy 2- osteotomy 3-alevoplasty etc 962. A pedunculated white lesion on the palate that is rough, what is it? 1- fibrome 2- papiloma 3- 963. Porcelain has tooth matching color by? a. Dentist choice by metamerism b. Lab adds stains in the inside of the porcelain c. Lab glazes and polishes the porcelain in the end of the design 964. All of these could be considered as differential diagnosis of aneurysmal bone cyst except? osteomas, fibrous dysplasia, central giant cell granuloma, hemangioma 965. For routine tooth extraction ,all is true except? 2 major forces are luxation and rotation teeth are extracted by luxation forces teeth are extracted by rotation forces (Rotation only for single rooted teeth. Not all) class 2 lever is used in tooth extraction 966. Compared to a full thickness flap, a partial thickness(split-thickness) flap will A. increase the loss of marginal bone. B. reduce infraosseous defects. C. provide improved surgical access. D. increase the amount of attached gingiva. E. reduce healing time 967. Which of the following is not an action of epinephrine when given in high doses? increases liver glycogenolysis causes bronchoconstriction produces rise in bp evokes extrasystoles in the heart produces restlessness and anxiety 968. What will you see in a diabetic patient? general gingival recession, gingival abcess, necrotizing gingiva, periodontal abcess 969. At high concentration of fluoride in drinking water (4ppm) the caries incidence: a) remains the same b) decrease c) increases 970. Moderately developmentally disabled 5-year-old child is crying excessively and resisting physically during an emergency dental visit. Which of the following methods of patient management should the dentist use in this situation? Voice control, Home, Physical restrain – Master app 971. What will prohibit mesial drift of tooth toward edentulous area? Proper axio- occlusal contact (opposing and adjacent tooth) – T 972. Proximal resistance form of amalgam restoration comes from what? a. convergence of buccal / lingual wall b. retention grooves in axiobuccal / axiolingual walls c. Dovetail – DD: dovetail provides resistance to proximal displacement. 973. Oral granulomas, apthous ulcer, rectal bleeding is seen in. Wegeners granulomatosis, ulcerative colitis, crohns disease 974. Which treatment has the least successful long term prognosis on a deep carious lesion on #3? 1. Direct pulp capping, 2. Indirect pulp capping, 3. Pulpotomy, 4. Pulpectomy and RCT - Direct pulp capping is for noncarious exposure only. (Mosby) 975. What would be the most reasonable cause for a tooth’s symptoms to change from reversible to irreversible pulpitis? a) Accumulation of traumatic injuries b) Bacterial involvement inside pulp chamber c) Increased intra-pulpal pressure 976. Excess interocclusal space causes: a) decrease VDO b) increase VDO c) same VDO 977. Biotransformation of drug causes – lipid soluble, protein binding, therapeutic active - makes it water soluble (less lipid soluble, more ionized) 978. Fear causes – inc pain tolerance, intensify pain - Fear often increases the person's perception of pain, and pain then increases feelings of fear and anxiety. 979. Which part of curette tip is adapted to the toot (a) distal 3rd (b) middle 3rd (c) proximal 3rd end – Lower third (1/3) 980. Diagnose Chronic apical abscess vs Chronic (a) EPT (b) Radiograph (c) Thermal test – I think answer is “c” but EVERYBODY say “a” 981. Which drug is effective against Herpetic simplex, Herpes Zoster and Varicella Zoster (a) Amantadine (b) Valiclovir (c) other IDK – According to Mosby PG 335 – Table 8-39: VALACYCLOVIR is effective against HSV and VZV- FOSCARNET is the only drug effective against HSV, VZV, and CMV. 982. Radiograph showing radiopacity in lower right mandibular area spreading from 2nd molar to 1st premolar (a) Perifying ossifying fibroma (b) CGCG (c) Fibrous dysplasia (d) IDK Fibrous dyplasia is radiopaque, not well circumscribed and ground glass appearance. 983. Pt avoiding dentist becoz of ugly ulcer on palate and also shows concern while sitting on chair, what do you say? (a) Don't worry I will look at it in detail (b) It seems you are concerned with that condition in your mouth (I picked this one as we don't give false assurance to any pt) 984. A dentist conductes a study about satisfaction of patients treated in 1 month period of time which study is this? ( basically a cause effect relationship result) (a) Case control (b) Cohort (c) Cross sectional (d) Clinical trial (my ans I just picked because it is interventional study I might be wrong) 985. Informed consent can have all of the following EXCEPT: A) Informed consent must be presented in advance of the treatment. B) Informed consent must contain treatment options. C) Informed consent must be in written form. D) Informed consent must contain risks and benefits of the treatment….. 986. One ques was on paraphrasing: they gave 4 totally confusing statement...I was supposed to pick which was NOT paraphrasing pt words..... 987. Patient complains, “Why do I have to stay here for so long for you to do this, why can’t you finish it already?” A) Because that’s how treatment works you idiot. B) That’s how long it takes to provide quality care. C) It seems like you’re upset, may be we can reschedule you for another day for longer appointment. D) It seems you are upset, what are your concerns about the procedure we’re doing today? 988. Question about what do you need for caries: Bacteria, supporting carbs and a susceptible tooth 989. Primary tooth requires additional reduction on which surface (a) Mesial and distal (b) lingual (c) buccal (d) other option DD à In the case of first primary molars, the buccal bulges often are very prominent. It is sometimes necessary to remove them to get the preformed crown to fit over the buccal prominence. 990. Incisal guide table is for (a) for anterior teeth arrangement (b) condylar guidance 991. PID ques about changing from 8 to 16 inch all other parameter remain only change in exposure time from 0.5 to which one (0.5*4=2) 992. In X-ray tube , Electrons are produced by ? A- molybdenum cup B- leaded glass C- tungsten filament D- copper filament 993. X-rays are produced when (1)protons strike the anode. (2)electrons strike the anode. (3)the anode is heated above 3,000 degrees C. 994. x ray effect is called: thompson effect or photoelectric effect 995. Epinephrin given along with erectile dysfunction medication what effect is produced: epinephrine is a vasoconstrictor!! so epinephrine will reduce the effect of the drug 996. Removal of subgingival calculus is termed as (a) scaling- (b) root planing – removal of infected cementum (c) curettage – removal of infected pocket lining 997. Ques asking which procedure is most conservative when width of caries is more than 1/3 of intercuspal space a) amalgum b) inlay c) onlay d) cronw 998. Epi reversal is due to – alpha 1 blocker 999. Force put on crown, where is center of translation or rotation? Halfway down root (axis of rotation located in the apical 1/3 of the root – tipping) 1000. Best to debride infected oral wound? 3% hydrogen peroxide 1001. Pt with Alzheimer dz, what do you do? Continue to monitor 1002. Lidocaine – mepivacaine (Cross allergy, both are amide) 1003. Cocaine produces vomiting by – activating CTZ in brain 1004. Actinic occurs with - SCC 1005. Amnesia related ques of which ans was Alprazolam – T - anterograde Amnesia 1006. Sertraline (Zoloft) adverse effect or something – frankly speaking I forgot what it was in my test I randomly picket xerostomia hope it is right - Yes zoloft or sertraline... cause xerostomia 1007. Zoloft what does it act on? Serotonin – it’s an SSRI (selective serotonin reuptake inhibitor) 1008. Complement activated by – a) T cells b) B cells c) lymphokines d) immunecomplex 1009. What do we write the consult for: A) To gain certain information B) To gain clearance C) To have a better relationship with patient’s physician, of course. 1010. Wheel chair transfer ques with option – sliding method still is best technique to transfer pt 1011. Disabled kid, best measure: Consistency 1012. Articular disk has 25 mm opening with click then on closure there is again a click, when is there is another click what is it due to – disk rest on condyle on opening and moves forward on closure (indirectly disk displacement with reduction) 1013. Best Amalgam: High copper spherical amalgam 1014. Mandibular 3rd molar root lost: submandibular space 1015. Which of the following is clinical sign of Leukemia: Bleeding from gums, pale conjunctiva, fever 1016. Acromegaly causes: Excessive growth of mandible 1017. Radiograph of zygomatic arch – CT scan, NO submentovertex in options 1018. Patient does not have tooth #11 and has all the premolars, which one has the Worst Prognosis: A) fixed bridge from #10-12. B) RPD with pontic for #11 – C) Implants with canine guidance 1019. Pt with medullary carcinoma of thyroid a-hyperparatyrodisum b-MEN (Pheochromocytoma and MEN cause thyroid cancer) c-interstinal polyps 1020. Kid-8 year old- 3mm crowding. TX a-primary canine extract b-primary canine disking c-molar extract d- molar disking 1021. blood flow test in pulp? ts Doppler ( for blood flow) or like this thing , was before rq with options 1022. IRM added in 1990, what was that? a-ZnPo4 b-titanium c-silver d-PMMA 1023. Discolouration with ant tooth, endo treated portion of carious teeth, need to be restore? TX a-porcelin veneer b-FCC – Full Ceramic Crown c-metal crown d-composite 1024. After injection-pt feels tachycardia, weak, wheezing, lethargy. Reason? Anaphylaxis, hyperglycemia, anxiety attach 1025. Edge to Edge ant bite seen in photograph what is not recommended? a. PFM Crown b. All ceramic crown 1026. Patient has lithium overdoes its effects is greater on ? KIDNEY and TYROID. Nausea, diarrhea, convulsion, coma, cardiac arrhythmias, polydipsia, polyuria, inhibits the effect of antiodiuretic hormone on the kidney. TYROID ENLARGEMENT: increases stimulating hormone (TSH) secretion; may cause hypothyroidism. 1027. Most type of bacteria in ANUG ? A. Provetella b. Spirochete 1028. What type of fracture associated with exposed impacted tooth? A. Comminuted. B. Compound c. Simple - Compound. Bone would be exposed through the mucosa near teeth. 1029. Least likely virus to be found in the oral cavity infections – HIV 1030. Which lesion resolves by itself? Hematoma and hamartoma both in option, others were obvious tx needed. 1031. Bone marrow depression by what drug? – Chloromphenicol 1032. Tx of osteromyelitis? Was surgical options, no meds - Sequestrectomy, saucerization, curettage 1033. Tooth prep ques on anterior teeth, reduction on middle and incisal third for PFM: gingival 0.3, middle 0.5, incisal 0.7 1034. Finishing line in PFM (CHAMFER 1.0mm - LABIAL SHOULDER 1.5mm) and veneer (CHAMFER) – Butt joint (SHOULDER) for porcelain jacket crowns 1035. Pt came in for a 3 month recall, initial therapy doesn’t show any changes, pocket depths not exceeding 3-4 mm, what tx step will u do? srp and wait for 3 more months, surgery only if pockets greater than 4 mm 1036. Pt comes in for a 6 month maintenance appt, little improvement in pdl status and plaque control is efficient, though u can see infra gingival calculus, what was the reason? You didn’t do SRP properly, pt has no goof access to the deep pockets, pt only concerns about looks, etc 1037. Angry child, shows this behavior on the second but was cooperative on the first appt, what is the best method to control this behavior? N2O tx, GA, papoose board, voice control 1038. Validity – is the extent to which it actually tests what it claims to test. The validity of a test is determined by its ability to show which individuals have the disease in question and which do not. 1039. Sensitivity – percent of persons with the disease 1040. Specificity – percent of persons without the disease 1041. Reliability – equal to the repeatability and reproductibility of a test (level of agreement between repeated measurements of the same variable). 1042. Macroglossia is not seen in which of the following conditions, chose hyperparathyroidism, 1043. Pt with mid face ill developed, no ear pinna, which syndrome? Eagle’s, tracher Collin’s, apert, crouzon - Malformed ear= Trache Collin's 1044. Cause of angular chelitis: immune, speech therapy, poor home care Predisposing Factors: intra-oral Candida albicans infection, loss of inter-maxillary distance (decreased vertical dimension), trauma to the labial commissure induced by prolonged dental treatment. Also linked to Candida albicans. Treatment: NYSTATIN will eliminate the fungal infection. - BB 1045. Ethical principles and legal rules? Both are same, totally different, ethics exceed legal rules, rules exceed ethics. 1046. AED - automated external defibrillator (AED) is not used in? children, old patients, etc - ts not given to trauma pts, kids under 1 year old and ppl with high pulse A defibrillator must not be used on an individual who is conscious or has a pulse even if it is erratic but not life-threatening. 1. Responsive 2. Unresponsive WITH pulse 3. People under 55 LBS 4. People who are soaking wet (dry off chest) 1047. Aspirin patch is histologically? Necrosis, hyperkeratosis, etc 1048. Folic acid inhibited in? methotrexate, fluorouracil – people say both, BUT! - Methotrexate is a Folic Acid Analog and 5-Fluorouracil (5FU) is a Pyrimidine Analog. 1049. Bisphosphanates are not given in? metastasis of breast ca to bone, metastasis of prostate to bone, osteomyelitis, multiple myeloma 1050. Which is the most common oral site for metastatic cancer? Posterior mandible 1051. Which is the most common site for primary ? Tongue 1052. Worst prognosis? Floor of tongue 1053. Best prognosis? Lower lip 1054. Supra basilar split and 1055. Bone Grafting, which one shows worst prognosis? Max ant, mand ant, max post, mand post 1056. Trephination – Apical trephination is accomplished by aggressively placing a No. 15 to 25 k-file beyond the confines of the apex. Surgical trephination is a perforation of the alveolar cortical bone to release accumulated tissue exudates. A small (5-mm) horizontal incision is made with a No. 15 scapel blade at the level slightly apical to the root apex. A No. 6 or 8 round bur is used on a straight handpiece to penetrate the cortical plate above the root apex. If there is diffuse swelling (cellulitis), antibiotics are usually indicated. – DD A. Incision and drainage and trephination. 1. Objectives are to evacuate exudates and purulence and toxic irritants. Removal speeds healing and reduces discomfort from irritants and pressure. The best treatment for swelling from acute apical abscess is to establish drainage and to clean and shape the canal. Indications for trephination of hard tissues: a. If a pathway is needed from hard tissue to obtain necessary drainage. b. When pain is caused by accumulation of exudate within the alveolar bone. c. To obtain samples for bacteriologic analysis. Procedure. a. Incision and drainage is a surgical opening created in soft tissue for the purpose of releasing exudates or decompressing an area of swelling. Trephination refers to surgical perforation of the alveolar cortical bone to release accumulated tissue exudates. Profound anesthesia is difficult to achieve in the presence of infection because of the acidic pH of the abscess and hyperalgesia. The incision should be made firmly through periosteum to bone. Vertical incisions are parallel with major blood vessels and nerves and leave very little scarring. These procedures may include the placement and subsequent timely removal of a drain. Antibiotics may be indicated in patients with diffuse swelling (cellulitis), patients with systemic symptoms, or patients who are immunocompromised. 1057. Purpose of Hex in implants - antirotation 1058. Most common type of caries seen in kids 1059. Non working side interference 1060. Beclomethasone uses – Beclomethasone, Budesonide, & Flunisolide: special glucocorticoids (INHALERS) developed to treat chronic asthma and bronchial disease by readily penetrating the airway mucosa, but have very short half-lives after they enter the blood so systemic effects and toxicity are greatly reduced. - is inhalational steriod used as inhaler in asthma prevention. 1061. What is advantage of Beclomethasone – corticoid (topical and inhaler) 1062. Combination of tricyclic antidepressants (there was diferent combination but the correct was) A) imipramine + amitriptyline 1063. Doxycyline read its uses – Doxycycline (Vibramycin)- treats syphilis, rickettsia infections, Chlamydia, & mycoplasma infections, and is an alternative to mefloquine for malaria prophylaxis. - Prevents further breakdown of periodontal tissues by blocking collagenase. 1064. Picture - Hyoid bone (both sides) 1065. Pano- inferior border of mandible 1066. Cocaine - vasoconstrictor 1067. Opioid side effects – Common Side Effects: sedation and drowsiness (by depressing the conscious centers of the brain), dizziness, & nausea. The MOST common side effect of the narcotic (opiate) analgesics is NAUSEA. Narcotic analgesics DO NOT cause peptic ulcers or insomnia. 1068. Fracture at root apex: splinting for how many days? 7-10 days, 2-3 weeks, 4-6 weeks – Horizontal fracture - rigid splinting for 3 - 4 months 1069. Avulsed tooth - flexible splint for 7-10 days (1-2 weeks) 1070. How long after extraction can you insert the complete denture???? 4 weeks, 1 week, 6 weeks, 8 weeks (8-16 WEEKS) 1071. Pt takes too much opioid, what do you see? A. insomnia b. irritability c. headache d. pt feels cold – because of hypothermia / hypothension 1072. Mepivacaine indication – mepivacaine has less of a vasodilator effect compared with the others and is the drug usually chosen when a vasoconstrictor is not used with the local anesthetic. 1073. Question on upcoding – reporting a more complex and/or higher cost procedure than was actually performed. 1074. Down coding: a practice of third party payers in which the benefit code has been changed to a less complex and /or lower cost procedure than was reported where delineated in contract agreements. 1075. Bundling: systematic combining of distinct dental procedures by a third party payer that result in reduced benefit for the patient/beneficiares. 1076. Unbundling: separating of dental procedure into component parts with each part having a charge so that the cumulative charge of the component is greater than the total charge to patients who are not beneficiaries of a dental plan for the same procedure. 1077. Bacteria seen in chronic periodontitis – P. gingivalis (P. gingivalis, T. forsythia, P. intemedia, C. rectus) 1078. Chronic periodontitis most common in – black males 1079. Percussion used for- symptomatic apical periodontitis 1080. Ept indications – usually elicits a response at a HIGHER current than normal if the tooth being tested has CHRONIC PULPITIS. Acute pulpitis - indicated by a lower than normal current, as acute inflammation mediators lower the pain threshold. Chronic pulpitis - indicated by a response at a HIGHER current than normal. Hyperemia - indicated by a LOWER than normal current, but a higher current than with an acute pulpitis. /Abscess - indicated by no response at any current level. - BB 1081. Benzoyl peroxide initiator- self cure 1082. Caoh indications - Calcium hydroxide may be used to induce apical hard tissue formation. Use calcium hydroxide for reparative dentin. Typical liner used with direct restorations. - Mosby 1083. Q on galvanic shock patient had electric pain after restoration 1084. Q on reversible pulpitis irreversible pulpitis 1085. Recession - apical positioning flap? I thinks it is contraindicated - Free Gingival Graft Indications: Prevent further recession and successfully widen (increase the width) of attached gingiva, used therapeutically to widen attached gingiva after recession occurs and prophylactically (to prevent), corrects localized narrow recessions or clefts, but NOT DEEP WIDE RECESSIONS. Pedicle Flap (Laterally Positioned Flap) - areas where narrow gingival recession. Used to correct or prevent recession by providing root coverage, creating a wider band of gingiva, and in the absence of recession to widen the zone of gingiva. Coronally Positioned Flap - a full -thickness mucoperiosteal flap almost exclusively used to restore gingival height and the zone of attached gingiva over isolated areas of gingival recession. 1086. Most prevelant - type two diabetes? Type 2 diabetes is the most common form of diabetes. 1087. Interaction between nitroglycerin and epinephrine is what type of antagonism? Allosteric, Physiologic, Biochemical, Competitive 1088. Color stability in light cure - Tegdma 1089. Composite class 2 restoration maintained by - extent till caries, retention n resistance form, rest i forgot options 1090. Rubber dam leakage - holes placed too close 1091. Rest thickness at margin - 0.5/1.5 or 1mm? – Occlusal rest 1.5 mm (Mosby) 1092. Treatment of nug what antibiotics n mouthwashes. - The treatment of NUG or NUP includes debridement, hydrogen peroxide (or chlorhexidine) rinses, and antibiotic therapy (Pen. V) if there is systemic involvement (manifested by fever, malaise, and lymphadenopathy). Patients with HIV-asssociated NUG require gentle debridement and antimicrobial rinses. (DD) 1093. What distunguish myocardial infarction from angina – thrombosis 1094. Indirect sympathomimetic drug? Diphenyl - Amphetamine is the ans 1095. Occlusal adjustments after composite restoration or amalgam: green stone, diamond bur, steel bur or carbide bur? - Remaining excess composite = finishing diamond burs, discs, strips, and the margins finely polished. Aluminum oxide disks provide the most desirable finished surface for a composite resin. - BB / Green stone is used to remove a relatively large bulk of amalgam. 1096. Sodium hypochlorite doesn't - chelates 1097. Sodium hypo - dissolves necrotic tissues 1098. If a patient is taking chantix what else need to be included in his smoking quitting regimen 1)use nicotin patches 2)zyban 3)behavioural counsellingis the ans 1099. Community fluoride: 0.2% / week in underprivileged areas . true is the ans 1100. Case q's about side effect of drug that cause altered taste sensation? Cyclobenzaprine ans , calcium carbonate 1101. Loosening and premature loss of deciduous teeth seen in early stage of - hyperphosphatasia - hypophosphatasia IS THE ANS - psuodophosphatasia vit d resistant rickets - vit d deficient rickets 1102. Patient complains of pain due to oral mucositis after radiotherapy, the pain is best treated with: a. Nystatin b. Benzyl hydrochloride IS THE ANS c. Topical conticosteroid d. Morphine 1103. Large filler particles in composite increase the strength of hardness? False - Small size filler particles in composite resins results in better finishing and greater resistance to occlusal wear. - BB 1104. Large filler particles in composite increase polishibility & finishing? False Smaller filler particles are used to produce a resin with a relatively smooth finished surface - BB 1105. Pt. wd radiation therapy effects- carcinogenisis? Osteoradionecrosis? Oral mucous membrane: (1) Near the end of the second week of therapy, the mucous membrane begins to show areas of redness (mucositis), (desquamated epithelial layer, secondary yeast infection by C. albicans is a common complication and may require treatment. 1106. Pt. has white spot on cervical area of tooth, what is the treatment- fluoride varnish or no treatment 1107. Reverse smile - pt chin upward 1108. In class V amalgam preparation for an incipient lesion, the ideal internal form of the preparation has which of the following features? a. Axial wall is flat b. Mesial and distal walls converge c. Occlusal and gingival walls converge d. Axial wall is uniformly deep into dentin 1109. Lithium – bipolar 1110. In preparing a class I cavity for dental amalgam, the dentist will diverge the mesial and distal walls toward the occlusal surface. This divergence serve to a. Prevent undermining of marginal ridges b. Provide convenience form c. Resist the forces of mastication d. Extend the preparation into areas more readily cleansed. 1111. Which bur is used to converge axial wall of the crown, ????/No. 173 1112. Elongation of which papillae - hairy tongue (HYPERTROPHY of the FILIFORM PAPILLAE) 1113. Facebow – The facebow transfer is NOT a maxillo-mandibular record. Rather, it is a record used to orient the maxillary cast to the hinge axis on the articulator. The facebow transfers the maxilla/hinge axis relationship to the articulator during mounting of the maxillary cast. 1114. TCA antagonist – Physostigmine - Physostigmine's primary therapeutic role aims to ameliorate delirium as a result of the anticholinergic (more accurately, antimuscarinic) toxidrome resultant from the blockade of muscarinic receptors by agents such as atropine, antihistamines, tricyclic antidepressant (TCA), amongst other xenobiotics. 1115. Class V glass ionomer prep should : not bevel at all – Not for Gic, bevel only for composite. 1116. What kind of bur cuts more efficiently? Diamond 1117. 12 year girl had AML and bone marrow replacement most likely to find intraorally? Candidiasis – children w/ leukemia are very susceptible to candida fungal infections, thus, nystatin rinses are effective tx. 1118. Which drugs cause cleft lip and palate = anticonvulsants , valium, vitamin deficiency or excess 1119. Place a FPD and it has occlusal deflection, what it the immediate result? A. fracture B. pain on biting C. sensitivity to cold. – The most common complaint after cementation of a fixed bridge is sensitivity to hot/cold and is an indication of a deflective occlusal contact. Inmediate correction of the occlusion must be made. 1120. Which of the following has decreased ALP and early loss of teeth – hypophosphatasia. 1121. Retention maxillary complete denture = peripheral seal 1122. Retention mandibular complete denture = denture stability in covering as much basal bone possible without impinging on muscle attachment 1123. PT fractured many FPDs you made her, why? Bad metal frame work design – Repeated fracture of a porcelain fused to metal (PFM) is due primarily to an Inadequately Designed Framework. 1124. Why is there a cross-linking agent to dimethacrylate? So you can layer acrylic without getting craze/fracture lines. STRENGTH-for proper adhesion between incremental curing. 1125. When will Amelog. Imp. Have the most effect on the maxillary centrals? A. 1-6 months – calcification of maxillary centrals 3-4 months. (Laterals 10-12 months) 1126. What type of drug is PROZAC? Prototype SSRI (selective serotonin reuptake inhibitor) 1127. Max strength of porcelain? CONDENSATION - Max condensation, less porosity = stronger porcelain! 1128. Most caries in primary teeth seen where? distal to mandibular 1st 1129. In young patients, stains are more prominent on which area of the teeth? cervical 1130. Which of the following does a .02 taper indicate for a K-file? (1)0.02mm increase in diameter per 1mm of file length (2)0.2mm increase in diameter per 2mm of file length (3)0.2mm increase in diameter per 1mm of file length 1131. Picture of traumatic granuloma (pyogenic granuloma) in buccal vestibule, what is the Tx? Excision, If pregnant, lesion may regress after birth – BB 1132. NSAIDs works on? Platelet reversibly – except aspirin 1133. What causes gingival hyperplasia? Calcium Channel Blockers: Verapamil 1134. Which thyroid drug adds iodine to thyroxine decreasing its level – prothiouracil. 1135. Which antiretroviral causes pancreatitis and peripheral neuralgia – Stavudine 1136. Forcepts – elevation, luxation, compression, reflection? – Luxation – Elevation and reflection is periosteal, compression is fingers. 1137. THE BUCCINADOR MUSCLE IS PIERCED BY THE NEEDLE WHEN PERFORMING AN INFERIOR ALVEOLAR NERVE BLOCK. – DD 1138. Intraoral lesion of TB seen as – tonsillitis and ulceration (ulcer in the mid-dorsum or tip of the tongue) 1139. Hyperventilating : tachypnea and tachycardia 1140. Most caries susceptible tooth – maxillary 1st molar – upper 1st molar are most commonly affected (Kaplan) 1141. Must difficult to change – HUE, easiest – CHROMA 1142. Which of the following should NOT be prescribed for a patient receiving warfarin? A. Acetaminophen. B. Metronidazole. C. Penicillin. d. Codeine 1143. In pediatric patients 1. asthma has a decreasing prevalence. 2. asthma is an acute inflammatory disorder. 3. asthma leads to increased caries. 4. asthmatic attacks can be triggered by anxiety 1144. Positive Nikolsky: Pemphigus Vulgarys, Pemphigoid, Epydermolysis Bullosa 1145. Radiologic damage is less with: A. more oxygen B. decreased are of exposure (or less oxygen) 1146. Congestive heart failure: pedal edema, dyspnea and orthopnea 1147. Least sedative drug: Chlorpheniramine, Fexofenadine (second generation) 1148. A patient has a crown on tooth #30. On trying to close the jaw, the jaw deviates to the left. What is the reason? Interference on buccal inclines 1149. Which of the following is the most important factor affecting pulpal response? (1)Heat (2)Depth to which dentinal tubules are cut (remaining dentin thickness) - ASDA (3)Desiccation

MANNA BHATT RQs

1. K sparing drug: Amiloride, triamterene, Spironolactone Spironolactone

Thiazides Na and Cl cotransport Dec resorption of Na and Cl Hydrocholrothiazide - prototype Loop diuretic Na/K/2cl cotransport Inhibits resorption of Na and Cl Furosemide -prototype Bumetanide Ethacrynic acid Torsemide K sparing Na channel block: spironolactone Eplerenone - aldosterone antagonist: Amiloride Triamterene Carbonic anhydrase: Acetazolamide Weak diuretic Used in altitude sickness Osmotic diuretics Mannitol Glycerin Urea Used in edema after neurosurgery or trauma to CNS

2. Virax in oral cavity? Virax (Acyclovir) used for Rx of Herpes infection

3. What procedures you cant do in AIDS patient? Antifungal prophylaxis

4. Opiods effects. The short-term effects of opiate use can include: Feelings of euphoria, pain relief, drowsiness, sedation. Long-Term Effects of Opiates: Nausea and vomiting, abdominal distention and bloating, constipation, liver damage (especially prevalent in abuse of drugs that combine opiates with acetaminophen), brain damage due to hypoxia, resulting from respiratory depression, development of tolerance, dependence. 5. Papoose contraindication: Mentally compromised pt 6. Battery 7. Lot of prostho occlusal interference questions 8. Which study doesn’t show cause and effect: Cross sectional, and also examine two variables at the same moment. 9. Drugs those blocks prostaglandins has increased effect on gastric mucosa? No it decrease gastric mucosa and increase gastric acid (peptic ulcer). Aspirin and other cox inhibitors. 10. Patient with squeletal prognatic maxilla and lower class 3 molar relationship. What do u do per surgical ortho treatment? a.Labial movement of both upper and lower incisors b.Lingual movement of upper and lower incisors c.Labial movement for upper incisors and lingual for lower d.Lingual movement for upper incisors and labial for lower We do pre surgical with brackets bring lower lingually, and for upper surgically we do lefort 1. 11. Combination syndrome 12. Chs of band and loop 13. Least fracture resitant ; lithium, feldpathic, zirconia 14. Pka ( ph when drug is 50% ionized and 50% non ionized ) has effect on what? onset

15. 16 kg of 3 year old how many mgs LA to give? 16 x 4.4 : 70.4 mg

16. What meds you give in osteomyelitis? Clindamicin 17. Pt with bizarre behavior and disorientation you give what? ; insulin, glucose 18. Initial stages of sedation what pt feels? 19. Condensing osteitis? Excessive bone mineralization around the apex of an asymptomatic vital tooth. Radiopacity may be caused by low-grade pulp irritation. This process is asymptomatic and bening. It does not require endodontic therapy.

20. 2 questions on Periapical Cemental ossifying 21. What cyst in roots of mandibular premolar? lateral periodontal (also is the least common gral. cyst)

22. What lesions are not radiopaque? ; Adenomatoid Odontogenic Tumor (AOT), ameloblastic fibroma, odontoma

23. Least likely to occur, AOT, odontogenic myxoma Adenomatoid Odontogenic Tumor (AOT): Teens, females; anterior jaws; in association with the crowns of impacted teeth. Well – circumscribed unilocular RL lesion, may have small opaque foci. Enucleation, totally bening, encapsulated lesion that does not regres.

24. Which anticancer drugs effect on folic acid? Methotrexate

25. RG pictures Rg ameloblastic fibro odontoma 26. 9 year old kid swollen gingiva, recureent skin infections 27. what is complication of maxillary molar extractions 28. what is easily curable, macule, hematoma, or something? 29. 2 questions on Incisal guidance 30. RPD I bar fractured what you do? Soldiering 31. What does conjugation do to a drug? Make IT more water soluble

143. Problem with manual dexterity, what will he have problem with? Flossing Brushing

144. Dexterity comes by what age? A.3-4 years B.1-2 years C.5-6 years D.7-8 years

143. What surface of a tooth benefits the most from systemic fluoride Roots Pits and fissures Smooth surfaces

144. Which surface part of the tooth gets the least benefit of flouride? 1-Occlusal, 2-Proximal, 3-Root, 4-Facial The use of fluorides is the best approach to preventing caries. Fluoride, however, is believed to be least effective on the occlusal or chewing tooth surfaces.

145. Characteristic feature of Achondroplasia? -open bite -midface deficiency

145. Patient with achondroplasia, what will you most probably see a) class 2 b) open bite c) class 3

146. Which is more prone to injury in mand molar extraction : 1) IAN 2) Lingual nerve 3) None Trauma to inferior alveolar nerve may occur in the area of the roots of the mandibular third molars. Lingial nerve travels very close to the lingual cortex of the mandible in this area.

147. For a population, the researcher divides the number of disease cases by the number of people. By so doing, the investigator will have calculated which of the following rates: a- incidence b- odds ratio c- prevalence d- specificity

148. Orange stain is added to porcelain in order to? Decrease value, increase the chroma of a basically yellow shade. Staining a porcelain restoration will reduce the value (as will using a complementary color). It’s almost impossible to increase the vale. Master app: orange stain is commonly used to change the hue of porcelain.

149. A 50yr old male patient has been advised for multiple extractions in relation to tooth #4, #6, #15, #20, #22, #25. Which of the following is the correct extraction sequence? A) # 4, #6, #15, #25, #22, #20 B )#4, #15, #6, #25, #22, #20 C) #15, #4, #6, #20, #25, #22 - Canines are extracted last D) #25, #22, #20, #6, #4, #15 E) #15, #6, #4, #25, #20, #22

150. Which of the following would you NOT prescribe for a patient receiving Warfarin (Coumadin®)? 1. Acetylsalicylic acid. 2. Metronidazole. 3. Erythromycin. 4. Codeine. A. (1) (2) (3) B. (1) and (3) C. (2) and (4) D. (4) only E. All of the above. Metronidazole and Erythromycin inhibits warfarin metabolism. Aspirin inhibits platelet aggregation and causes bleeding.

151. Where would you look in a scientific journal to find the dependent and independent variables · Intro · Materials · Methods ** Moby pg 225 · Conclusion · Summary

152. Which antibiotic is appropriate for premedication in the penicillin allergic patient? a. Cephalexin b. Clindamycin c. Erythromycin d. Amoxicillin e. Ampiciilin

153. Which one is the most likely to promote proliferation of subgingival, black-pigmented bacteroides. A. Oral cont raceptives B. Propranolol Underal®) C. Chloroth iazide (Diuril® 154. The drug-of-choice for the treatment of adrenergically induced arrhythmias: quinidine. lidocaine. phenytoin. propranolol. Propranolol is a non selective B locker so it blocks adrenergically B1 receptor, B 1 receptor action is heart rate and force

155. Propranolol and epinephrine given together: epinephrine is adrenergic neurotransmitter causing increase in HR and PB... propranolol would block beta causing bradycardia instead

156. Which of the following locations would a perforation demonstrate the best prognosis? (1)Apical 1/3 of root (2)Middle 1/3 of root (3)Coronal 1/3 of root Perforations located close to the apex have better prognosis than those near the crestal bone (Google books) - talking bout root surfaces. "Coronal third of the root, the prognosis is poor." Mosby pg 20.

157. Study among smokers and nonsmoker for 6 years (2010-2016) to develop disease? 1 Cross sectional study 2 Cohort study 3 Case Control study 4 Interventional study Cohort study: prospective cohort study – a general population is followed through time to see who develops the disease. The investigators choose or define a sample of subjects who do not yet have the outcome of interest. Retrospective cohort study: used to evaluate the effect that a specific exposure has had on population. Investigators choose or define a sample of subjects who had the outcome of interest. They measure risk factors in each subject that may have predicted the subsequent outcome.

158. Case control study based on A risk B exposure. C disease D incident Case-control study: people with a condition (cases) are compared with people without it (controls) but who are similar in other characteristics. Hypothesized causal exposures are sought in the past medical records. Prevalence.

159. A study which is conducted in different cases to find out the etiology of different diseases varying in different subjects: A) case control study B) clinical trial C) cross sectional stud

160. Cross-sectional study: A. Descriptive B. Analytical C. Experimental

161. Hyperplastic may resemble which of the following? 1. fissuratum 2. Lingual varicosities 3. SCC 4.median rhomboid . 5. Prominent fungiform papillae.

162. Sarcoidosis resembles? a,Median Rhomboid glossitis b,Benign migratory glossitis c-granuloma No tuberculosis and histoplasmosis ..choose from above. Sarcoidosis is a disease of unknown etiology characterized by granuloma formation in a variety of organs.

163. Where does the epithelial for a graft come from? with a. Donor epithelium b. Donor connective tissue c. Recipient epithelium d. Recipient connective tissue

164. Ultrasonic scalers are contraindicated with composite restoration ... T When scaling porcelain and composites, the use of an ultrasonic scaler is contraindicated. Porcelain may fracture or lose marginal integrity. Composites have shown surface alterations, and amalgams have shown a loss of marginal integrity and surface damage.

165. The longest acting .most potent and most toxic LA 1 lidocaine 2 dibucaine 3 bupivacaine 4 tetracaine

166. Class II amalgam restoration has a overhang at gingival margin. This might have been caused by which of the following? a. poor adaptation of the matrix band b. poor carving c. did not wedge the matrix band Overhang - wedge Overcontour- matrix

167. Excessive VDo= less freeway space decreased VDO= more freeway space? T

168. Not reversible index is: a.perio b.gingival c.DMFT d.OHI-S

168. DMFT index is only used for permanent teeth. T/F

169. DMFT index limitation means? This only shows you the history of decay, missing, and filled teeth in ones mouth. It doesn't give you anymore info than that. So lets say the patient has root caries; DMFT will not tell you anything on that, or if the patient has any sealants, it will not give you any info on that either.

170. DMFT (decayed-missing-filled teeth index) – Best index DMFS (surfaces)

171. What will account for the anterior space for permanent mandibular incisors 1. Flaring max. Incisors 2. Primate space 3. Leeway space 172. Which allows more space for eruption of permanent mandibular incisors? Leeway space (For late shift) - Leeway is the diff in MD width bet primary C 1st n 2nd molars and perm C 1st n 2nd premolars. Primate space Leeway space - Difference in the size b/w primary posterior teeth and the permanent canine and premolars. Max- 1.3 mm per quadrant Mandi- 3.1 mm per quadrant Primate spaces- Naturally occurring space in primary dentition. Max- B/w lat incisor and canine... Mandible- B/w canine and 1st molar

173. 8 years old Mandibular second primary molar, painful You did extraction what space maintenance after A. band and loop B distal shoe (at 8 yrs old permanent first molar already erupted)

174. Little girl had ALL, had radiolucency in furcation of primary 2nd molar. What is the treatment? • Extraction • Pulpotomy • Pulpectomy

Radiolucency in furcation: In primary molars, the initial irreversible pulpitis radiographic sign is furcation radiolucency. Luglie found that 77% of the primary molars studied had accessory canals in the furcation area, explaining why the radiolucency appears there first. In permament molars the radiolucency appears at the apex because it is where most accessory canals are located. The size of the furcation radiolucency in a primary molar is nor a contraindication to pulpectomy.

Pulp therapy is contraindicated: Patients susceptible to bacterial endocarditis Patients with leukemia Patient with nephritis Patients with cancer Patients with depressed polymorphonuclear leukocyte and granulocyte counts.

Contraindications for pulpectomy: Nonrestorable tooth Internal or external root resorption Teeth without accessible canals (commonly first primary molars) Significant bone loss.

175. Carbide bur with more cutting blades A. rough surface at high speed B. Rough surface at low speed C. smooth surface at high speed D. smooth surface at low speed Therefore, low speeds are generally reserved for excavating decay, using disks, and polishing, while high and ultra speeds are used for bulk reduction, obtaining outline form, and removing old restorations. (Kaplan)

176. Antibiotic contraindicated with ALCOHOL are Metronidazole, Tinidazole, Antimalarial, flurazolione, Griseofulvin

177. Antibiotic contraindicated with ALCOHOL are Metronidazole, Tinidazole, Antimalarial, flurazolione, Griseofulvin

MARNE NBDE RQ’s ….. y más!

1. Proscar is used to treat? Proscar (Finasteride) is used to treat benign prostatic hyperplasia (BPH).

2. Motion sickness medication (not include antihistamines): Scopolamine The most effective single medication for prophylaxis against motion sickness is SCOPOLAMINE.

3. 3 years old child with 5 mm intruded: Observe Extraction Ortho eruption

4. Most common respiratory emergency Hyperventilating Asthma

5. Upper first molar forceps: 150

Maxillary forceps 99 --> Maxi anterior 150--> maxi premolar 18 53 88 (R & L) --> Maxi first and second molar 210--> Maxi 3rd Molar Mandibular forceps 103--> Mandi anterior 151--> Mandi premolar 13, 15,16,17,23--> Mandi first and second molar 222--> Mandi third Deciduous forceps 101--> for all teeth

6. 1997 law CHIP: T - For children in families whose income too high to qualify for Medicaid and private is too costly for them. Offers basic preventive and diagnostic services. Dental coverage was not a requirement in state (chip) then in 1997 its included in 49-50 states (mosby 221 page)

7. Dual cured vs light cured, color stability? T - Light cure more stable

8. Image: looked like Geography tongue, (but was not include in option) In lateral border of tongue, was not there 3 weeks ago Erytropakia: clinical term to describe any erythematous (red) area on a mucous membrane, that cannot be attributed to any other pathology. Geo. Tongue: inflammatory condition of the mucous membrane of the tongue, usually on the dorsal surface. It is characterized by areas of smooth, red depapillation (loss of filliform papillae) which migrate over time. The cause is unknown, but the condition is entirely benign (importantly, it does not represent oral cancer), and there is no curative treatment. Uncommonly, geographic tongue may cause a burning sensation on the tongue, for which various treatments have been described with little formal evidence of efficacy.

9. Nitroglycerin (antianginal = coronary artery vasodilator) side effects: The two most common adverse effects caused by nitroglycerin are orthostatic hypotension and headache DD131. It is sublingually effective within 2-4 minutes – 0.3mg). Antianginal drug: Nitrate: Nitroglycerin: This drug is the single most effective agent available for the management of acute angina episodes. Note: It dilates mostly veins. Indications: angina, acute myocardial infarction, and congestive heart failure. (FA)

10. Patient taking digitalis and diuretics. What's the patient is suffering from? CHF - Cardiac glycosides or “digitalis”, Digoxin is the most versatile and widely used. They are used to treat most SUPRAVENTRICULAR ARRHYTMIAS, CARDIOGENIC SHOCK AND CHRONIC HEART FAILURE. Drug interactions: many drugs affect digoxin levels. However, digoxin does not affect the levels of other drugs, In addition, when beta– blockers are added to digoxin in patients with AV conduction abnormalities, complete heart block can result. Erythromycin, clarithromycin and tetracycline may increase digitalis absorption and toxicity. Thyroid replacement therapy increases dose requirements of digoxin. Drugs that lower plasma potassium levels (Thiazide and loop diuretics) increase digitalis toxicity. Of both digitalis and diuretic given what the pt will have? It will increase digitalis toxicity and lead to arriyhymia.

11. 22 years old separated lesions in tongue and pharynge, fever, malaise. Herpetic gingivoestomatitis: Acute herpetic gingivostomatitis (also known as primary herpetic gingivostomatitis) generally affects children under the age of three and young adults. There are prodromal symptoms (fever, malaise, irritability, headache, dysphagia, vomiting, lymphadenopathy) 1 to 2 days prior to local lesions. Then small, yellowish vesicles form, which rupture quickly, resulting in shallow, round, discrete ulcers with an erythematous halo - DD Aphtous ulcer : also called mouth blisters, is a painful mouth infection caused by coxsackieviruses. Usually, herpangina is produced by one particular strain of coxsackie virus A (and the term "herpangina virus" refers to coxsackievirus A, but it can also be caused by coxsackievirus B or echoviruses. Symptoms include sudden fever with sore throat, headache, loss of appetite, and often neck pain. Within two days of onset an average of four or five (but sometimes up to twenty) 1 to 2 mm diameter grayish lumps form and develop into vesicles with red surrounds, and over 24 hours these become shallow ulcers, rarely larger than 5 mm diameter, that heal in one to seven days. These lesions most often appear on the tonsillar pillars (adjacent to the tonsils), but also on the , tonsils, uvula, or tongue.

12. Neurogenic sarcoma associated with what? Neurofibromatosis Neurofibroma may appear as solitary lesions or as multiple lesions as part of the syndrome called neurofibromatosis type 1 (von Recklinghausen disease of skin). The solitary neurofibroma is most commonly found on the skin: in the oral cavity, the tongue and buccal mucosa are most commonly affected. Derived from Schwann cells and perineural fibroblasts. Treatment: 1. Solitary: surgical excision – 2. Neurofibromatosis: removal is impractical due to number of lesions, it is best left untreated because multiple recurrences may be associated with malignant transformation to neurogenic sarcoma. The importance of the lesions is the high risk (5% -15%) of malignant transformation.

13. Axillary freckling, lich nodules. Neurofibromatosis type I (von Recklinghausen's disease). Clinical features: six or more café-au-lait macules that are greater than 1.5 cm. These are usually smooth-surfaced (Coast of California) / Two or more neurofibromas OR one plexiform neurofibroma (patognomonic for the condition) / Axillary freckling (called Crowe’s sign) / Iris hamartomas (called Lisch nodules) / Optic gliomas and several types of osseous lesions are associated.

14. 4 years old kid is Fearful Bossy Uncooperative

15. In US most dental pay is: private pay? Out of pocket

16. Histology shows parakeratinized epithelial lining and basal paragliding cells OKC (Keratocystic odontogenic tumor - KOTs): Most common in patients aged 10-40, multiple lesions found in children may be component of the “nevoid basal cell carcinoma syndrome” – Gorlin syndrome, the chief site of involvement is the mandible in the posterior body and ascending ramus, the tendency to grow in an anterior- posterior direction without bony expansion, carry a recurrence rate of 30%. Radiographic: well – demarcated area of radiolucency with corticated margins, unilocular or multilocular, they cannot be distinguished from other cysts radiographically. Microscopically: the lining epithelium is uniformly thin, (6-8 cell layers) and has wavy parakeratin, the basal layer is palisaded with intensely staining (hyperchromatic) nuclei, no rete pegs are present.

17. Researcher has set alpha 0.05. Results showed p value 0.01 and researcher reject null hypothesis. What type of error is it? / Experiment was done and error 0.05 was the goal of the experiment. After experiment was completed, the error was 0.01. The question asks what type of error was it? Type 1 error If p<0.05, reject the Ho: the observer outcome is judged to be incompatible and the alternative hypothesis is adopted. In this case, the results are said to be “statistically significant”. If p>0.05, accept the Ho. A type I error occurs when the null hypothesis (H0) is true, but is rejected. It is asserting something that is absent, a false hit. A type II error occurs when the null hypothesis is false, but erroneously fails to be rejected. It is failing to assert what is present, a miss.

18. Anterior teeth class 4 big composite done few weeks ago. The filling is acceptable but too light. What to do? Re do Observe Apply composite tint

19. How to differentiate endodontal and periodontal abscess Pulp testing .. same with vitality test Percussion

20. Patient had tooth extraction and wants to sleep at night. What analgesic do you prescribe? Naproxen (long lasting)

21. Anterior, fractured tooth needs crown lengthening. Which surgery do you do? Gingivectomy APF with Osseous contouring APF without osseous contouring

22. Veneer reduction on facial: 0.5 mm 23. PFM is too opaque on Invisalign 3 rd: insufficient 2 plane reduction

24. Central tendency question: Mean: adding, dividing Median: middle measurement in a set of data where half the data are above and half the data are below the number. Mode: most frequent FREQUENNCY DISTRIBUTION Positively skewed: large number of low scores and small number of very high scores. Negative skewed: large number of high scores and a relatively small number of low scores.

25. When is elective RCT? Contraindications Leukimia, uncontroll DM , recent MI

26. Gingivectomy indications: Gingiva Hyperplasia/Hypertrophy and suprabony pockets!

27. Gingivectomy incision? Above mucogingival junction 28. Remineralized enamel characteristics: Shiny and hard, darker 29. When does enamel start to demineralize (critical ph) 5.5 – T 30. Which is most important: value – T 31. Which show saturation of color: chroma Hue is color Chroma is saturation of color

32. Which is not a differential diagnosis of DI? AI DD Ectodermal dysplasia

33. When supernumerary tooth is seen? Cleidocraneal dysplasia T – Gardner too

34. Multiple osteomas seen in: Gardner 's 35. Patient is behaving bad but dentist ignored him and continues treatment. What hi is doing? Extinction

36. Aspirin side effect (multiple side effect in each option) Overdosage of salicylates (acute aspirin toxicity) is life threatening and requires intensive supportive treatment in a hospital. Initial symptoms include RESPIRATORY ALKALOSIS with HYPERPNEA and TACHYPNEA, NAUSEA, VOMITING, HYPOKALEMIA, TINNITUS, HEADACHE, DIZZINESS, CONFUSION, DEHYDRATION, HYPERTHERMIA, HYPERACTIVITY and HEMATOLOGIC ABNORMALITIES, progressing to COMA and RESPIRATRY COLLAPSE. Chronic aspirin toxicity: SALICYLISM, CNS EFFECTS, BLEEDING and GI DISTURBANCES. Aspirin is an irreversible platelet inhibitor and can reduce blood clotting to prolong bleeding. Low doses of aspirin taken regularly can have a cardio-protective effect. These doses reduce thromboxane production in platelets to result in the inhibition of platelet aggregation. In this way, aspirin has the ability to inhibit the formation of life-threatening thrombi (blood cloths).

CONTRAINDICATIONS: bleeding disorders (aspirin will increase bleeding time), do not use in children (Reye syndrome), pregnancy (specially during the third trimester), peptic ulcers (aspirin may cause bleeding of the GI tract), ASTHMA, RHINITIS, NASAL POLIPS, concomitant use of anticoagulants. 37. commonly found in? Max Lat 38. Fluoride replaces which group: hydroxyl - What the fluoride treatment does is replace hydroxyl groups with fluoride.

39. The size of radiolucency in #8 increase after treatment it can't be due to Apical scar Change in angulation Canal leakage

40. Chronic periodontitis most seen in? Black male – T

41. Contraindications of elective RCT? Recent MI, uncontrollable DM

42. Arbitrary articulator

43. Group function: Only possible when Anterior/Canine guidance absent! Otherwise posterior teeth disoclude! Group function occlusion is characterized by having working contacts. Sometimes called unilateral balanced occlusion, is an occlusal relationship in which all posterior teeth on a side contact evenly as the jaw is moved toward that side (working side). All teeth on the non-working side are free of any contact. The group function of the teeth on the working side distributes the occlusal load. The absence of contact on the non-working side prevents those teeth from being subjected to the destructive, obliquely directed forces found in non-working interferences. It also saves the centric holding cusps, the mandibular buccal cusps and the maxillary cusps from excessive wear. THE OBVIOUS ADVANTAGE IS THE MAINTENANCE OF THE OCCLUSION. Some relationships are not conductive to cuspid protected occlusion (canine – diclusion of all of the posterior teeth) such as CLASS II or end-to-end relationships. Some relationships are not amenable to group function such as CLASS II, deep vertical overlap. When placing a crown on a maxillary canine, if you change a canine protected occlusion to group function you increase the potential for a “non-working side” interference.

43. Balanced occlusion: All teeth contact during all excursive movements in complete dentures -- CR coincides CO - no anterior guidance

44. Orthostatic hypotension ( meds who can cause it): Opiods, anti hypertensives, nitrates, hypoglycemic

45. Medication interacting with nitroglycerin: EPINEPHRINE (physiologic antagonists)

46. Medication interacting with LA

47. Treatment for cardiovascular disease: Statins

48. Nitroglycerin, propanolol, and something else are all used to treat which of the following conditon? angina - Nitroglycerin is a vasodilator, dilates the coronary arteries for proper blood flow

49. Which of the following is not directly related to a drug toxicity of nitroglycerine? a. Dizziness b.projectile vomiting c.tachycardia d.Headaches

50. Which of the following medications increases the risk of bleeding? a) Dabigatran - Blood thinner. It can treat and prevent blood clots, reducing the risk of stroke. b) Sprinolactone c) Verapamil d) Nitroglycerin e) Losartan

51. A patient who uses nitroglycerine has A. rheumatic heart disease. B. asthma. C. coronary artery disease D. high blood pressure. E. cardiac arrhythmia. Indications: angina, acute myocardial infarction, and congestive heart failure. (FA)

52. Nitroglycerin dilates the coronary arteries in angina pectoris by a.Decreasing the heart rate reflexly b.Increasing the metabolic work of the myocardium c.Direct action on smooth muscle in the vessel walls d.Increasing the effective refractory period in the atrium e.Blocking beta-adrenergic receptor

53. Which antitubercosal drug inhibits the syntesis of arabinogalactan ?? a)isoniazid b)rifampin c)pyrazinamide d)rifabutin e)ethambutol

54. Systemic is associated with which medication? hydralazine, procainamide, isoniazid, all.

55. Which of the following drugs is associated with the reaction of stevens johnson syndrome? a.quinidine b.valproic acid c.ethosuximide d.isoniazid

56. which of the following agents is used for HIV infections? a. Amantadine b. Acyclovir c. Zidovudine d. Ribavirin e. Isoniazid

57. The most potent anti tubercular drug is Isoniazid (often given in a four drug regimen – bacteriostatic and bactericidal) Rifampicin: most active against bacteria undergoing cell division (bacteriostatic OR bactericidal) Pyrazinamide Ethambutol: active only against mycobacterium Because the mycobacterium organism tends to develop resistance to any single antitubercular drug, combination drug therapy is standard in the treatment of tuberculosis.

58. Which of the following has ototoxic adverse effects? a. Metronidazole b. Vancomycin c. Tetracycline d. Ceftaroline e. Isoniazid 59. Antibiotic most likely to cause failure of oral contraceptives Penicillin Rifampicin Tetracycline Cephalosporin Macrolide

60. Locally delivered antimicrobials used to treat infected periodontal pockets include all of the following EXCEPT A. Metronidazole B. Chlorhexidine C. Clindamycin D. Doxycycline fibres E. Doxycycline polymerics

61. What is the best antibiotic to be given in LAP (localized ): Metronidazole or doxycycline – best for perio

62. Which of the following would you NOT prescribe for a patient receiving Warfarin (Coumadin®)? 1. Acetylsalicylic acid. 2. Metronidazole. 3. Erythromycin. 4. Codeine. A. (1) (2) (3) B. (1) and (3) C. (2) and (4) D. (4) only E. All of the above. Metronidazole and Erythromycin inhibits warfarin metabolism. Aspirin inhibits platelet aggregation and causes

Patient who r taking Warfarin should not take the following Medications : 1- metronidazole & antifungal which ends with zole (ketoconazole) 2- antibiotics (tetracycline, macrolides) 3- antiplatelet (aspirin) 4- NAIDs

63. The most appropriate antibiotic for a periapical is A. pen V. B. cephalosporin. C. erythromycin. D. metronidazole E.ampicillin.

64. Which one mostly use for nausea vominting after surgery promethazin diphenhydramine chloropromasin

65. Which of the following drugs is most likely to dry secretions in the oral cavity? A. Diazepam B. Promethazine C. Physostigmine D. Propantheline E.Diphenhydramine

66. Each of the following non sedating antihistamines would be contraindicated in an individual taking cimetidine for heartburn except one. Which one is the exception? 1 astemizole 2 Diphenhydramine 3 fexofenadine - only antihistaminic to be taken with cimetidine 4 hydroxyzine 5 terfenadine

67. When patient has asthmatic attack on dental chair, all of the following can be given by IV except: 1. Epinephrine 2. corticosteroids 3. Aminophylline 4. Diphenhydramine – avoid antihistamines

68. Each of the following is an advantage of midazolam over diazepam EXCEPT one. Which one is this EXCEPTION? A. Less incident of thrombophlebitis B. Shorter elimination half-life C. No significant active metabolites D. Less potential for respiratory depression E. More rapid and predictable onset of action when given intramuscularly 69. Most BDz sedative used in dentistry ? midazolam - diazepam

70. What benzo do you give to a 37 yo pt with liver cirrhosis? a. Diazepam b. Midazolam c. Oxazepam Follow LOT (Lorazepam, Oxazepam, Temazepam)

MEPERIDINE (Demerol): narcotic that produce SEDATION and RELIEVES PAIN MIDAZOLAM (Versed): primarily anti-anxiety, short acting, great for short procedures. Comes as a liquid for pre-operative sedation in children and as injectable for IV conscious sedation. Benzodiazepines should never be taken with any form of alcohol. Serious potentiation of the sedative effect of each will occur leading to unexpected inebriation and respiratory depression. Flumazenil (Mazicon): a benzodiazepine antagonist, may be used to reverse the residual effect of benzodiazepines in the event of an overdose ADVERSE EFFECTS of NITROUS OXIDE: - Decreased mental performance, audiovisual ability, and manual dexterity. - AT high doses and/or high exposures: reduced fertility, spontaneous abortion, neurological and kidney disease as well as bone marrow suppression (DD#39)

71. Hyperbaric oxygen treatment is used in A) osteoradio necrosis B) bisphosphonate related osteo necrosis C) both

72. Which is the injectable bisphosphonate that can create complication in dental treatment ? Palmidronate , Etidronate

73. Bisphosphonate used for treatment of multiple myeloma – T

74. Which of the following represents the most common form of gingival periodontal disease in school-aged children? A. Juvenile periodontitis B. Localized acute gingivitis C. Primary herpetic gingivostomatitis D. Necrotizing ulcerative gingivitis

75. A 20 year old student presents with clinical symptoms of necrotizing ulcerative gingivitis (NUG). Food intake for the last 24 hours indicates a soft diet lacking in fruits and vegetables. The patient’s diet is important to investigate further because A. A deficiency of certain nutrients causes NUG. B. NUG may be limiting the food choices the patient is making. C. NUG can be cured through modification of diet. D. Patients with NUG lose interest in eating.

76. Metronidazole can be used to treat A. denture stomatitis. B. recurrent aphthous ulcers. C. necrotizing ulcerative gingivitis (NUG). D. primary herpetic gingivostomatitis

77. What is not indicated for management of acute necrotizing ulcerative gingivitis? Pain medication Saline rinses Light debridement Systemic antibiotics Topical steroids - because it will further depress immune system

78. Which of the following organisms are pathognomonic of acute necrotic ulcerative gingivitis A. Spirochaetes and fusobacterium SP B. Spirochaetes and eikenella corrodes C. Polymorphs and lymphocytes D. Actinobacillus actinomycetes comitans oral capnocytophaga E. Porphyromonas gingivalis and prevotella intermedia 79. All of the following should be considered for systemic antibiotic except A. Extraction of tooth with acute dento alveolar abscess B. Necrotic ulcerative gingivitis (NUG) unless it is acute. C. Extraction of 38 or 48 with acute D. Full mouth extraction for a patient with perio disease

80. Necrotizing ulcerative gingivitis and acute herpetic gingivostomatitis can be differentiated clinically by A. location of the lesions: interdental papillae involved in ANUG but not in herpes B. temperature of the patient. C. pain. D. lymphadenopathy.

81. In which of the following situations can topical corticosteroids be used? A. . B. Candidiasis. C. . D. Erosive lichen planus. E. Necrotizing ulcerative gingivitis

82. Which of the following periodontal disease causes the most rapid destruction of alveolar bone Periodontal abscess. Chronic periodontitis. Phenytoin induced gingival hyperplasia Necrotizing ulcerative gingivitis

83. Localized gingival recession of a permanent mandibular incisor in an 8 year old can be caused by A. vitamin C deficiency. B. . C. localized aggressive periodontitis. D. traumatic occlusion. E. necrotizing ulcerative gingivitis. NOTES: Diabetes doesn’t directly cause gingival recession but is a risk factor for periodontal conditions. may cause temporary pain and tooth mobility during occlusal contact but does NOT typically cause gingival recession. Necrotizing ulcerative gingivitis may cause gingival recession at some point but it is NOT a common dental condition.

84. What is mainly contraindicated for Orthodontics tooth movement? a) NSAIDs b) Corticosteroid EFFECT: ANTI-INFLAMATORY ACTION, IMMUNOSUPPRESSION, AND AN ANTI-ALLERGENIC ACTION. TOXIC EFFECT COCORTICOIDS: GROWTH INHIBITION, HYPERGLYCEMIA, OSTEOPOROSIS, PSYCHOSIS AND SALT RETENTION. Remember: pharmacologic effects of mineralocorticoids include an increase in sodium retention and an increase in potassium depletion, which can lead to edema and hypertension if excessive and may lead to dehydration and hypotension if insufficient. ADVERSE REACTION: Cushing syndrome, hyperglycemia, osteoporosis, peptic ulcers, and an increased risk of infection. (DD#129) 85. GUIDED TISSUE REGENERATION not done in? Shallow infrabony defects, grade 3 furcations, 1,2 wall defects

86. Which of the following tetracycline class drugs should only be taken once daily due to its long half life? (1)Demethylchlortetracycline (2)Doxycycline (3)Chlortetracycline

86. How do you determine arch length? Primary teeth - distal to to 2 molar to distal to 2molar / Permanent - distal of second primary molar to distal of second primary molar Arch width ? Inter canine distance

87. Which medicament can be used during pulpotomy procedure? a.calcium hydroxide b.EDTA c.MTA d.Flowable composite NOTE: Used in replacement of formocresol, however, because of the high cost, it is not often used.

88. Connective Tissue Graft is the most predictable treatment modality for root coverage - T

DD#101, 102, 103

SELECTIVE GRINDING IN WORKING – SIDE RELATION: RULL OF BULL - Buccal cusp inner inclines of Upper teeth - Lingual cusp inner inclines of Lower teeth SELECTIVE GRINDING IN BALANCING SIDE RELATION: LUBL - Lingual cusp inner inclines Upper teeth (NBDE2) - Buccal cusp inner inclines Lower teeth - Never grind the maxillary lingual cusp (primary centric holding cusps)

Working interference – LUBL. Correction- BULL Non-working Interference: Balancing side (non-working side) interferences generally occur on the inner aspect of the facial cusps of mandibular molars. Correction- LUBL

Protrusive Interference: occurs between the DISTAL inclines of the facial cusps on MAXILLARY POSTERIOR teeth and MESIAL inclines of the facial cusps of MANDIBULAR POSTERIOR teeth – DUML. Correction – MUDL (Grind MESIAL inclines of MAXILLARY teeth and DISTAL inclines of MANDIBULAR teeth).

CENTRIC: Centric interference (forward slide) can be corrected by grinding mesial inclines of maxillary teeth and distal inclines of mandibular teeth – MUDL.

Drugs and their antidotes 1. acetaminophen - acetylcycteine 2. benzodiazepine - flumazenil (Mazicon) 3. coumadin - vitamin k 4. curare - tensilon 5. cyanide poisoning - methylene blue 6. digitalis - digibind 7. ethylene poisoning - antizol 8. heparin - protamine sulfate 9. iron - desferal 10. lead - edetate disodium (edta), dimercaprol (bal), succimer (chemet) 11. lovenox - protamin sulfate 12. magnesium sulfate - calcium gluconate 13. morphine sulfate - naloxone hydrochloride 14. methotrexate - leucovorine 15. mestinon - atropine sulfate 16. neostigmine - pralidoxime chloride (pam) 17. penicillin - epinephrine 18. vincristine (oncovin) - hyaluronidase,also apply moderate heat to disperse drug and minimize sloughing. (oncovin - iv administration only)

Poison antidotes arsenic ------dimercaprol, succimer barbiturates (phenobarbital)------urine alkalinization, dialysis, activated charcoal beta-blockers------glucagon caffeine, metaproterenol, theophylline------esmolol carbon monoxide ------100% oxygen, hyperbaric o2 cholinesterase inhibitors------atropine cyanide------nitrite, sodium thiosulfate ethylene glycol ------ethanol gold ------dimercaprol heparin ------protamine sulfate iron salts ------deferoxamine isoniazid------vitamin b6 lead ------caedta, dimecaprol, succimer methanol ------ethanol, fomepizole, dialysis methemoglobin/cyanide poisoning------methylene blue muscarinic receptor blockers ------physostigmine opioids ------naloxone organophosphate cholinesterase inhibitors ------pralidoxime phencyclidine hydrochloride (pcp) ------ng suction quinidine, tca's ------sodium bicarbonate salicylates ------urine alkalinization,dialysis, activated charcoal snake bites ------antivenin tissue plasmogen activator (tpa), streptokinase------aminocaproic acid warfarin ------vitamin k, ffp alcohol ------Disulfiram (Antabuse)

INTERACTIONS: NITROUS OXIDE ------VITAMIN B12 synthesis in the human body by interfering with the enzyme methionine synthase, depleting the body of VITAMIN B12 (brain and nerve damage).

CONTRAINDICATIONS: COCORTICOIDS ------Latent TB or fungal infection, AIDS, herpes infections and patients with peptic ulcer disease (specifically, gastric ulcer) – these drugs themselves may cause peptic ulcers, congestive heart failure. Orthodontic tooth movement. (DD125) NITROUS OXIDE ------Head injury, bowel obstruction, pneumothorax, middle ear and sinus infections, COPD (emphysema or bronchitis – NOT ASTHMA, there ARE NOT contraindications for the use of nitrous oxide sedation in asthmatic patients), first trimester of pregnancy, with whom communication is difficult (autistic patients), having a contagious disease since it is difficult to sterilize entire tubes. NITROGLYCERIN ------Myocardial infarction with hypotension, hypotension, and glaucoma. (FA)

DARSHIKA SHAH'S Rq's

1) Three dentist hired a hygienist - patient injured by hygienist, whose liability? Hygienist Hygienist and attending dentist all d 3 dentist 4th dun remember

2) 3mm crowding in Mandibular ant permanent teeth? No treatment Extract primary canine Disk distal of primary canine And 1 more option

Early treatment: Mixed dentition minor crowding (up to 4 mm) may be corrected by proper utilization of the leeway space.

3)Class 2 dea prescription? Hydrocodone + ibuprofen = CLASS III Hydrocodone + acetaminophen = CLASS II Codeine + acetaminophen = CLASS III Oxycodone + acetaminophen = CLASS II

4) Tooth extraction? On day of dialysis 1 day prior to dialysis 1 day after dialysis No ext

5) Which requires elective root canal? Pain in tooth Exposed pulp Severely tilted Dun remember the 4 th option Sometimes the elective root canal treatment of a tooth with a healthy pulp is indicated within a broader framework of restorative care. If severely tilted, preparation of the tooth for a crown is likely to involve hitting the pulp horn, with the possible need for endodontics on that tooth.

6) Clamp no.212 (or some no. Dun remember exactly) is used for class 5 restoration Where to punch the hole? Long sentences asking where to punch the hole - facially, lingually

7) Class 5 - restoration of choice? Composite Rmgi = Resin modified glass ionomer Dun remember other 2 options 8)Most common in man? Haemophilia Diabetes Dun remember other 2 options

9)CaOh success depends on? Providing 2mm of gic base

10)Which material most stable in moisture Polysulphide Polyvinyl siloxane

11)Wat does not aid in restriction and resistance of crown? Parallel walls Surface area Less convergence 1 more option

12)Tooth crown ration fracture most common with? mandibular first molar (max CI in trauma horizontal fracture, mand molars - vertical fracture)

13)patient allergic to? Nickel Cobalt Chromium Berrylium

14)Patient I dun have time to quit smoking? Precontemplation Contemplation Denial Acceptance 15) Which is trademark name? Generic Registered Patent Something dun remember the exact question

16) Patient disoriention and something else? Insulin Glucose Epinephrine Atropine

17) Dementia? Short term memory loss Long term memory loss 18) Most common disease in old age? depression

19) Lingual flange recorded by? mylohyoid, genioglossus (lingual frenum), palatoglossus (retromylohyoid area), superior pharyngeal constrictor (distolingual extension). There is one option to choose will go with genioglossus it effect the length of lingual flange)

20) Distobuccal by? The distobuccal extension is determined by the position and action of the masseter muscle

21) Mandibular buccal frenum which muscle? Triangularis Zygomaticus Caninus One muscle from t dunno 4th option dunno Mandibular labial frenum = orbicularis muscle Mandibular buccal vestibule = buccinator muscle. The buccal vestibule: proper extension into this area provides the best support for the mandibular denture. This area is referred to as the buccal shelf.

The lower buccal labial frenum is: also morphologically similar to the upper buccal labial frenum but again less developed. It contains muscle fibers from the depressor anguli oris, or triangularis (another muscle of facial expression)

22)Which sinus involved in fracture ( I dun remember which)? Maxillary (Mx lefort one) Frontal Naso ethmoidal One more option

23) Which movement of eye restricted in fracture of floor of ? Sup Inf Lateral Medial

Type I orbital floor blow-out fractures FEATURES ' Limitation of ocular movement on up- gaze.

24) Question asking what is leeway space? Leeway space is the size differential between the PRIMARY POSTERIOR TEETH (canine, first and second molars), and the PERMANENT CANINE AND FIRST AND SECOND PREMOLAR - about 3.1 mm per side in the mandibular arch and 1.3 mm per side in the maxillary arch. (Mn 2.5,Mx 1.5 for each arch)

25) Scrap amalgam Sealed (with sulfide) Metal container Open Mercury chloride solution

26) Where is the retentive arm placed? Gingival 1/3 (Between middle and gingival third below high of contour) - reciprocal is made of base metal alloy and retentive is wrought wire

27) Metal can be used in denture base for the reasons except? Metal has good adaptation and resistance. Otherwise adequate contour is hard, no esthetic and poor retention. Metal allergy for some patients 28) The retentive arm of clasp features? Retentive arm is rigid ant 1/3 rd, semi rigid middle third, and flexible terminal 1/3rd. Passive until activated.

29) What does opaquer porcelain help in all except? Opaque porcelain masks the dark oxide color and will provide porcelain metal bond. Opaque does not make the main color of the restoration.

30) Y is gold preferred over amalgam? Ideal contour and very biocompatible. Gold thermal expansion near to tooth, gives ideal contours, better marginal integrity, more strength, also very biocompatible.

31) Which property is imp for burnishing the restoration? yield strength

32) The inflamed red spots in smokers palate? nicotonic stomatitis

33) Most common site of caries? Pit n fissure

34) Red complex which bacteria? PTT - Porphyromonas gingivitis, Tannerella forsythia and Treponema denticola.

35)10mA 1sec and .5secs same effect of the film What is the ma? 10 5 20 (inverse relation)- time is reduced by half so it will double

35) 1 film was given - underdeveloped and options related to it? Too high temp Reduced time: Underdeveloped(light film)- developer too cold, developer deplinish, inadequate time N more 2

36)16 kg girl max dosage of lidocaine? 16*4.4=70.4 4.4 mg/kg for lidocaine without vasoconstrictor and 7 mg/kg with vasoconstrictor for both child as well as adults.

37)Buccal root distal to palatal root. Where was d come placed? MESIAL – SLOB (SAME LINGUAL OPPOSITVE BUCCAL)

38) The bur used to polish porcelain? Steel Carbide Diamond (for cutting) 30-fluted carbide finishing bur is used to plane the porcelain surface and to remove the striations created by the diamond instruments.

39)The bur with more flutes? cut efficiently and polish efficiently Does not cut efficiently and polish efficiently Cut efficiently and do not polish efficiently Nor cut nor polish The greater the number of cutting blades on a bur results in LESS EFFICIENT cutting but a SMOOTHER SURFACE (polishing burs are of this type). A lesser number of blades on the bur results on MORE EFFICIENT cutting but a rougher surface (crosscut fissure burs at high speed or low speed are of this type).

40) The type of speed for implant site? High speed high torque Low speed low torque High speed low torque High torque low speed

41) The margin on cementum. Which material to be placed in gingival third? Glass ionomer cement ( GICs) or RMGIC W/ sandwich technique

42) Repair of porcelain process? micro etch, etch, silane bonding

43) Melanoma location? Palate and maxillary gingiva/alveolar ridge

44) Cause of mucocele? Mucocele caused by ruptured salivary duct, usually due to trauma, seen on the lower lip.

45) Treatment of ranula? complete excision with gland in new version of DD. Surgical excision if recurs the excise subling gland dbm.

46) Pierre robin was - Cleft palate 50%, retrognathia and .

47) This was related to Gardener. They didnt mention gardener anywhere just 3 symptoms wat is 4th that u wud check: Intestinal polyposis, supernumerary teeth, , dermoid cysts, and colon cancer.

48) Ration of cleft lip in Caucasian? 1:1000 Lip 1:1000 Palate 1:2000 Both 1:700

49) Chronic periodontitis common? Male Hispanic Female Hispanic Male black Female black

50) Prevelance can be related with which case study? Descriptive

51) Cause and effect which case study? Clinical trail if no effect then cross sectional study

52)Osteomas, glossptossis wat wud u check for? Gardner syndrome

53) Duct of submandibular gland? Wharton

54) Plastic heat sensitive materials sterilization? Cold (2%glutaradehyde)

55) Does not reoccur ? Ameloblastoma Odontogenic myxoma Aot – Adenomatoid odontogenic tumor (does not recur) dd card 112 Okc 56)12 month rct increase in size of radiolucency all the reason except? Healing by apical scar - healing by apical scar will not change in size Insicive canal Different angulation Leakage

57) Moa of suphauryl anti diabetic drug? increase insulin production by stimulating b cells of pancreas

58) Which can be diagnosed only histologically? difference between Radicular cyst vs Granuloma

59) After 12 months increase in size of lesion in rct treatment teeth except? Healing by apical scar Insicive canal Different angulation Leakage

60) Pulp necrosis type of wat resorption ? Inflammatory Surface Replacement b N 1 more option

61) Cellulitis- Neutrophilia Neutropenia Lymphocytosis One more option

62) OSHA blood borne pathogens - msds sheet: True - OSHA not related to MSDS sheet

63) Placement of pfm (porcelain fused to metal) crown margin ant? Supra At crest of gingiva In between crest n epi Crown-to-root ratio: Minimum—1/1 Best—1/2 Shoulder margin – when esthetics are imperative (minimum of 1 mm of porcelain can be stacked at the margin hiding the metal)

64)N d wat makes penicillin allergic? Beta lactame ring

65) Patient has BCC how wud u tell d patient? Good prognosis do you need to reappointment this cancer in not as dangerous as others do you want I called you guardians the biopsy show that you have a cancer and we do our best to help u out

66) Wat is best X-ray for detecting bone loss? Bitewing

67) Most common finding of cherubism? Bilateral swelling of jaw with premature loss of prim teeth and delay eruption of perm teeth, soap bubble appearance.

68) Potassium sparing drug? spironolactone

69) Gingivectomy indications contraindications? Indication - supragingival pocket, enlargement / Contraindication - infrabony pocket, inadequate width of attached gingiva.

70) facial vertical axis divided? 3 equal thirds – Its 3 plane vertically, 5 planes horizontally

71)cleidocranial wat is absent: clavicle

72)most common salivary gland Tumor: pleomorphic adenoma (mixed tumor)

73) lots n lots of questions related to case study . Which case study tells wat.

74) lot of patient management questions related to desensitisation mostly

75) consent not taken? Which ethic? Violation of Autonomy

76) treatment with out consent? Battery - legal term in which perform a procedure without consent

77) liquid in gic: Polyacrylic acid

78) statements true reg zinc poly carboxylate: Good strength not irritating pulp replace zoe chemical adhesion high film thickness Chemical bond to enamel

More More RQs 1. What procedures you cant do in AIDS – Prophylaxis 2. Chs of band and loop - Given early loss of 1st primary molars 3. What cyst in roots of mandibular premolar - Lateral periodontal 4. What lesions are not radiopaque? ; AOT, Ameloblastic fibro odontoma – ameloblastic fifroma – DD: An opaque focus appears within the ameloblastic fibro-odontoma owing to the presence of an odontoma. This lesion, therefore, presents as a combined lucent-opaque lesion, the ameloblastic fibroma is completely lucent. 5. Least likely to occur, AOT, odontogenic myxoma? 6. What is Brown tumors – Hyperparathyroidism 7. Conditions have macroglossia? - Acromegaly, beckwith wieddeman syndrome, Hypothyroidism,amyloid oasis,down 8. You don’t do RCT with PA radiolucency in which patients? Diabetes (uncontrolled DM) 9. Why you clean tongue? ; esthetic or to prevent odor 10. Treatment of concussion – do nothing 11. Too light x-ray? Why - Developer temp cold,under devolped,exhausted devolper 12. Phelobith location? ; nerve, vein 13. If you take Rg 10 ma with 1 sec exposure and you take another x ray with .5 sec what MA will you give you want same density rg? 5, 10 or 20 (Inversely proportional) 14. Mouth breather features - open bite constricted maxilla post cross bite, narrow arch too, short upper lip 15. Mechanism of fluoride? - Hydroxyl group from hydroxyapatite exchange to form fluorapatite 16. Lefort 1 includes what structure? Maxilla 17. When you make rest on molar you make rest deep enough in marginal ridge, buccal incline, lingual? 1.5 mm marginal ridge 18. Epinephrine contraindicate in thyroid? T/F - T in In hyperthyroidism 19. Abcess include marginal and interproximal gingiva called? Gingival abscess, pericornal, periapical 20. Whats the indication of half erupted third molar in 18 years old – extract? but 17 -21 is when normally 3molars are erupting. In ADA specifications.. Extraction of 3rd molar is not indicated if it is not causing any problems 21. You can avoid legde if canal is Small, large, short or curved? Short 22. # of granulocyte for sugery – 15000 23. Beta 1 selctive actions - Increase heart rate 24. Papoose contraindication - Mental retarded child, Long procedures, cooperative child 25. Hemophilia test – PTT 26. Alplrazolam - Anxiety and panic disorder 27. 5 year old child not cooperative, wat to do? conscious sedation? voice control? ga ? 28. Precations for hep patient - why universal precaution? Its changed and expanded to be renamed as standered precaution card 63 pt 29. Endo perio operative surgery ortho 30. Patient smokes, is implant contraindicated – NO (its risk factor) 31. Primary molar differs from permanent in cervical constriction T 32. Forcep not used for extraction of root 33. Most common organism in implant failure; same as regular dentition – Anerobic 34. Ginseng reacts with which drug – aspirin

35. Ectodermal dysplasia features - Anadontia , conical ant teeth , thin hair , mostly in male , x link. Anhydrosis. 36. Cleidocranial dysplasia what is missing? clavicle 37. Identify periapical cemental dysplasia - Cemento-osseous dysplasia (COD) is a benign condition of the jaws that may arise from the fibroblasts of the periodontal ligaments. It is most common in African-American females. The three types are periapical cemental dysplasia (common in those of African descent), focal cemento- osseous dysplasia (Caucasians), and florid cemento-osseous dysplasia (African descent). Periapical occurs most commonly in the mandibular anterior teeth while focal appears predominantly in the mandibular posterior teeth and florid in both maxilla and mandible in multiple quadrants. 38. Dentigerous cyst radigraph

39. Regional odontodysplasia identify - Ghost teeth 40. which week does dental lamina appear? 6 week 41. Piree Robinson syndrome - Cleft palate, retrognathia and glossoptosis 42. Ranula treatment - Marsupilization or excision with remove sub lingual gland 43. Convergence of buccal and lingual wall which tooth? Primary 1st max molar, 44. Distolingual extension of mandibular denture which muscle - Distolingual just superior constrictor muscle (DD) 45. Which sounds will you look for placing anterior teeth. - Fricative (labiodental) F and V, formed between the maxillary incisors contacting the wet dry lip line of the mandibular lip. This sounds help determine the position of the incisal edges of the maxillary anterior teeth. 46. Incisal edge of max anterior touch were? On the vermilion border 47. Major connector function? Stability and rigidity 48. In distal extension where do you put rest? mesialy 49. Most esthetic pontic design? Ovate 50. Where to place margin of anterior crown - Subgivingival 51. Lot of clinical qs presenting different scenarios and you have to choose appropriate treatment 52. 3- 4 qs on occlusion. But most of them direct 53. Necrotic pulp with open apex what is treatment of choice? Apexification 54. 1mm exposure what do you do? DPC, pulpotomy,PUlpectomy 55. Patient comes after 1 week of composite says he does not like the color what will you do? Repeat restoration, add composite and some other options - Redo 56. Which bur do you use for porcelain polishing? Steel, carbide ,diamond 57. Action of beta blocker on smooth muscle? Since B2 adrenergic receptors can cause vascular smooth muscle dilation beta-blockers may cause some vasoconstriction. 58. Action of beta blocker on cardiac muscle - Beta Blockers are drugs that slow the heart rate, decrease cardiac output, lessen the force with which the heart muscle contracts and reduce blood vessel contraction. - Dec cardiac output,dec hr 59. Which patient requires antibiotic prophylaxis? 2 qs about that - Prosthetic valve, Cyanotic, Previous endocarditis, Valves problems from recently inplanted heart 60. MOA of sufonylurea - Inc insuline by stimulation of beta cell in pancreas 61. All are immunosuppressive except? Some wiered choices – Methotrexate, Mecaptopurine, Cyclosporine (All are immune suppression ) 62. Antimetabolite for cancer which affects folic acid? METHOTREXATE 63. Premolar forcep all except? 150,151 for pm, 99, 203 – EXCEPT 23 64. Bacteria in chronic periodontitis? DD: P. gingivalis, T. forsythia, P. Intermedia, C. rectus, E. corrodens, F. nucleatum, A. actinomycetemcomitans (Aa). P. micros, and Treponema and Eubacterium species. 65. Bacteria in NUG? Fuso, Spirochete, Prevotella 66. Gingivectomy where to put incision? A beveled incision is made apical to the pocket depth. 67. What does biologic width comprise of? JE + CT = 2.04 68. What makes up the periodontium? GIngiva, Alveolar bone, Cementum, PDL 69. Most common complication of extn of max molar? Alveolar fracture - Sinus perforation 70. Lefort 1 which sinus affected? Maxillary, ethmoid,frontal,mastoid 71. A boxer wearing mouthgused comes with complain of jaw opening in the morning. What is your diagnosis? MPDS 72. Light radigraph cause? Under developer 73. Lot of qs on patient management. Some straightforward some twisted wording. Moby is must 74. Cold sterilization which chemical used? Gluteraldihyde 75. Sodium hypochloride does all except? Chelation 76. Common cause of failure of amalgam restoration? Moisture contamination, improper design, improper trituration. Failure- inadequate design - Fracture- water contamination 77. There is a fracture line on amalgam restoration what is the treatment of choice? Redo restoration to check the fracture of tooth, Just remove mesial side and check and some other options 78. Lot of class 5 what is the material of choice? GIC 79. Which impression material is moisture tolerant? PVS, polyviny, polyether 80. Which material has inherent property of binding with water? PVS, Polyvinyl, polyether (hydrocolloid was not an option) 81. Modulus of elasticity which property? Siffness (DD) 82. Finger spring applied to move the tooth where will be the most force applied? Tooth dstal,medial - mesio distal direction 83. Nitrous oxide most common side effect? Just nausea and vomiting (dd) 84. 16kg boy what is the maximum dose of LA - 16*4.4 (dose/kg) = 70.4 85. A defiant child does not want to cooperate which method to use? GA, voice contol, HIM 86. Gold standard for behaviour modification? Systemic desensitization, modelling,voice control 87. Lot of clinical qs about indications and contraindications of material use in restoration? 88. Day 2 practice ASD A PAPERS and treatment options for various ortho and perio cases.

MORE RQs 1. Which immunoglobulin is concentrated in gingival clevicular fluid: IgG 2. Middle-aged male has a fluctuant mass in the midline of neck: A) Thyroglossal duct cyst B) Brachial cleft cyst. 3. What else do S. mutans produce along with dextran after breaking down sucrose: A) mucopolysaccharides B) macros C) levans D) proteins 4. Nerve involved in Bell’s palsy: VII 5. 4mm implant, how much do you need buccolingually: 6mm 6. Mandibular 3rd molar root lost: which space ? 7. IAN block needle infection where: Pharyngeal, Pterygoid, medial pterygoid muscle 8. Veneer facial reduction: 0.5mm 9. PFM buccal margin depth - 1.5mm 10. Patient comes back after 1 year of composite restoration with pain and sensitivity – microleakage 11. Radiograph to check integrity of Zygomatic arch – CT/ Submentovertex 12. TMJ radiograph - MRI for articular disc 13. Behcet’s syndrome associated with: Aphthous ulcers 14. Bradycardia treatment - Atropine, scopolamine 15. Which is NOT used to inhibit salivary secretion - Pilocarpine or Cevimeline 16. Which is contraindicated in nitrous: nasal congestion – COPD 17. Which of the following confirms the diagnosis of xerostomia: A) location of probing depths of >4mm. B) location of anterior restorations. (I picked this instead of A because chemotherapy causes xerostomia which leads to class V lesions) C) location of partial denture flange 18. Over titration of Amalgam leads to: decrease setting expansion, decrease corrosion, increase strength 19. Cavernous thrombosis infection via - lymphatic vessels.. as are on the upper lip is valveless (upper lip infection) 20. Patient does not have tooth #11 and has all the premolars, which one has the Worst Prognosis: A) fixed bridge from #10-12. B) RPD with pontic for #11 C) Implants 21. Metalloceramic crown which type of bevel - Chamfer 1mm 22. On working interferences and how to correct it - BULL - working side LUBL - non workin, MUDL - centric and DUML - protrusion 23. Wear facet on the mesio marginal ridge of MB cusp of mandibular molar - Protrusive interference 24. Macroglossia where is not seen (hypoparathyroidsm) 25. Most prominent cells in crevicular fluid – PMN 26. Ignoring patient behavior - Operant extinction 27. A 5 years old fell and max incisors intrude 3 mm ( tx) – OBSERVE 28. Ectodermal hypoplasia - Affect all structures derived from ectoderm: Anhydrosis, Spars hair, Anadontia or hypodontia, fine hair, delicate skin, no sweat glands, nails deformed. 29. Prominent cell in cellulitis - lyphmphocyte , its chronic 30. Most tooth lost in perio treatment - Max 2nd molar 31. Where external bevel incision is made – Giginvectomy - Used in gingivectomy and Its done apical to pocket (junctional epi) and coronal to mucogingival junction. 32. Disadvantages of modified widman flap - Not eliminate pocket depth, But it removes pocket lining and then shrinkage and reduction 33. Cleidocraneal dysplasia - No clavicle - Supernumerary teeth delay erruption if teeth. Frontel boosing 34. Early lost of primary teeth Papillon le fever - Papillon levefe, autosomal recessive, impaired in T and B lymphocyte, palmer-planter keratosis , advanced periodontitis in both primary and permenant due to dominant bac (A.A) teeth float in soft tissue, excessive bone loss and mobility. 35. KOC how to diagnose it - Microscopically , the lining epithelium is thin and Parakeratinzed , basal layer is palisaded with prominant staining nuclei ,, the lumen cintain large amount of KERATIN debris and clear fluid similar to serum transudate( dd) 36. Dentigerous cyst (radio and patio) - Impacted / unerupted - radiolucency > 5mm upto cej - Cyst is lined by epithelial cells derived from reduced enamel epithelium 37. Why to extract bone impacted 3 molar - caries, recurrent pericoronitis, improper eruption path, resorption of agjacent rooth. 38. Modeling reshaping behavior - successive approximation - Behavior shaping a.k.a. successive approximation shaping is used when an existing behavior needs to be changed into a more appropriate or new behavior. The strategy involves use of reinforcement of successive approximations of a desired behavior. 39. Cross section study - No cause and effect 40. Bundling/unbundling - Bundling pay all procedure together - Unbundling separate charging. 41. MOA of sulfas - If its sulfanylurra then stimulate insulin from b cell of pancrease (Stimulate beta cell to inc insulin) - If its sulfonamide then ab compete with gaba and inhibit folic acid syng 42. Epi should be avoided in excess of: thyroid hormone, Yes because it causes hypertensive crisis with Grave's disease 43. Max dose of LA for 3 years old with 16 kg – 70.4 44. Changing the exposure time and how will change MA with the others factors remaining the same – (For example: ma :10 in 1 sec - If exposure time changed to .5 seconds thn MA will be 20 or 5 ?? 45. Hardest type of ceramic: Zirconia (From weak to hardest: feldespathic, leucite- reinforced, lithium disilicate, glass-filled, aluminia, zirconia) 46. Why Zirconia is one of the hardest ceramic? - Due to its sintering technique -- slip cast technique which increases toughness 47. Diffencial diagnosis of perio endo lesions – vital pulp test (ept) 48. Order of treatment in case of perio – Emergency, Control, Reevaluation, Maintenance 49. The most important factor in choosing color for a crown – Value 50. Finish line of gold crown - in dd ideally its feather edge , but chamfer is used. 51. Finish line of PFM - All CERAMIC shoulder, GOLD chamfer, PFM buccal shoulder and lingual chamfer.. but if not given.. chamfer considered. 52. # of bur used with gold onlay – 557 53. What not to use for pm (# of forceps) - 23 cowhore 54. Safe analgesic to use in pregnant patient (acetaminophen not an option) Tylenol 3 55. Class 2 narcotics – Percocet 56. Medication use in Parkinson's - Levadopa, carbidopa, Amantidine 57. Alcohol addiction and use of Tylenol - Liver toxicity, so it must be avoided 58. Regional odontodysplasia - Ghost tooth, short roots , open apical foramin, enlarged pulp chamber, the thinness and poor mineralization quality of enamel give it the gost shape tooth, affect permenant more than primary, max anterior is more affected 59. Sjogren syndrome - Dry eye(xerohthamia), Dry mouth xerostomia, Rheumatoid arthritis 60. Meds in miastenia gravis – Neostigmine + atropine y pyridostigmine. Endrosphonium to diagnoses Myastenia Gravis, not for treatmrnt because of short duration 61. Side effect of albuterol - Canidida and xerostomia (Dryness of mouth inc susceptabilty to candidal infection) 62. Side effect of nitroglycerine: Orthostatic hypotension, Headache, Nausea, Flushing of face 63. How does the insurance make money-1-pay per case 2-Barter, HMO, 64. There are only mandibular anterior in the mouth and planned was the CD... what do u look for?1-Balanced occlusion 2-Canine guidance 65. Maxillary molar thin attached gingiva what you do not do on the distal-FGG, Apically PF, Distal wedge (Apically displaced flap is done in adequate attached gingiva) 66. Nitrous oxide mechanism of action - It works on central nervus system( reticular activating system ans limbic ) its sympathomimetic, the only inorganic used , weak anesthetic and strong analgesic , used in conscios sedation ( stage one anesthsia )(dd) 67. Dens in dente most common involved tooth – LI 68. Most common anomaly-DI, AI DI-1 in 7000 69. Worst possible outcome after splinting the avulsed tooth-Inflammatory resorption , Replacement resorption - external root resorption( inflammatry) 70. Down syndrome what do u see-enlargement of maxilla, mandible, Mid-face def 71. Biological width from where to where - Junctional epithelial + C.T = 2.04 72. Menarche- skeletal maturation, cognizant maturation, emotional maturation 73. Characteristic of arrested caries-they did not mention anything about colour but came up with soft under enlarged gingiva, gingival recession. 74. Epinephrine what it does, Prolong the duration of LA, vasoconstriction 75. Intrinsic and extrinsic pathway of NSIAD - Lipooxygenase and cyclo oxygenase inhibition pathways - nsaid dont ecffect neither intrinsic nor extrincic pathway of coagulation,,, it inhibit platalet aggregation by inhibiting thromboxane A2 76. Main disadvantage of GP cones: Does not adapt so requires Zoe 77. Asymptomatic pigmented lesion on the palate 4mmx3mm what you will do for it ? A) cone beam and excisional biobsy b) cone beam and excisional and antiobiotic treatment C) no treatment. 78. Whats the heart rate for a child 3 years I believe – 110 79. The most chronic peridontistis will be found on - black males 80. SNA, SNB, ANB- related 82-80-2 - SNA mx to cranium 82 normal More mx protruded Less mx retruded - Snb mn to cranium 80 normal More mn prot Less mn retr - ANB mx to mn 2-4 More class 2 Less than 2 three 81. Primary tooth intrusion what u do- Observe 82. Rct contra - Leukemia , uncontroll DM , recent mi 83. Patient stand on the corner looking at the floor and his hand is folded together after you give him and introducation what you ask him or do next - What bring u here today 84. Xylitol can be most effective in reducing caries by - Increasing salivary flow, inhibiting formation of bacteria 85. If you have 2 proximal cavities how to manage? Prepare larger first, fill smaller first 86. Epilepsy which drug, grandma and petit – Phenytoin, Peti is exothusmide - grand mal- phenytoin 87. Germination,fusion ,,erosion what are they know definition 88. Pregnancy safe drug – Promathzine, Tylenol 3 (Amoxylin, Aceta, Tylenol 3, Promethazin) 89. epi +proponolol what will happen? Hypertensive crisis - Increase BP accompanied by reflex bradycardia 90. Xray to identify zygomatic arch - Smv and CT 91. Myxoma: honey comb, most common odontogenic mesenchymal tumor, Benign mesenchymal tumor. Hony comb appearance and tennis racket. May show sun ray app of osteosarcoms. Cz displacement but not resorption of roots, arise from follicular connective tissue resembling dental pulp tissue. Odontogenic myxoma (bromyxoma). 1. Uncommon to rare tumor of myxomatous connec- tive tissue (primitive-appearing connective tissue containing little collagen similar to dental pulp). Either jaw affected. Radiolucency, often with small loculations (honey- comb pattern). Treated with surgical excision; moderate recurrence potential owing to lack of encapsulation and tumor consistency. 92. Ameloblastoma, Benign, Most common epithe tumor, Soap buble with local invasion, Mn post. 93. Pagets, Max affect more, Hyoercementosis , Cotton wool appearance, Mx, Cotton wool, Inc alkphosphotase, Osteosarcoma, Cotton wool, and loss of lamina dura. Tendency to malignancy osteosarcoma. 94. Gingivectomy incision - External bevel 95. Intracanal medicament – Caoh 96. Crossbitecase,case 97. About ageing of amalgam what happens - Inc marginal seal 98. Difference betn reversible and irreversible 99. Most common in school going - Marginal gingivitis 100. What is diff betn infected and affected - Infected we have to remove and not remenerilze. 101. Which force for implant - horizontal is worse 102. Some q about sinus lift - Bone graft at the floor of sinus for implant 103. FN plane - Porion orbitale (Porion to orbitale Frankfort plane) 104. Bestxray for mesio dense - Occlusal 105. Porcelian fracture - Porosity in porcelain is result from inadequte condensation card 29 pros) ,,, porcelain fracture in pfm , is poor design is main coz of fracture (dd) 106. Nitous oxide contra - Nasal congestion, 1st tri, Head injury, Mental retar, COPD 107. .1st primary erupt at 6 to7 MNTHS 108. Most impacted tooth - Lower3rd 109. Paresthesia which fracture? Angle 110. 100% Humidity of alginate – Irrevesible, Synersis 111. About informed consent what it doesnot contain – COST 112. Max anterior common perforation where? - Mesial 113. Drugs for xerostomia - Pilocarpine cevimeline 114. Treatment of xerostomia - stop medication or modify them use of saliva subistitute or use of hard candy sugar free 115. Atropine - Cause xerostomia, Anticholenrgic 116. Cholinergic - Inc secretion, Miosis, Reduce bp, Inc gastric motility 117. Antichoinergic - Atropine, glycopyrrate, propanthaline, scopalmine 118. Antidepressant – Tca, Moa inhibiters 119. Read about gracy and universal curette, also about specificity ,sensitivity calculation. 120. Collimination function - Reduce x ray exposure, lead 121. Whitexray pics cause - Not enough developing 122. Chin up pic - Reverse smile – frown 123. Implant q one is temp - 47- less than 1 mint 124. Other was about antirotational element - Hex either external or internal 125. About hue, Color, Index100, Hue selected first 126. What u select 1st hue chorma value? HUE 127. What you cant change hue chorma value 128. h2 blocker ranitidine - TRUE 129. Sulfynoyl urea moa - Stimulate insulin by acting on beta cell 130. Moa of sodium hypochlorite, Irrigation..5.25% ,removes organic debris, antimicrobial 131. Sodium hypochlorite is not chelating - T 132. Most radioresistance - Muscle and nerve 133. Verrucose leukoplakia-site - Buccal mucosa 134. Terfenadine -- erythro contraindicated (T...Terfenamide (sedane) c.i for erythromycin)

Ana Karina Perez RQs

1. Patient hip prosthesis with a strok a year ago you give prophylaxis or call to the doctor - No need to prophylaxy in both situation 2. First symptom of Adrenal gland dysfunction? joint pain and pigmentation 3. Sialolith in which gland? – submandibular 4. Erythroplakia (erythroplasia)1. High-risk, idiopathic red patch of mucosa. 2. Most represent dysplasia or malignancy. 3. Biopsy mandatory. Much less common than idiopathic leukoplakia. Cause unknown (idiopathic), some are tobacco related Usually occurs between 50 and 70 years old. High-risk sites: floor of mouth, tongue, retromolar area. Microscopy: Mild to moderate dysplasia (10%) Severe dysplasia/carcinoma in situ (40%) Squamous cell carcinoma (50%) 5. Multiple myel. resemble to? idiopathic histiocytosis, both have punched out appreance 6. Neuropraxia? Neuropraxia is a type of peripheral nerve injury, and is known as the mildest form of nerve injury. It is classified as a transient conduction block of motor or sensory function without nerve degeneration, although loss of motor function is the most common finding. 7. Advantage of auto cure composite - No light needed 8. What happen when you move when take a pano – Distortion due to patient movement: a. Movement in the same direction as the beam. There is prolonged exposure of the same area, with increase in horizontal dimension of the image. b. Movement in the opposite direction as the beam. The horizontal dimension of the image in the region is decreased. (In the option was distortion below and too I think is below) 9. Dif between luz led and halogen - No heat - no fan - so light weight, requires less power- so battery - so portable. No retinal damage 10. Open coil between two teeth how is the force: Continuous or intermittent - coil give continius force not decrese like elastic ... coil still active all time 11. Concusion - No treatment. Tooth is neither displaced nor loose but respond to percussion 12. Best cement for veneer – resin cements (light cure) 13. Men with 54 years old with post cross bite best tto: expansion with appliance, surgery or do nothing - Surgery its same q in first aid 14. Complex odontoma and compound odontoma - Compound—ant Complex—post 15. I had a question of ginkgo biloa – Anticoagulant, Not given with aspirin and warfarine, and used as periphral artery vasodialator. 16. Demensia i had 2 q about this - Alzheimer's- short term memory loss – dementia 17. Patient inject local anesthesia iv what happen - Cns excitation, 1st sign of lydo toxicity: ligtheadedness, dizziness, visual and auditory disturbances, disorientation, muscle twiching, convulsions. 1st sign from epineprine: elevated pulse rate (symphatomimetic) followed by CNS depression. 18. Limit amount of exposure more in which? digital imaging 19. Function of conector major: Stability and rigidity 20. Patient white 50 years old have little caries what do you put: flúor, composic, glass ionome 21. Newly erupted tooth ehat tests for vitality – Cold 22. If the patient had good behavior before what do you do: tell show do or give a gift - Positive reinforcement 23. What is your name when you attend a nervous patient and compare it with the good behavior of the patient who are attend it - I think the q meant about modeling (observational learning ) 24. Común tooth with caries - Mand 1 molar 25. Común tooth with fracture - Mand 1 molar 26. Común tooth with periodontist - Max second molar 27. Reduction of porcelain - 2 mm for both metal and porcelain .5 mm for metal and 1 to 1.5 for porcelain 28. Interaction of amoxicilin with methotrexate - Yes, prolong the action of methotrexate 29. Tto of acute manic episode – Lithium 30. Tto for recession - Lateral repositioned flap (pedicle graft) in small recession area, for large recession area I think we do free gingival graft 31. Phonetic problem associated with f, v - Ant Teeth place too far anteriorly and superior 32. Which test is used to differentiate between endo and perio lesion - Both vitality and percussion test, ept (dd 2 cards mention that) 33. Which is worse to lose the first molar inf or a second molar in inf - Second molar 34. Most rigid impression material – Polyether 35. Highest chance of pulp necrosis – avulsion 36. Advantage of compound impression - Good detail 37. What can we use for sedation for a pregnant breat feeding woman – Promathazine 38. How Do you do better cleaning with ultrasonic with a thin point or more vibration - Moooore vibration 39. Tto to ranula – Marsupilization, Excision along with gland as per new dd 40. Which gland ranula – Sublingual 41. Tto to sialolith - Conservative tt is saliva stimulants or for larger surgical 42. Which study doesn't show cause and effect - Crosssectional (case control shows cause and effect..cross sectional doesn’t) 43. To what space go the tooth of 3 molar inf when yo found extraction - Retro pharyngeal 44. Pka has effect on what – Onset, onset, with low pka will have faster onset of action 45. Multiple odontoma in which syndrome - Gardner syn 46. Patient with bizarre behavior and disorientation you give what – Glucose 47. Tto to ADHD - amphetamine was in the option (amphetamine was in the option) 48. What do you do in a patient with autism - Use muffler on handpiece to reduce sound, give instructions one at a time, slowly - patient is Noise sensitive. 49. What cyst in roots of mandibular premolar - Lateral periodontal cyst 50. What is the complication of maxillary molar extractions - Sinus perforation 51. What is easily curable ; hematoma or macule ? 52. Nadal obstruction what sedation you can't give - Nitrous oxide 53. Tto concusion - no need 54. Features of arrested caries – hard, eburnated , black brown 55. Lefort 3 - Separation from cranial base 56. Which surgery for open bite - Lefort 1 57. Serpentile feature - migratory glossitis 58. Which study FDA do to check drugs - Clinical trial 59. Patient said I don't wanna smoking , behavior shaping - Contemplation , when patient is ready to change negative behavior (smoking) , precontemplation when individual is not considering in changing this behavior 60. Cleft lip and palate at what month - As average for both, 6-8wks 61. Bucal limitation in mand denture – Masseter (buccal vestibule is buccinator and distobuccal is masseter) 62. Ideal test for kidney function - Protein creatinine ratio and blood urea nitrogen 63. Reversal acetilcolina – Physostigmine 64. Nerve affected when there is a damage on the uvula - option cnV, VII, IX, X, XII (vagus) 65. Ancillary freckling seen in – Nurofibromatosis 66. Radiographic is too light why - Underdeveloped, depleted delevoper,temp too cool 67. 3 mm lack of mand arch how you treat - Interdental striping 68. Smokeless tabacco cause what - Verrucous carcinoma 69. Lefort 1 include what structure - Pterygoid maxillary, Palate- greater palatine artery ecchymosis - guerins sign 70. Lefort 3 include what structure - Complete separation of midface at level Naso- orbital-ethmoid complex and zygomaticofrontal suture area. Fracture extends through orbits bilaterally. 71. Anug resemble what - Primary herptic gingivostomatitis 72. When do you do elective rct - Not enough crown structure 73. What % in community water fluoridation: 0,7-1,2 or 0,5 -1 74. Best bone for implant - D1 - mand anterior 75. Fearful patient how you do respond 76. There is a small white stain: amelogesesis imperfecta, for flour , dentinogenesis imperfect - 77. A little Pigment on the gingiva if for melanin, smoking a lot of question on day 2 of pigmentation and xerostomia 78. Patient of class 2 div 2 – picture 79. Picture of osteosarcoma - Sun ray pattern 80. Reversal benzodiazepines – Flumenazil 81. A lot question about flap don't remember the question 82. Hypides in pano 83. Best rx for interproximal caries - Bitewing , and digital is best 84. Q about protein morphogenetic - BMP- present in auto genous grafts - has progenitor cells -- help in forming new bone 85. Most común seizure: gran mal or febrile - Grand mal, febrile for kids 86. Which antibiotic work on gingiva? azithromycin 87. automated defibrilator, how does it work? a- monophase function b-2 shocks c- contraindicated below 12 year old d- discharge when needed 88. which bacteria causes elastenase, collagenase? gingivalis 89. 15 year old has fever, malaise, vesicles, lymphadenopathy? acute herpetic gingivostomatis 90. Necrotisizing sialometaplasia - Most common in palate, Resembles scc, usually occurs due to anesthesia with adrenaline, Minor salivary gland 91. Porosity in pfm? 92. Cancer of salivary glands and which one has perineurial invasion - adenoid cystic ca- swiss cheese; perineural invasion - Adenoid cystic carcinoma (ACC) is an uncommon form of malignant neoplasm that arises within secretory glands, most commonly the major and minor salivary glands of the head and neck. 93. Fracture also with paresthesia / angle of mandible, BODY 94. Xesrostomia and normal flow rate 95. osteoradionecorosis/hyperbaric oxygen 96. Lateral periodontal abscess is best differentiated from the acute apical abscess by = pulp test 97. What will not regenerate after rct = dentin formation 98. 3 partner dentist and hygienist hurts patient who involved in a low suit = only dentist supervising dentist and hygienist 99. How long after extraction you insert the complete denture = 8 weeks 100. Which one is more affect in male = hemophilia 101. Where does the epithelial for a graft come from = donor connective tissue 102. Macroglossia not found in: A)hypothyroidism B)hyperparathiroidism* 103. Radiographic image of (painless lesion, bone expansion) A)fibrous displasia (there was ground glass appearance to identify)* 104. Most supernumerary located in the: ant max.* 105. Cleidocraniodysplasia - supernumerary* 106. Plasma cell disorders multiple myeloma* 107. Which situation least require insulin A)trauma* 108. Herpangina - enterovirus* (coxsackie A) 109. Material least resistant - high leucite* 110. Which anethetic without vasoconstrictor is best to use - mepivacaine* 111. Do not use in myasthenia gravis A)erythromicine B)peniciline C) imipinem* 112. Broadest spectrum antibiotics - pen g procaine* 113. most common found A)odontoma* 114. Smoothest cutting but not efficient: carbide bur 115. Nitrous oxide side effect: nausea 116. Pt with many sinus fistula: actinomycosis 117. Wheezing during expiration: asthma 118. Where do u prefer GA? Ext of 2 yrs old 119. Pt came with abscess. What u will do first? Incision and drainage 120. Anterior guidance: both horizontal and vertical 121. Wear facet in primary dentition why? Don't remember options (one was habit) 122. Garre osteomyelitis- onion skin 123. Chronic osteomyelitis- moth eaten 124. Scleroderma- purse string mouth + extrusing teeth 125. Von recklinghausen- Cafe au lait spots; lisch spots on iris; crowe sign= axillary freckles 126. Fibrous dysplasia- ground glass 127. Pagets dis- cotton wool 128. Cherubism- bilateral; soap bubble ; perivascular cuffing 129. Multiple myeloma- punched out 130. How many percent have access to fl community water? 80 is the ans 131. MOA of sulfanamid - inhibit PABA required for folic acid synthesis - inhibit folic acid synthesis by inhibiting dyhydrofolate reductase by competeting with PABA 132. MAO of sulfonylurea - Stimulates insulin production from beta cells and increase sensitivity to insulin 133. Which one is class 2? Percocet is the ans 134. INR - extrinsic pathway ans 135. Which is correct about conjugation? adding a molecule to the drug ans 136. Which of these cognitive behavior decrease in a normal process of aging? learning I think is answ

Today's Rq's 1. What happens when you change from 8’’ to 16’’? 4 times 2. Stages of AIDS and no. of Leukocyte count.

3. Treacher Collin Syndrome - Treacher Collins syndrome, autosomal dominant syndrome caused by mutation of TCOF1. Coloboma is part of a set of characteristic facies that features craniofacial malformations, such as downslanting eyes, ear anomalies, or hypoplasia of zigomatic bone and jaw (micrognathia). Defect in all 1st pharyngeal pouch derivatives, Malformed ear, mandibular hypoplasia, hypoplasia zygomatic bone. 4. Ectodermal dysplasia - Lack of sweat glands, oligodontia or anodontia, concave nasal bridge. X-linked recessive condition that results in partial or complete anodontia. 2. Patients also have hypoplasia of other ectodermal structures, including hair, sweat glands, and nails. 5. What is the key factor for a denture to be successful? fabrication design, stability 6. 4 yrs old child – fluoride supplement? 0.25 mg if water fluoride concentration is 0.3-0.6ppm and 0.50mg if water fluoride concentration is less than 0.3ppm 7. Infection stage of syphilis? - Secondary stage 8. Optimum treatment for palatal papillary hyperplasia? a) Radical dissection b) Electrosurgery 9. 3 canals- which premolar? 1st PM max 10. Triangular canal central incisor- why? To expose pulp horns 11. Angles Class II- chances of trauma the most - True class II div 1 12. Most broad spectrum antibiotic? Tetracycline and chloramphenicol 13. Angioedema not caused by which drug? Ace inhibitors, angiotensin receptor blockers, nsaids, penicillins and quinolones CAUSE angioedema. So the answer would be any other than these classes 14. Dens in dente: Mostly in maxillary laterals 15. SLOB rule - Same lingual, Opposite buccal 16. Most difficult to floss where? Mesial aspect of max PM1 17. Dental clicking- increased VDO, also if porcelain teeth are used in dentures. 18. Function of rest. - provide vertical support for RPD and resist vertical forces for occlusion 19. Question on function of axial guidance - Axial guidance can be done using guiding planes in the surveyor used for single path of insertion 20. Horizontal axis of condyle. - rotation movement of mandible 21. As teeth ages, what increases? Hue / chroma - Hue unchanged, chroma increases and value decreases. 22. Patient wakes up in the morning with sore pain in muscles? Bruxism/Sinusitis 23. Difference between 245 and 330 bur. - Length of them 245=3mm / 330=1.5mm - 245 is longer 24. Prostaglandin analogue? Lantanoprost, treatment for glaucoma 25. Aspirin and Yinseng? What happens if given together – no, bleeding induces 26. Filtration used in X-ray machines - Lead is used for "collimation". Aluminium for "filtration" 27. Thompton effect? It must be Compton effect which is increase in wavelength of x rays or gamma rays. 28. Flouride water level? 0.7-1.2ppm 29. Inner and outer line angles. 30. Angioedema - Acquired angioedema is a specific type of allergic reaction. a. Precipitated by drugs or food (shell sh, nuts). b. Mediated by mast cell release of IgE. c. Results in characteristic soft, diffuse swelling of , neck, or face. d.Hereditary angioedema is a rare form that is an autosomal dominant trait. Acquired angioedema is a rapidly developing allergic reaction that results in characteristic nonerythematous swelling of lips, face, and neck. 31. Diphenhydramine- anti motion sicknes, Benedryl even given when pt is allergic to both amide and esters, Anti histamine , anti cholinergic , sedation 32. Difference between snuff dipper and nicotina stomatitis - nicotinic stomatitis lesion seen on palate due to smoking. Snuf dipper lesion seen on buccal side and it is mainly due to smokeless tobacco. Snuff--- dysplastic , premalignant Nicotinic --- non malignant 33. Pt. sitting with folded arms and legs shaking. What do you say to him? What brought you here 34. Most commom reason for pt. to have a negative attitude for dental treatment.a) Own dental experience b) Heard from peer/friends 35. Clinical test for Rheumatoid arthritis? Clinical test - watch toes and fingers - they become crooked --- something like that --- skeletal deformity. 36. Sjogrens syndrome is associated with? Lymphoma? True 37. Reasons for eruption failure. - Ankylosis,primary eruption failure, impaction, down syndrome etc 38. Critical pH for demineralization of dentin? 5.5 (enamel 5.5- dentin 6.2) 39. Situtaions for temporary restoration. - incomplete root.,,deep caries approaching pulp 40.Reciprocal anchorage. - Elastics to close diastema 41. Diabetes in children can lead to? Blindness 42. B.P. 160/140. Pulse 90. What do you do? Repeat after 15 mins/ call the physician immediately 43. What happens when you increase water in gypsum. - Decrease strength increase setting time, decrease expansion increase porosity. 44. LAP t/t: Tetracycline 250mg thrice for 14 days ---- severe cases--- metronidazole and penicillin, Sc/Rp and antibiotics 45. Area of least attached gingival? Which tooth - Mand 1st pm 46. How to check clinically for the prognosis of PDL attachment. – Probing 47. Pt. with alcohol abuse? What to check in lab report before extraction? Blood alcohol level – INR 48. Pain on eating sweet- reversible pulpitis 49. Ossifying fibroma - A. Ossifying fibroma. 1. Common fibro-osseous lesion. Can be considered similar or identical to cementifying broma, although some may reach considerable size. Clinical features. Radiographically appears as either a well- circumscribed lucency or a lucency with opaque foci. Seen in adults and young adults, typically in the body of the mandible. A variant known as juvenile ossifying fibroma occurs in younger patients and may exhibit an aggressive course. Microscopically composed of fibroblastic stroma in which new bony islands or trabeculae are formed. 5. Treatment—curettage or excision; recurrences rare. 50. Pt receive blow to eye orbital floor less common t/f 51. In Facebow transfers relation of arches? - In centric 52. Epinephrine action on which receptor in epinephrine reversal? Epinephrine has effect on both alpha and beta.....but epinephrine reversal is due to beta receptor stimulation. - Epinephrine action on Beta receptor in presence of alpha blocker causes Epinephrine reversa 53. Initiator in heat cured resin – light cured = diketone photoinitiator (camphoroquinone), self-cure = benzoyl peroxide. 54. Cause of brown discoloration of gingival margin of resin restoration – microleakage 55. Skewed distribution - Negative and positive.. bimodal distribution 56. Cimentidine function - Cimetidine is a drug that blocks the production of acid by acid-producing cells in the stomach. 57. Topical antibiotic in perio – doxicicline 58. Child with multiple caries - give GA 59. Least congenital missing tooth - Max canine 60. Gingseng contra with aspirin 61. Major connector function: rigidity nd stability 62. incisal position during record – Midline of incisal part of upper incisors 63. First pass metabolism - Enterohepatic circulation (ORAL DRUGS) 64. Other rq Polymerization shrinkage In composite depends upon? Adhesive bond Filler content Elastic modulus All - DD: Fillers are placed in dental composites to reduce shrinkage on curing. 65. Pulpitis 5 days after class ll composite is due to - Increase occlusal contact Microleakage Undercured composite 66. Which sound during wax try in – Sibilant – s and z 67. Least occurrence cyst - Lateral Periodontal Cyst 68. young female pt with vital tooth canine features with AOT 69. Ortho tipping before of veneering before? – before 70. Most imp feature of single implant – Antirotational 71. Drug bio- transformation - Excretion of drug..by passing through 72. White rough peduncalted lesion on palate? papilloma or fibroma 73. Primary and secondary stress bearing areas in maxilla - Primary - Residual alveolar ridge, Secondary – Rugae 74. Primary and secondary stress bearing areas in mandible - Primary Buccal shelf, Secondary – residual alveolar ridge 75. Best area for successful implant - ant mandible 76. Reason for image distortion 77. Reason for light radiograph - Low Kvp, Ma and exhausted developer and overfixing 78. Pedicle graft - excellent esthetic, less chance of failure, good vascularization and single recision. 79. Epi of free gingival graft – Donor epith 80. Gingivectomy indication - Gingival hyperplasia.. abnormal growth.. pseudopockets present, SUPRABONY POCKETS. 81. Gingivectomy contraindication - Infrabony defect 82. Clotrimazole which form torche 83. Topical antifungal - Nystatin and clotrimazole 84. Dentist did wrong treatment and didnt tell pt ... whats that term - violating Verasity 85. Disc movement first it moves 25 mm click is hear than 5 mm. so what side it goes in 5 mm - Ant to normal 86. Tooth size and morphology in which stage of tooth development - Bells stage 87. Mandibular denture position in relation to tongue – Below 88. Unbunding - dentist seperating treatment charges which actually can be counted as single procedure. 89. Lateral surface of tongue asymptomatic blue lesion in old pt since 5 yrs hemangioma or varicosities (because of age) 90. Lactating mother sedation drug – Promethazine 91. Fever in children drug of choice – Tylenol, acetaminophen 90. Nephrotoxicity by which drug - Aminoglycoside 91. Reduced insulin dose in what – IV sedation 92. Wheel chair transfer – Sliding 93. - in pemphigoid , pemphigus, and lichen planus also 94. Antibiotic and surgery is treatment for what lap or anug? Antibiotic-lap, Surgery- ANUG 95. Herpes peak age - 2-5 years 96. Difference between affected and infected dentin - infected always need to be removed. 97. Movement for recording buccal frenal area of mandible - Upward and outward 98. Opioid allergy – IRRITATION, Itching, flushing, and dec bp 99. Pain killer in renal disease – Oxycodone 100. Cocaine – mydraisis 101. BDZ antagonist – Flumazenil 102. Neurofibromatosis - Cafe au lauts spots, axillary freckling and lisch nodules 103. Direct retainer uses? Retention.

TANYA’s RQs (September 20, 2017) 1. Two questions about mandibular plane angle 2. Mandibular plane angle in class 3- steep or flat (cl3 flat cl2 steep) 3. A steep mandibular plane angle predisponate a person to --- and a flat mandibular plane angle predisponate a person to ---? long face, short face 4. Class 2 malocclusion A- increase mandibular plane angle B- decrease mandibular plane angle 5. Which of the following correlate with a steep mandibular plane? Select all that apply 1. Long ant facial vertical dimension 2.greater maxillary- mandibular plane angle 3. Ant open bite 4. All 6. Anterior incisal guide table 7. Reason for Incisive guide table?. Anterior guidance 8. Purpose of incisal guidance, mount casts..? adjust condylar guidance .. begin prep 9. Least strength of ceramic crowns - options - pressed leucite ceramic (strong porcelain is glass infiltrated alumina.) 10. Most important cause of ceramic crown fracture is due to - options were all lab procedures like firing, condensation - Inadequate framework/insufficient reduction of tooth, Yes low temperature in degassing, will affect the bond and more prone to fracture card 141 pros. Insufficient temperature or insufficient time 11. Floss and brushing dexterity 12. Patient has problem with manual dexterity what will he have problem with? A) floss B) Brushing C) both - why not both?? because the pt can use an electric toothbrush 13. Perforation of root canal which is difficult 3mm apical to gingival sulcus (CORONAL THIRD), 3mm coronal to gingival sulcus, apical 3rd,middle 3rd? 14. Behavior studies by which author? Freud - Freud theory is that the unconscious mind governs behavior to a greater degree than people suspect. Indeed, the goal of psychoanalysis is to make the unconscious conscious 15. Few questions about anxiety 16. Pt try to hide fear and anxiety, what we look for to understand it: Micro expression 17. More difficult to handle: Fear - Anxiety 18. Elderly person physically abused whom should u report? Human Health services 19. Kid abuse whom should u report? Social service – In pt management of dental decks, it says it depends by the state - "Once abuse is suspected against a child, elderly, or disabled pt, it must be reported to the appropriate agency (of the state) 20. Border molding impression compound and one question was about its thermoplastic property. 21. What is the primary indicator of accuracy of a border molding on a custom impression tray? stability, retention - Satbility and lack of displacement in card 1 pros , so i think both if in one option 22. Ability of impression compound to be adjusted due to which property? Thermoplastic 23. Polyether imp comp - sticks to teeth 24. Which of the following imp material will distorts the least when it store for 24 hrs before puring: silicon imp material, polyether imp material - pVS/ Silicon, its most dimensionaly stable. 25. Cheek biting in complete denture caused by – Absence of buccal horizontal overlap 26. Null hypothesis - Null hypothesis tested in chi test - T 27. P value in null hypothesis above 0.5 means is: statistical insignificant case control study - If the observed probability is less than or equal to .05 (5%), the null hypothesis is rejected (i.e., the observed outcome is judged to be incompatible with the notion of “no difference” or “no effect”), and the alternative hypothesis is adopted. In this case, the results are said to be “statistically significant.” If the observed probability is greater than 0.05 (5%), the decision is to accept the null hypothesis, and the results are called “not statistically significant” or simply NS—the notation o en used in tables. 28. Purpose of statistical tests in clinical trials is: A) to establish validity B) to find alpha value C) to reject null hypothesis D) to test specificity of experiment 29. Randomised control trial 30. Which if the following has problem in recall bias:- a. Cohort study b. Cross sectional study c. Randomised control study d. Case control study 31. WHich case study used in rare(Odd) disease investigation?? Case cOntrOl 32. Apically positioned flap 33. The most predictable for treatment of perio disease?1. Scaling & root planing 2. The modified Widman flap 3. Free gingival flap 4. Apically positioned flap 34. Whats the problem if you wanna perform apically positioned flap surgery in th mand second and third molar? external obligue bridge 35. Curette angulation 36. If correct angulation is present when working with a universal curette then ...? the lower shank is parallel to the tooth surface 37. Three q's about perio maintanence therapy steps and prognosis was so confusing 38. Anug 39. Antibiotic for ANUG ? Pencillin V (DD) 40. ANUG mouth rinse? Chlorohexiden, Hydrogen peroxide (DD) 41. Anug is usually accompanied by metallic taste...T or F 42. Squamous cellular carcinoma initial lesion picture 43. Pierre robin syndrome features 44. Patients with pierre-robin syndrome will have? mandibular retrogranthia - cleft palate, mandiblar retrognathia ,glossoptosis 45. Pierre Robin syndrome is associated with 1. Glossoptosis 2. Cleft palate 3. Respiratory problems 4.all of the above 5. A and B 46. Which disease does not cause cancer: HIV, HPV, I don’t remember the other options 47. Allergy a lidocaine anesthetic 48. Which pair of anesthetics is most likely to cause cross allergy? 1. Lidocaine and mepivocaine = T 49. Chroma 50. With age what increase hue value chroma!! - Chroma increases, value dec and hue unchang 51. Hue What should be selected first when picking a shade? hue (DD) - First pick HUE and VALUE is considered THE MOST IMPORT in shade selection. 52. Value 53. Erosion due to acidic foods and drinks is usually found on the cervical ______portion of the tooth - facial 54. Facial portions = 3 VER-TI-CAL, 5 HORIZONTAL 55. Free space 56. Free way space? 2 – 3 mm 57. Free way space when at physical rest T/F 58. Open bite or increased free way space = decreased bitting force T/F 59. Phonetics 60. When do you check phonetics for a CD? Wax try in, Tooth try in – DD#20 PROST 61. During try-in of CD, which phonetics is used: Labio-dental, Linguo-dental, Linguo- alveolar, B m p sounds 62. Radioresistant cells – NERVE, MUSCLE 63. Radiosensitive cells - Bone marrow lymphocytes, basal epithelial, small lymphocyte 64. Which are the most radiosensitive cells in the human body? Basal epithelial cell, Endothelial cell, Salivary gland acinar cell, Nerve cell, Erythrocyte 65. Radiographic errors 66. Most important characteristic for choosing major connector- Stability & rigidity 67. Histogram shows- Variance – It is an estimate of the probability distribution of a continuous variable (quantitative variable) 68. Hazard communication standard was prepared by- OSHA 69. Pedal edema, dyspnea, orthopnea are signs of- CHF 70. A dentist cannot do what to correct the small discrepancies- manipulate the cement- water ratio. 71. Dexterity comes by what age? 6-8, mean 7yrs 72. Dexterity comes by what age? A.3-4 years B.1-2 years C.5-6 years D.7-8 years 73. Most common complication of N2O - bronchospasm was what I marked (nausea, vomiting and peripheral neuropathy was not in option) 74. Most complication of nitrous oxide: vomiting, behavioral problem, (nausea was not an option) – DD: the most common complication associated with nitrous oxide sedation is a behavioral problem (laughing, giddy). Most common adverse effect: nausea and vomiting. Headache and disorientation can be avoided by administrating 100% oxygen after nitrous oxide has been discontinued. 75. Best bone for implant- d1, d2, d3, d4 76. Question on sensitivity, specificity, (definition) 77. A diagnostic test that correctly identifies 20% of screened patients as being dx free has low: Sensitivity, Specificity, Precision – sensitivity is defined as the percent of persons with the disease who are correctly classified as having the disease (those who have the disease). Specificity is defined as the percent of persons without the disease who are correctly classified as not having the disease (those who do not have the disease). 78. What does identify persons with the disease? Sensitivity, Specificity 79. What causes varices on the tongue? Age 80. Lady presents with blue swelling under tongue? I put ranula 81. Picture said: “erythematous, bleeding swelling” mandibular swelling right next to premolars on R side? I put pyogenic granuloma - T 82. Patient with diabetes which finding is not consistent? increase collagenase in crevicular fluid, increase glucose in crevicular fluid, increase gram negative in crevicular fluid, decrease in thickness of basilar lamina of blood vessels in periodontium. 83. Most common reason for failure of dental amalgam? moisture contamination, improper prep design- not enough depth, improper titrutration, improper condensation - In card 7 operative said most coz failure is moisture, and improper preparation and not enough depth is reason o fracture, card 120 oprative 84. Crouzons 85. Pt has cranial dysostosis, midface deficiency, hypertelorism and beaten metal appearnce? a. Crouzons b. Treacher Collins 86. Automatic defibrillator 87. Automated defibrillator expander is used in: cardiac arrest, both kids and adults. 88. Papoose Board 89. 8 years old parent with negative behavior, you use for immobilize extremities: Papoose board 90. Most convergence in which primary tooth – mandibular 1st molar 91. Least antiplatelet NSAID - celecoxib 92. 4 to 5 questions on implant you should know all implant distance - Distal between 2 implants 3 mm, From IAN 2 mm, From mental 5 mm, Tooth and implants 1.5 93. A lot about endodontics some were very basic questions 94. Plaque ph - so least than 5.5 95. Distolingual what is the best filling material - amalgam (DD) 96. Conical shape what caries? 97. The conical shape of pits and fissures caries shows: A. Two triangles pointing toward the pulp B. Two triangles pointing toward the occlusal surface C. Two triangles with their bases on the DEJ D. Two triangles with their apices on the DEJ 98. Transillumination 99. Which will show up on transillumination best? Craze line 100. Cracked tooth 101. Patient comes back few months after RCT & Crown with pain upon biting, what happened...cracked tooth 102. Crack tooth syndrome is most likely found? Mandibular 1st molar 103. Neuropraxia: This type of injury is characterized by a segmental block of the conduction of the nerve action potential caused by structural damage to the myelin sheath. The axon and connective tissues including endoneurium, perineurium, and epineurium are not affected. 104. Neuropraxia: involves both perineurium and epineurium, only perineurium, only epineurium, none of the above 105. Neuropraxia reversible or irreversible? Reversible 106. Which salivary gland tumors is associated with neuropraxia (nerve damage): A.Mucoepidemoid carcinoma B. adenoid cystic carcinoma(ACC) C. - adenoid cystic ca has tendency to invade surrounding nerve fibers-- perineural invasion 107. Which of the following glucocorticoids is the strongest? Dexa, Corticoid, Hydrocortisone 108. Disinfectant for dental chair (min) cavicide? 109. Le fort 1 fracture à Maxillary sinus 110. Maxillary sinus fracture mostly with: Le fort I 111. Collimation does everything except –reduce pt exposure, reduce operator exposure, film fog, reduce average energy of x-rays 112. The Dentist completes exam and advises x-rays but the pt refuses. What should the dentist immediately do? When a patient refuses to have dental radiographs, the dentist must decide whether diagnosis and treatment can take place without the recommended radiographs. No document can be signed by the patient that releases the dentist from liability. 113. What is true of osteoradionecrosis. 114. Damage to the blood vessels as oppose to (nerve, muscles) predisposes a patient to develop osteoradionecrosis. true or false 115. Osteoradionecrosis can be differentiated from osteomyelitis as: a. Osteoradionecrosis show periosteal reaction b. Osteoradionecrosis does not show periosteal reaction c. Cannot be differentiated on basis of periosteal reaction as both show it d. Cannot be differentiated on basis of periosteal reaction, as both do not show it. 116. X-ray identification: - Median Palatal Suture, Zygomatic process of maxilla, Dorsal Surface of tongue, Odontoma – RL, RO, RL, RO 117. The severity of response increases with the amount of X-ray exposure. This effect is called. Deterministic, Stochastic, Genetic 118. Most common recurrent cyst : okc 119. xray for annuerysm cyst - CT or MRI 120. Secondary herpes site - Genitals is HSV 2 for HSV 1 its lips 121. Concentration of apf flouride gel - 1.23% - Since the early 1960s, acidulated phosphate fluoride (APF) has become the most widely used fluoride compound for professional application. APF has a pH of about 3.0 and was developed after experimental work showed that the topical uptake of fluoride by enamel was greater in an acidic environment. The agent has been tested in several concentrations, the most common being 1.23% fluoride, usually as NaF, in orthophosphoric acid. 122. Orange stain is important to change chroma or hue? hue 123. Cleft palate prevalence: Palate 1:2000 – The incidence of cleft lip with or without cleft palate occurs in 1 in 700 – 1000 births (DD), together 1:700 (most prevalence) 124. Retained primary teeth is characteristic for both cleidocranial dysplasia and ectodermal dysplasia - T/F 125. Multiple supernumerary teeth are most commonly found in A. cherubism. B. cretinism. C. hypothyroidism. D. cleidocranial dysplasia 126. Cleidocranial dysplasia = supernumeraries 127. Hypothyroidism clinical signs - Cold to touch, thinning of hair, delayed eruption of permanent 128. Acromegaly clinical signs - Mand 129. What is called when u do not take patient consent ?? assault , nonmaleffi, battery 130. Which bur is used for the crown cutting? 169 131. Which forceps is used for mandibular premolar extraction?? 151 132. Behcets syndrome is associated with ? aphthous ulcers 133. I got three ortho cases, one squamous cell carcinoma 134. Altered cast 135. Altered cast technique? Support 136. Gracey’s curette no. 13 and 14 used for? Distal surface – 1-4 ant, 5-6 ant and pm, 7-8 post, 11-12 post mesial, 13-14 post dista. 137. After a gingivectomy how does the site heal? a. from the epithelium of the pockets b. epithelium of the adjacent alveolar mucosa c. endothelium of the blood vessels d. primary intention 138. The gingivectomy approach to pocket elimination results in A. healing by primary intention. B. adequate access to correct irregular osseous contours. C. retention of all or most of the attached gingiva. D. None of the above. 139. pic 140. 2 questions were on gingivectomy 141. Gingivectomy is used to remove suprabony perio abscess T/F 142. Gingivectomy is used to remove suprabony pocket T/F 143. Pt. have denture, after 5 years he complains of ulcer and inflammation in lower buccal vestibule. What is the diagnosis: 1/Hypertrophic frenum. 2/ Epulis fissuratum 144. Epulis fissuratum should not be removed before the fabrication of new denture T/F 145. What is the cause of epulis fissuratum a. unstable denture b. under extention c. over extention d. traumatic occlusion – DD: The cleft-like kesions of epulis fissuratum result primarily from overextension of denture flanges. The overextension may result from long-term neglected or settling subsequent to residual ridge resorption. Traumatic occlusion of natural teeth opposing an artificial denture may also cause this condition. 146. Denture stomatitis: localized or generalized chronic inflammation of the denture bearing mucosa. Clinically, there is redness and a burning sensation. There may be or may not be discomfort. Trauma and secondary fungal infection appear to be the most likely cause of denture stomatitis. Treatment: 1. Improved oral hygiene 2. Tissue rest 3. Antifungal therapy (nystatin) 4. Resilient conditioners 5. New, well-fitting dentures.

HOW TO DIFFERENTIATE ENDODONTIC AND PERIODONTAL ABSCESS:

DRY SOCKET: Thought to develop because of increased fibrinolytic activity causing accelerated lysis of the blood clot. It is most common following extraction of the mandibular molars. Smoking, premature mouth rinsing, hot liquids, surgical trauma, and oral contraceptives all have been implicated in the development of a dry socket. Careful technique and minimal trauma reduce the frequency of patients developing dry socket. The patient develops severe, dull, throbbing pain 2 to 4 days after a tooth extraction. The pain is often excruciating, may radiate to the ear, and is not relieved by oral analgesics. TREATMENT OF DRY SOCKET: - Flush out debris with slightly warmed saline solution – gently - Place a sedative dressing in socket (eugenol). The dressing should be removed within 48 hours and replaced until the patient becomes asymptomatic. (1. The gauze provides an attachment for the obtundent paste so it stays in the socket 2. Eugenol is the active component in most sedative dressings) - Nonsteroidal anti-inflamatory analgesics should be prescribed if necessary / ANTIBIOTICS ARE GENERALLY NOT INDICATED.

Dry socket is the most common complication seen after the surgical removal of a MANDIBULAR MOLAR. Curetting a dry socket can cause the condition to worsen because healing will be further delayed, any natural healing already taking place will be destroyed, and there is a risk of causing the localized inflammatory process to be spread to the adjacent sound bone. Can occur in 3%of mandibular third molar extractions. Will heal with irrigation and local treatment for pain control.

FLUORIDE - [ SOME IMPORTANT FLUORIDE DOSAGE FACTS ] - 1. Estimated Toxic Dose--> 5 to 10 mg/kg - 2. Estimated Lethal Dose (Adults)--> 2.5 to 5.0 gm (F- alone) & 5 to 10 gm (for F- in NaF) - 3. Estimated Lethal dose (kids)--> 500mg (for <3yrs) & - 16mg/kg (for >3yrs) - 4. Water fluoridation--> 0.7 ppm [ADA] - 5. Water Fluoridation range--> 0.7 to 1.2 ppm [ADA] - 6. Skeletal fluorosis starts at--> 3 ppm (chronic use of F-) - 7. starts at--> 1ppm (chronic use of F-)

ANESTHETICS: - Too much anesthetic in the bloodstream can cause toxicities to the CNS and cardiovascular system. - Toxicity: The CNS effects include restlessness, stimulation, tremors, convulsive seizures followed by CNS depression, slowed respiration even coma. The cardiovascular effects include bradycardia and reduction of cardiac output. - Ester local anesthetic allergic manifestations include nasolabial swelling, itching, and oral mucosal swelling. - LA have NO effect on potassium at the nerve axon

LIDOCAINE TOXICITY AMALGAM 1. Primary retention form—retention form preparation features lock or retain the restorative material in the tooth: 1) Mechanical locking of the inserted amalgam into

surface irregularities of the preparation (even though the desired texture of the preparation walls is smooth) to allow good adaptation of the amalgam to the tooth. (2) Preparation of vertical walls (especially facial and lingual walls) that converge occlusally. (3) Special retention features, such as locks, grooves, coves, slots, pins, steps, or amalgam pins, that are placed during the final stage of tooth preparation.

2. Primary resistance form—resistance form preparation features help the restoration and tooth resist fracturing as a result of occlusal forces. (1) Resistance features that assist in preventing the tooth from fracturing. (a) Maintaining as much unprepared tooth structure as possible (preserving cusps and marginal ridges). (b) Having pulpal and gingival walls prepared perpendicular to occlusal forces, when possible. (c) Having rounded internal preparation angles. (d) Removing unsupported or weakened tooth structure. (e) Placing pins into the tooth as part of the final stage of tooth preparation (note: this strategy is considered a secondary resistance form feature). (2) Resistance form features that assist in preventing the amalgam from fracturing. (a) Adequate thickness of amalgam (1.5 to 2 mm in areas of occlusal contact and 0.75 mm in axial areas). (b) Marginal amalgam of 90 degrees or greater. (c) Boxlike preparation form, which provides uniform amalgam thickness. (d) Rounded axiopulpal line angles in class II tooth preparations.

MERCURY TOXICITY Excessive saliva is a prominent toxic effect of mercury. The presence of mercury in the body is determined by a urine test. Treatment may include gastric lavage with milk and egg white or sodium bicarbonate, chelation with British anti-lewisite (BAL), and fluid therapy. Note: British Anti-Lewisite (BAL) or Dimercaprol and penicillamine are two drugs currently marketed for promoting the excretion of mercury, lead, and several other agents. Mercury that is absorbed into the circulatory system may be deposited in any tissue. Higher-than average accumulations occur in the BRAIN, LIVER and KIDNEY. Mercury does not collect irreversibly in human tissues. There is an average half-life of 55 days for transport through the body to the point of excretion. Thu mercury that came into the body years ago is no longer present in the body.

SEQUENCE OF TEETH EXTRACTION Maxilla before mandible Distal before mesial Start with third molar, second molar, second premolar, first premolar, lateral, central. Finally, you extract the first molar then the canine as last teeth to be extracted. The two teeth that are the most difficult to remove, the first molar and canine, should be extracted last. Removal of the teeth on either side weakens the bony socket on the mesial and distal side of these teeth, and their subsequent extraction is made more straightforward.

Sham Sham's RQs

1) Distance between implants? 3mm 2) Hemophilia test – PTT 3) Mouthwash in children: NaF, Chlorhex (sodium fluoride used when there are caries) 4) Mouthwash given to a comprimsed child – Listerine, Chlorhex (there is over the counter listerene for kids without alcohol - both NaF, if for disability NaF not there then listerine) 5) Benzodiazepine moa - potentiate GABA, not inhibit. Potentiate the action of GABA – Mosby: enhance the effect of y-aminobutyric acid (GABA) at GABAA receptors on chloride channels; this increases chloride channel conductance in the brain (GABAA receptors are ion channel receptors). Benzodiazepines produce their calming effects by DEPRESSING THE LIMBIC SYSTEM & RETICULAR FORMATION through potentiation of the central inhibitor neurotransmitter (neurons) gamma-amino-butyric acid (GABA). Tolerance and physical dependence can occur with prolonged high dosage, but they are much safer than barbiturates. 6) Abfraction – Mosby: Abfraction is tooth loss in the cervical area caused by biomechanical loading. Occlusal loading resulting in tooth flexure, mechanical microfractures, and tooth substance loss in the cervical area; may appear similar to erosion. 7) Erosion - Mosby: Erosion (sometimes called corrosion)—usually in the cervical area of facial surface of tooth; may be caused by acid beverages or citrus fruits. Erosion is wear secondary to chemical presence. 8) Bucolingual space needed for 4mm implant- 8mm? 7mm was not given – 2mm on bucal and 2 mm on lingual. So, for a 4 mm diameter implant, I need 4.0+1.9 + 1.9 = 7.8 mm. – Carranza: Assuming an implant is 4 mm in diameter and 10 mm long, the minimal width of the jawbone needs to be 6 to 7 mm, and the minimal height should be 10 mm (minimum of 12 mm in the posterior mandible, where an additional margin of safety is required over the mandibular nerve). This dimension is desired to maintain at least 1.0 to 1.5 mm of bone around all surfaces of the implant after preparation and placement. (BL 6mm - MD 7 mm) 9) Alplrazolam: BDZ, sedative/antianxiety, medium duration of action benzodiazepine, it Inc the frequency of opening of chloride channel in GABA receptors in the brain. It Inc the effect of Gaba at Gaba 1. 10) Biphosphonate, there is difference between osteoradiorecrosis (with radiation therapy) and osteonecrosis with bisphosphonate. Bronj (bispohophonate related osteonecrosis of jaw) 11) Hip joint- premedicate or not - No need it. Dentin 12) 14 mm crowding- ortho and surgery, ortho only, surgery only - Ext and ortho should b enough 13) Warfarin- wat test – INR 14) Patient smokes, is implant contraindicated? No 15) 5 year old child not cooperative, wat to do? conscious sedation? voice control? General anesthesia? 16) Most common problem in clinic? Syncope 17) Mech of action. of fluoride - Replacing of HA with hydroxy fluoroaptite 18) Precations for hep patient? Standard precaution, no other precaution necessary 19) How to treat a patient. give an order ex: perio, endo, operative? ortho last - endo perio operative ortho 20) OP PIC- osteosarcoma or ossifying fibroma? Ossifying fibroma: radiographically appears as either a well-circumscribed lucency or a lucency with opaque foci. Osteosarcoma sunburst/ossifying fibroma wellcircumscribed rl/ro 21) Currete used for mesial surface of tooth – 11-12

22) Forcep not used for extraction of root - not 23. 23 is cow horn. 23) Commonly contraindication for surgery recently? Bisphosphonates 24) Primary molar differs from permanent in cervical constriction – T 25) Most common organism in implant failure; same as regular dentition 26) Dislodged clot; sedative dressing 27) Treatment of ANUG: Debridement, H2O2/ chlorhexidine rinses, antibiotics if systemic symptoms (fever, lymphadenopathy). If it is systemic Metronidazole, amoxicillin. 28) How to treat a patient with implant supoorted dentures: fluouride - yes Fl for abutment in overdenture, need options to decide the answer. Not for implant. 29) Epinephrine mech of action - epi stimulates both alpha 1,2 and beta 1,2 receptors. Epinephrine is used to treat anaphylactic shock. It stimulates alpha 1-adrenergic and 2-adrenergic, beta 1- adrenergic and 2-adrenergic receptors. B2 receptor stimulation aids in relieving bronchospasms. 30) Battle sign - Battle's sign, also mastoid ecchymosis, is an indication of fracture of middle cranial fossa of the skull, and may suggest underlying brain trauma. Battle's sign consists of bruising over the mastoid process, as a result of extravasation of blood along the path of the posterior auricular artery.

31) Pejrre robin sy drome - Retrognathia, glossoptosis, cleft palate. Google: Pierre Robin sequence is a set of abnormalities affecting the head and face, consisting of a small lower jaw (micrognathia), a tongue that is placed further back than normal (glossoptosis), and blockage (obstruction) of the airways. The three main features are cleft palate, retrognathia (abnormal positioning of the jaw or mandible) and glossoptosis (airway obstruction caused by backwards displacement of the tongue base). 32) Gingivectomy icsion - external bevel. Its excision starts from apical of pocket (JE), but coronal to mucogingival junction, secondary healing. 33) Cauliflower shaped lesion - Verrucous carcinoma, condyloma accuminata, papilloma. 34) Asymptomatic periodontitis, long- standing, asymptomatic or mildly symptomatic lesion. It is usually accompanied by radiographically visible apical bone resorption. Bacteria and their endotoxins cascading out into the apical region from a necrotic pulp cause extensive demineralization of cancellous and cortical bone. Occasionally, there may be slight tenderness to percussion or palpation testing. The diagnosis of asymptomatic apical periodontitis is confirmed by the following: (1) General absence of symptoms. (2) Radiographic presence of radiolucency. (3) Confirmation of pulpal necrosis. A totally necrotic pulp provides a safe harbor for the primarily anaerobic microorganisms—if there is no vascularity, there are no defense cells. Asymptomatic apical periodontitis traditionally has been classified histologically as apical granuloma or apical cyst. The only accurate way to distinguish them is by histopathologic examination. 35) Primary- cold or percusiion or ept 36) Permanent - ? thermal or ept - tricky question, EPT for perio and thermal for endo 37) Which perio conditions we need antibiotics – Localized Agressive periodontitis (ANUG only if systemic involvement). 38) Ranitidine - H2 antagonist, used to treat GERD 39) Methotrexate- Antimetabolite antineoplastic agent, anticancer drug with folic acid. 40) Diazepam - BDZ, sedative, anxyolitic, long duration of action. 41) Which ka most common epilelsy in children - Petit mal 42) Status epileptic medication - Diazepam 43) Maxillary sinus which view – Waters 44) Buccal frenum - Triangularis or buccinators 45) Pontics - Most aesthetic is Ovate most common is modified ridge 46) Succedaneous tooth - All permanent except 1st, 2nd and 3rd molar 47) Most common emergency in dental – Syncope 48) Pregnant women which nerves get suppressed - ? 49) Apexogenesis does what? maintenance of pulp vitality to allow continued development of the entire root. Apical closure occurs approximately 3 years after eruption. The key is to allow the body to make a stronger root. This procedure relates to teeth with retained viable pulp tissue in which the pulp tissue is protected, treated, or encouraged to permit the process of normal root lengthening, root wall thickening, and apical closure. Nonsurgical endodontic therapy can be performed more safely and effectively to treat the pulpal disease. Indications: (1) Immature tooth with incomplete root formation and with damaged coronal pulp and healthy radicular pulp. Contraindications: (1) Avulsed teeth. (2) Unrestorable teeth. (3) Teeth with severe horizontal fracture. (4) Necrotic teeth. Prognosis—good when pulp capping or shallow pulpotomy is done correctly; conventional pulpotomy is not as successful. Success rate depends on the following: (1) Extent of pulpal damage. (2) Restorability of the tooth 50) Apexification: not vital pulp therapy because the tooth is pulpless. 2. Definition— method to stimulate the formation of calcified tissue at the open apex of pulpless teeth. 3. Indication—infected teeth with open apices in which standard instrumentation techniques cannot create an apical stop to facilitate effective obturation of the canal. Technique—disinfection of canal followed by induction or placement of an acceptable apical barrier. a. Calcium hydroxide and MTA have been used to create an apical barrier. (1) Calcium hydroxide may be used to induce apical hard tissue formation. A thick paste of calcium hydroxide must be placed in the canal and replaced every 3 months until a hard tissue barrier forms, against which gutta-percha may be placed to fill the canal. This traditional technique may require 1 year for hard tissue formation. MTA can be packed into the apical 3 mm of the canal, and the remainder of the canal can be filled with gutta-percha at the same appointment. MTA has established biologic outcomes in terms of healing and root- end closure at least comparable to teeth treated with calcium hydroxide. Advantages of MTA compared with calcium hydroxide—treatment can be completed in less time, improved patient compliance, reduced cost of clinical time.

RQs

Q1- What will not set off an event in a child with sickle disease Trauma Cold Infection Nitrous oxide

Q2-What not to do in veneer: try in paste for shade apply silane to inner surface apply bonding agent etch enamel with hydroflouric acid

Q3-What is the purpose of making a record of protrusive relation and what function does it serve after it is made A. To register the condylar path and to adjust the inclination of the incisal guidance. B. To aid in determining the freeway space and to adjust the inclination of the incisal guidance. C. To register the condylar path and to adjust the condylar guides of the articulator so that they are equivalent to the condylar paths of the patient. D. To aid in establishing the occlusal vertical dimension and to adjust the condylar guides of the articulator so that they are equivalent to the condylar paths of the patient.

Q4-9 year old child with POOR oral hygiene needs ortho treatment, what do we do? A- no treatment B- removable appliances C- fix appliances D- semi fix appliances

Q5-You will let the patient to sign the informed consent when? a) after you discuss the treatment plan b) directly after the diagnosis c) after done with extraction procedure

Q6-Patient evaluated after perio therapy wht would u check Attachment loss Pocket depth Bleeding on probing Oral hygiene If q asking what the dentist will check on pt mouth to evalute success of treatment will be C, bleeding in probing: The best criterion to evaluate the success of scaling and root planning is NO EVIDENCE OF BLEEDING ON PROBING. Bleeding on probing indicates inflammation in the tissue. The amount of inflammation present is used to determine the effectiveness of periodontal instrumentation and home care by the patient. If q asking the rate of success depend on what ? So D, Patient oral hygiene – the best indicator of success of a periodontal flap procedure is postoperative maintenance and plaque control by the patient.

Q7-Researcher has set alpha at 0.05. Results showed p value 0.01 and researcher rejected null hypothesis. What kind of error is it? - They had set the value at 0.05 and result came out 0.01which is less than the set value so it's a null hypothesis and since they rejected it which means there was incorrect rejection which is type 1 error

Q8- Proscar (finestride) is used to treat? Benign prostatic hyperplasia Q9-Motion sickness medication – Scopolamine Q10-3 year old 5mm intruded - Observe, just 3 yrs not at all any damage to permanent. Q11- Most common respiratory emergency – Hyperventilation Q12-Upper first PM forceps - For maxillary premolars can be 150, 65, 286. For mandibular premolars 151 From 13-47 from other RQ Q48-Balanced occlusion use for what? Complete Denture Q49- Incisal guide table? There was no removing or lifting pin in the option - Incisal guide table: construction of a custom that preserved anterior guidance, from Mosby. Anterior guidance must be preserved by means of construction of a custom incisal guide table, especially when restorative procedures change the surfaces of anterior teeth that guide the mandible in excursive (lateral, protrusive) movements, from Mosby. Q50- Anterior guidance purpose? Protects posterior teeth in lateral function Q51-Longitudinal studies what factors are least important? Q52-Pka depends upon? Ionized and non ionized ions, Onset of action. Low pka--- more free ionized---faster onset Q53- Whats the purpose of posterior disocclusion? its christensen phenomena, to avoid non working interfrence which is destructive Q54- Finish margin of pfm? Pfm - chamfer/ shoulder Q56-Renal failure which pain killer? Tylenol, Oxycodone for hepatic. Q57-Wat are the reason for the crossbite? Retained primary tooth, construction of Max Q58- Lesions most commonly seen in jaw? PA lesions? Q59- Pyogenic granuloma by which bacteria? Pyogenic granulomas (PGs) are benign vascular lesions that occur most commonly on the acral skin of children. The term pyogenic granuloma is a misnomer. Originally, these lesions were thought to be caused by bacterial infection; however, the etiology has not been determined. Q60- Prevelance and incidence question? simple one - Prevalance: total number of cases incidence: New number of cases Q61-Palatal tori question? horshoe was there in the option, Yes horseshoe/ U shaped major connector used for RPD Q62-10% calcium hydroxide varnish best use where? Amalgam restoration, zinc phosphate, in the walls of preparation cavity, not cavosurface areas Q63-Conjugation does wat? covalent bonding with glucuronic acid to make water- soluble for excretion Q64-Trisomy 21? Down Syndrome Q67-Sibilant sound when to check? DD: phonetics are tried in at the time of the "wax try- in of the trial denture" - wax trial with teeth set Q68-Osteomylitis which bacteria? Staph.areus Q69-Facebow record? Maxilla&hinge axis - it transfers the relationship of maxillary arch and to the casts. It records the upper model's (maxilla) relationship to the External Acoustic Meatus, in the hinge axis. Q70-Lingual bar? Shaped like a half-pear tapered toward the tissue in the superior border and has its greater bulk at the inferior border. For a lingual bar, the depth of the vestibule should exceed 7 to 8 mm. This is the simplest and most commonly used major connector. – Mosby Major conector of RPD, indicated for lingual vestibule of 7mm minimum Q71-CD wat type of occlusion? Balanced Q72-what will have wavelength ? Hue Q73- Neutometesis - Severe nerve damage, neuromatosis- cause tumors on nerve, also called neurofibromatosis, both the nerve and the nerve sheath are disrupted Q74-median? Placing the number in value order and find the middle, or example: 1,1,1,2,2,4,6. the median is 2. nothing to do with average Q75- Palatal expansion does not need a labial bow why? Cause it has no effect on palate, not for expansion. Q77- Albuterol side effect? Xerostomia Q79- fluxetine action? Ssri - Prozac (fluoxetine) is an antidepressant belonging to the selective serotonin reuptake inhibitor (SSRI) class of drugs. Prozac is mainly used for the treatment of major depression, obsessive-compulsive disorder, and panic disorder Q81-best imaging for condyle: Reverse towne

1.Metalloceramic crown which type of bevel Chamfer 1mm (lingual?) Chamfer 1.5mm Shoulder 1mm Shoulder 1.5mm (labial) Metal ceramic restorations PFMs - A chamfer finish line and all margins should be placed supragingivally when possible. The necessary thickness of metal substructure is 0.5mm. The minimal porcelain thickness is 1- 1.5mm. Thus, the tooth reduction required for a PFM crown is -1.5-2.0mm. The labial shoulder width is ideally 1.5mm. The tooth preparation reduction for metal-ceramic restorations (1.5 to 2.0 mm) must provide space for metal (0.5 mm) and porcelain (1.0 to 1.5 mm).

2.Benadryl do what: Antitissive,h1 blocker,anesthetic, cause Xerostomia (anticholinergic, antihistamine, sedative). 3.Electrosurgery for..hemorrhagie and excess tissue, Side effect -- ging recession. 4.Where go item when swallowed by accident - Right bronchus 6.Demineralization of enamel A. ph of plague less 6.4 B.Dentin ph 6.4 C.Start at subsurface D. go like enamel rods 7.Junctional epithelium attached by Fibers Or hemidesmosom 8.Pinkham pre-cooperational child is - Lacking cooperative ability (the “pre-cooperative” patient) a. Very young children with whom communication cannot be established nor comprehension expectedb. Children with specific debilitating or handicapping conditions 9. Main reason of Osteoporosis US A.nutrition B.genes C.environment 10.Sedation is for- Anxious 11.Which is least has lichenoid eruption 1.erythem 2.amalgam tatto3.drug rn Amalgam produced lichenoid reaction. So answer is A 12.Water contamination with amalgam cause: Delayed expansion 13. AORS is mean??? Adjusted Odd Ratios 14.Reciprocal anchorage done by… Reciprocal – Type of anchorage when two units of teeth move towards each other with equal distance. For example in Diastema by using elastic bands. 15.Oxycodone hypotension and itching by – Itchin by Histamine release. Oxycontin (oxycodone hydrochloride) is an opioid drug used for the management of moderate to severe pain, usually for an extended time period. Oxycontin is not an "as needed for pain (PRN) drug. 16. Porcelain bevel types: Shoulder and chamfer, Chamfer and beveled shoulder, Chamfer nad chamfer Unlike the PFM restoration which accepts any marginal design (bevel, chamfer, shoulder), marginal tooth preparation for the ALL-ceramic crown or porcelain jacket crown MUST BE A SHOULDER. 17.Tx initial for large subgingival caries - Glass ionomer filling 18. Intial tx for local aggresive perio – Tetracycline 19 Which one is. Acidogenic bacteria - Strep mutants acidigenic , & aciduric 20.Most common odontogenic tumor Ameloblastoma Amelob fibroma Odontoma Adenomagoid tumor (ameloblastoma is most common epithelial odontogenic tumor) 21.Incisial and coronal parts to be matched in porcelain metal crowns done by: Internal coloring of porcelain External glazing Polishing and grinding to change light reflection Firing under high temp 22.Uncarious short premolar for retainer under fpd Crown Inlay Onlay ¾ 24.Ian lingually located,tube of xray moved inferiorly and beam sup, location of nerve: Mesial Distally Apically Occlusialy 25.Attachemen loss in furcation primary etiology - Oral biofilm 26. in which stage - Morpho -shape 27.Which one centrally acting muscle relaxant Dandtrolene, Diazepam 28.Stimulated salivary flow - 1ml per min 29.Has only md incisor.beat shape of occlusion... Balanced in centric Balanced in working Group function Canine function 30.Intial for caries: Bacteri carbohy susceptible toot, Bacteria polysaccharide enamel MOST COMMON epithelial odontogenic tumor in children – mucoepidermoid 31.Antabuse - Aldehide dehydrogen, antabuse inhibits the enzyme Aldeyhyde dehydrogenase, this is used to treat alchoholism. Its actually called acetaldehyde dehydrogenase 32.Beclometasone for asthma Short acting - its short acting, used only during attacks Less potent than cortisole 33.Fiber post - Elsatic as dentin 34.Pt try to hide fear and anxiety, what we look for to inderstand it - Micro expression 35.Wheel chair people what is true: Sliding transport the best Ask pt to remove catheters Ask pt do not buckle up Wheel chair transfer is mot appropriate 36.IG mechanism of action 37.Maslow theory - Theory of human motivation

1. how do you treat a. leave it alone b. excise c. give meds Treatment: consists of opening the lesion, curettage, and closure. It may contain blood, serosanguineous fluid, debris composed mainly of a blood clot, or may be completely devoid of solid material. 2. Which cyst is associate with anterior cyst w/ crown - dentigerous cyst (follicular cyst) or eruption cyst 3. Common cyst is lower anterior teeth and teeth is vital - periapical cemento dysplasia 4. Know different between and dentinogenesis imperfect AI: ectodermal - DI: mesenchymal 5. Which of the following represents the basic constituent of the most root canal sealer a. zinc oxide b. zinc stearate c. polyvinyl resin d. polycarboxylate e. zinc oxyphosphate 6. Aging of the pulp is evidenced by an increase in a. vascularly b. cellular elements c. fibrous elements d. pulp stones 7. In the normal dental pulp, which of the following histologic features is least likely to appear a. cell-free zone of weil b. palisade odontoblastic layer c. lymphocytes and plasma cells d. undifferentiated mesenchymal cells 8. the most commonly found salivary gland tumor is a. adenocystic carcinoma b. pleomorphic adenoma c. muco epidermoid carcinoma 9 the action f the Hawley appliance is mainly a. intrusion b. tipping c. bodily movement 10 a light force applied to the periodontal ligament during orthodontic treatment is considered a. intermittent b. direct c. continuous d. indirect For tooth movement, the force need not be continuous, but it is critical that the force be applied for a minimally acceptable period of time to elicit the biologic response necessary. The amount of force (heavy or light) determines the biologic pathway of tooth movement and the formation or lack of formation of a hyalinized zone with under- mining resorption. 11 the fluoride concentration in most dentifrices range from a. 1-5 ppm b. 900-1500ppm c. 450-700ppm d. 4000-6000ppm 12 in a full upper denture the post palatal seal is determined by a. the technician b. the depth of the vibration line c. 2-3mm 13 how is scrap amalgam is stored - under sulfide 14 the leas likely situation for a carcinoma to occur in the oral cavity is a. floor of the mouth b. alveolar ridge c. lateral border of the tongue 15 histologically, the loss of the rete peg often is a sign of a. pemphigus b. lichen planus c. pemphigoid d. syphills 16 which of the following represents the predominate type cell type in crevicular epithelium a. mast cell b. PMN c. macrophage d. lymphocyte e. plasma cell 17 which of the organisms are involved with periodontal disease a. P. gingivilits b. E. species c. C. rectus d. Bacteroid e. all of the abov 18 Each of the following has been associated with gastric limitation, except a. acetaminophen b. alcohol c. ibuprofen d. indomethacin 19 The most common reason for fracture of an amalgam in class 2 pedo molar toth a. insufficient deth b. saliva contamination during condensation d. line angle too sharp (sharp axis-pulpal line angle) 20 the best reason for RPD over fixed partial denture a. hygiene b. cooperation c. esthetic 21 where is the gold directed on an MO onlay spruce a, faces pulpal axial line angle b. occlusal floor c. pulpal floor d. gingival floor You never have it at a 90 degree angle, if you direct to the pulpal, gingival or occlusal it would be a 90 degree which will create a hot spot. So it needs to be attached at an angle to allow flow of gold. 22 Which injection post the greatest risk for a hematoma – PSA 23. For anterior crown the finish line should be a-at cervical edge b-below cervical edge c-between cej and epithelium lining d-between crest and attached gingiva 24. Behaviour modifiaction definition - a type of psychotherapy that attempts to modify observable, maladjusted behavior patterns by substituting a new response or set of responses to a given stimulus. Psychologists have developed many techniques to modify patient behavior by using the principles of learning theory. Examples of techniques/methods used mainly in pediatric dentistry. – A procedure that slowly develops behavior by reinforcing successive approximations to a desired goal. 25. Reciprocal anchorage? Elastic bands to close diastema 26. 3mm crowding in 8 yr child , what is the treatment plan? a-extraction of canines and observe b-exraction of primary canine and give lingual arch c-give lingual arch wire 27. Sequestra seen in – osteomyelitis 28. Morpphine over dose causes all except? a-somnolence b-constipation c- pinpoint pupils 29. Gingival retraction cord disadvantages? 30. antiviral given oral also cure mucous and systemic disaesa? Cotrimazole, fluconazole 31. Side effects of oral sedation 32. Nitroglycerine side effects 33. Necrotisizing sialometaplasia - Found on hard palate caused by ischemia to minor salivary gland 34. 15 year old has fever, malaise, vesicles,l ymphadenopathy? a- cat scartch disease b-acute herpetic gingivostomatis 35. Which bacteria causes elastenase, collagenase? a- strep b- staph c-gingivalis (Pseudomonas aeruginosa is the answer, but i don't have that in my choices) 36. Automated defibrilator, how does it work? a- monophase function b-2 shocks c-contraindicated below 12 year old d- discharge when needed 37. W hich antibiotic work on gingiva? a- erythromycin estolate, b-erythromycin stearate c- azithromycin 38. For which factor is least likely to refer endo case Dilacerations Calcification Inability to obtain anaesthesia Mesial inclination of molar 39. To prevent dimeraliztion of enamel from orthodontic treatment, which method is expensive? a-flouride varnish every 6 months b- 6 months prophylais c-prescribed mouth wash d- community fluoride water 40. patient has brown spot on lower left and with the explorer it is retentive? a- floweble resin b- amalgam c- composite d-pit and fissure sealant 41. Patient is having white flakes on her soft palate , gone when rubbed. using albuterol inhaler regularly since 1 week due to soccer game, what is your diagnosis? candidiasis. albuterol->dry mouth->alter pH-> oportunistic candid-> candidiasis 42. bis-phosphate used in all except? A) Prostate cancer to bone b) breast cancer to bone c) osteomyelitis (ans) D) metastatic 43. Which one is schedule 2? 1- Vicodin (3) 2- hydrocodon + Acet (3) hydrocodone containing products are being considered to be reclassified as Schedule II drugs 44. Which impression has water as by product? Polyether Hyrdrocoilled Silicon addation Condseation sillion ANS: should be polysulfide - Polyether, by product alcohol - Addition silicon no by product. 45. Pain on half of the face, that comes once a month and its? 46. Periodontal problems mostly assoicted with Hypertension Smoking, Deibetes plaque 47. freacture on the orbit of the left eyes which border of the maxillary sinus will it effect? Superior Inferior Postier Anterior 48. What was Chess and Thomas categories of Childern Temprture?I (Activity , rhythmicity, distraction, approach) 49. Stages of AIDS and no. of Leukocyte count. 50. Treacher Collin Syndrome - cleft palate, shortened soft palate, malocclusion, anterior open bite, enamel hypoplasia 51. Optimum treatment for palatal papillary hyperplasia? a) Radical dissection b) Electrosurgery 52. Difference between snuff dipper and nicotina stomatitis - Snuff is by smokeless tobacco and present in the buccal mucosa, nicotina stomatitis is present on palate therefore called smokers palate too. 53. Aspirin and Yinseng? What happens if given together – Bleeding 54. Prostaglandin analogue? Misoprostol 55. Clinical test for Rheumatoid arthritis 56. Sjogrens syndrome is associated with? Lymphoma? T - Sjögren's moderately increases risk for non-Hodgkin's lymphoma 57. Area of least attached gingival? Which tooth - Mand premolar 58. Brown tumors associated with which disease – Hyperparathyrodism, giant cell tumor of the bone. Brown tumors may be rarely associated with ectopic parathyroid adenomas[4] or end stage renal osteodystrophy. 59. Internal component of implant 60. Rubber dam retainer which property- options were 1 modulus of elasticity, 2 elastic deformation, 3 permanent deformation 61. Traumatic cyst treatment? a.Aspirational b.Marsupialization c.Curettage (dd) d.no treatment 62. A 14 year old patient presents to your office for multiple extractions. Patient’s mother informs you that her son took 325 mg aspirin the night before for a headache. Which of the following holds true regarding performing multiple extractions on the patient? a. Do the extractions b. Wait 2-5 days c. Wait 7-10 days d. Wait 12-14 days 63. Use of a gold casting instead of dental amalgam should be considered in the restoration of an MOD carious lesion on a maxillary second molar when A. greater sealing of the cavity is desired. B. the preparation is wider than a third of the intercuspal distance. C. esthetics is the primary concern of the patient. D. All of the above 64. Primary risk factor for periodontitis Tobacco (dd) Diabetes Smoking is one of the most significant risk factors currently available to predict the development and progression of periodontitis. 65. How do you diagnose periodontitis ? Bitwings PA (dd) Pano Yes PA to diagnose periodontitis it is the standard for PDL disease.... for bone loss DD mentioned Bitewings is the best 66. 1) If a tooth is not a candidate for full coverage crown , onlay is the best choice. 2) Conservative onlays have superior retention compared to full crowns . A- both are true B- 1 true n 2 false C- 1 false n 2 true D- both false - if tooth not candidate to full coverge crown , so its not indicated to onlay ( dd) 67. Minimum ferrule effect = 1.5mm 68. Lower denture is loose whats wrong with it? (over extended, under extended???? Overextended 69. The denture base completely covers what muscle a. Medial pterygoid b. Lateral pterygoid c. Masseter d. Buccinator 70. Which of the following are false regarding meperidine Select all that apply – A. most abused drug by health professionals B- most widely used narcotic in American hospitals C- less potent than morphine and produces slight euphoria with miosis D- promethazine is contraindicated in pts taking meperidine. E - used for obstetric anesthesia All true mentioned in ddsurgery except , c and d is false ( no miosis, used with promethazine)

Preet Kanwal's RQ + Mary Esma’s Post

1. Drug interaction of Aspirin with atenolol - long term NSAID use (longer than 1 week) decreses effect of atenolol – Mosby: NSAIDs can inhibit the antihypertensive effect of ACE inhibitors, B blockers, and diuretics. 2. External incision bevel: Gingevectomy 3. Amitriptyline: TCA (tricyclic antidepressant so tx for depression) 4. Aminophilline: bronchodilator so tx asthma 5. Atenolol: Cardio selective Beta 1 blocker 6. Question regarding tilted molar type of denture: 7. Ortho case regarding identification of class 1 class 2 class 3 8. Side effects of antibiotics: Fungal infection 9. Lichen planus, smokers palate 10. Erythroplakia, leukoplakia - precancerous condition 11. CI of nitrous oxide: COPD (Safe for asthma), nasal congestion, upper respiratory tract infection, intestinal obstruction, deformity of nasal structure and drug dependency. Pregnancy is NOT absolute contraindication. Mental retardation is also a contraindication. Sickle cell is not a CI. 12. Bevels in composite: Increase surface area, Increase retention, Aesthetic. Is called esthetic bevel, it reduce microleakage, improve esthetic, increase bond strength (dd) bevel angle is 45-60 (in dpm) 13. Bevels in amalgam: bevels for composite cavosurface --- amalgam only for gingival and axiopulpal ---- bevel only in permanent not primary 14. 200 patient last year 300 this year incidence 100 ÷ 1000 = 0.1 = 10% 15. Common lymphoma of jaw - Burkitt in jaws 16. Greatest reccurrence of cancer of oral cavity? Squamous cell carcinoma 17. Heparin – PTT - HEparin PTT....HEmophilia A PTT...learn this 2 together(HePTT) 18. Coumarin – PT/INR 19. Distance between Implant and tooth – 1.5mm 20. Palatalgingival groove is seen in which teeth - Max lateral 21. Bacteria found on acute pulpitis - anaerobic 22. Synostosis- late closure of sutures, no closure at all,? Early - Synostosis (plural: synostoses) is fusion of two bones. It can be normal in puberty, fusion of the epiphysis, or abnormal. When synostosis is abnormal it is a type of dysostosis. Synostosis within joints can cause ankylosis. (Eg. Craniosynostosis - prematurely fuses). 23. Alot of questions about rapport – mutual sense of trust and openness between indiviuals that, if neglected, compromises communication. Rapport is reciprocal, patients are more likely to respect a clinician’s beliefs and opinions if he or she is willing to truly listen to and respect theirs. 24. Trigeminal neuralgia not common in age before 30? T/F – over 50 years of age TN = Prototypic neuropathic fascial pain: Typically there is a trigger point and the pain presents as electrical, sharp, shooting, and episodic (seconds to minutes in duration). Most commonly seen in patients over 50 years of age. Carbamazepine (Tegretol) is still the mainstay of treatment. 25. Trigeminal neuralgia is characterized by all except - dull constant pain 26. Trismus which space is involved - trismus is main sign of masticatory space infection (masseteric space, pterygomandibular space, temporal space) - DD 27. Aspirin overdose which symptom didnt belong: pyretic (fever) was answer. bcaz aspirin is antipyretic. Present: bleeding from gi, Tinnitus, Nausea and vomiting, Acid base disturbance or metabolic acidosis, Decrease tubular reabsorption of uric acid, Salicylism, Delirium, Hyperventilation 28. Turners tooth local trauma or infection 29. 74 yr old patient needs a check up but first you gotta talk with his doctor bcause of his medical chart which organization is in charge of that? Some options... Osha, medicaid services, health insurance portability accountability act (hippa) 30. Modeling? make child observe her siblings or other px 31. Newborn whith 2 white lesions located in median palatal raphe: , something of the newborn? Epstein pearl present in median palatal Raphe in newborn 32. Lesion with suspected malignancy: incisonal biopsy 33. A pic of a patient with ulcerative papilas red in the whole mouth and red macules in the skin and patient felt tired: leukemia, peripheral giant cell geanuloma? 34. Syndorme with eyes bulging out - Crouzon 35. Pic of dentinogenesis imperfect - 36. Supernumerary teeth in what stage - Initiation 37. First sign of development of teeth 6th week? Dental lamina 38. If patient has been on penicilin and comes with fever and more pain, change antibiotic but clindamycin wasnt an option, options included erythromycin and tetracycline. 39. Most potent bronchodilator - Isoproteronol 40. Stages of treatment: Emergency... Maintenance And all that 41. T test: 2 means 42. Chi square: 2 categorical variables 43. Cohort retro and perspective: Risk factor, Retro= historical. 44. Case control (ODD): rare disease. 45. Exam that failed to prove 5 cases that were positive for disease: false negative 46. Chroma: Saturation of color 47. Patient whit green and orange stains: medications, diet,? Poor oral hygiene 48. Cleft lip and palate in caucasians 1:100 or 1:500 Cleft lip alone 1:1000 Cleft palate alone 1;2000 Both 1:700 0r 1:800 49. Difference between fear and anxiety: fear is focal anxiety is generalized, fear unknown anxiety known, ??? fear is known anxiety unknown 50. Which thing decreases or increases in age dont remember but i answered value - Chroma increase, value decrease and hue unchanged 51. Untreated decay frequently in black? more caries in hispanic- more untreated caries in black due to lack of finance (mosby) 52. Functions of the collimator in rx: Reduce exposure 53. Intruded primary central 5 mm with 3 yrs of age: Observe 54. Apexogenesis do not : root lengethening, root vascularization 55. Internal bleach: external or internal cervical reabsorption? external cervical resorption 56. One wall defect? Hemiseptum 57. Chancre what it resembles? I put squamous cell carcinoma 58. Nitrous oxide contra indication: sicke cell anemia or hemophilia? SCA is not CI, not absolute but relative - to prevent sickle cell crisis 59. The setting of vinyl polysiloxane silicone can be retarded by latex gloves, eugenol? 60. Imbibition in hydrocolloids? Imbibition (absorption of water from the air) and syneresis (loss of water to the air or surrounding environment) occur with both,, so its true it occur with hydrocolloids (dd). But mostly we see it in clinic with alginate, and popularity of agar impression is limited becoz it need special equipment. 61. Bone morphogenic protein? BMP are present in DFDBA bone graft which makes it osteogenic 62. Alot of questions about perio connective tissue and flaps 63. Which procedure cannot be done in the distal of the 2 mnd molar to increase attached gingiva in a moderate pocket something like that - Distal widge, cant be done if we dont have adequate attached gingiva,,, while Apf we use it to increase attached gingiva

NBDE YesIcan August 2017 1. Analog (definition) - Is a replica of the actual implant, for lab purpose 2. Distance between implants - 3mm - between mental nerve 5mm - between vital tissue 2mm - buccal, lingual, maxillary sinus, nasal cavity and inferior border all are 1mm - between adjacent natural teeth 1.5 - 2-3mm below cej of the adjacent tooth 3. Taurodentism (definition) - apical enlargement of pulp 4. Cross sectional study (3 questions to identified what type of study) – descriptive study, used for prevalence and incidence, there is no correlation in these studies, no cause and effect relation.

Cross-sectional study—study in which the health conditions in a group of people who are, or are assumed to be, a sample of a particular population (a cross section) is assessed at one time. Consider the hypothesis that drinking alcohol increases the risk of developing oral cancer. If researchers chose to conduct a cross-sectional study to explore this hypothesis, they might examine a group of men who drink alcohol and compare the occurrence of oral cancer among men who are not alcohol drinkers. The researchers could then determine whether there is an association between the presence of oral cancer and alcohol. Although this study is relatively quick and inexpensive, its potential to contribute to a judgment of causation is limited because it cannot determine whether the outcome (in this case, oral cancer) occurred before the men started drinking or if it developed as a result of some other cause (e.g., metastasis).

5. Clinical trial (2 questions) a.Clinical trials—Clinical trials attempt to evaluate the effects of a treatment. A clinical trial aims to isolate one factor (e.g., a new drug) and examine its contribution to a patient’s health by holding all other factors as constant as possible. Well-designed clinical trials use a double-blind design in which neither the subject nor the investigator knows to which group a subject belongs. is design helps prevent the potential for a biased interpretation of treatment effect (better or worse) that might occur if either the investigator or the subject knew to which treatment group (i.e., placebo or experimental agent) a subject belonged. Clinical trials compare the incidence of disease and side effects between the groups in the study to draw inferences about the safety and efficacy of the treatment or treatments under investigation. b. Community trials—in a community trial, the group as a whole is studied rather than the individuals in it. The more similar the communities, the more valid the results. A known example of a community trial was the 1945 Newburgh-Kingston water fluoridation trial. In this study, NaF was added to the water of Newburgh, New York, and DMFT was compared with Kingston, New York, which was non uoridated.

6. Hazard communication paper ( 2 questions, one to identify which institutions created it (OSHA) and one to identify what materials are named hazardous) - chemical hazard MSDS, For general hazard OSHA. MSDS regulated by OSHA through Hazard Communication Standard.

Hazardous chemicals—the OSHA hazard communication standard requires employees to receive training about the risks of using hazardous chemicals and the safety precautions required when handling them. Employees must be trained in identification of hazardous chemicals and PPE to be used for each chemical. This training must occur within 30 days of employment or before the employee uses any chemicals and annually thereafter. Just as with the blood-borne pathogen standard, a written plan identifying employee training and detailing specific control measures used in the work- place must be compiled for hazardous chemicals. Penalties can be imposed on the employer if the office is not in compliance. 7. Chemical sterilization - Gluteraldehye 10 hrs 8. Informed consent (autonomy) 9. Veracity (they gave me the concept) - Principle: Veracity (“truthfulness”). The dentist has a duty to communicate truthfully. 10. Radio resistant cells - Nerve was the answer 11. Cherubism - Bilateral jaws expansion, tx: no treatment 12. Ectodermal dysplasia ( 2 questions, they used words like anhidrosis (no sweat glands) and Hypothricosis (decrease in hair)

Hereditary ectodermal dysplasia.1. X-linked recessive condition that results in partial or complete anodontia. 2. Patients also have hypoplasia of other ectodermal structures, including hair, sweat glands, and nails. .

13. Most common side effect of nitrous oxide (Nausea) 14. INR ( What it measure? extrinsic pathway, intrinsic pathway…..) 0.8 -1.3 normal - for surgery on patients taking anticoagulants 2-2.5 15. A child 16 kg how much LA ( options 34-48-72-115) 16*4.4 is 70.4 16. Ludwing angina ( 2 or 3 questions , what space is not involve, antibiotic they gave me options of meds) - Sub mental, sub lingual, sub mandibular space involved, Retro pharyngeal not involved. Penicillin, metronidazole, clindamycin, and ciprofloxacin are often the antibiotics of choice. 17. Patient in dialysis do dental treatment day after dialysis 18. Suppression of cortisol (was a weird question, options, 20mg, 200 mg, 10mg, 1mg for 2 weeks in 2 years) 19. The more common site of caries: pit and fissures 20. Arthroscopy (they gave a short case question) - Steroid injections, disc manipulations, have a look in joint space

Surgical treatments for temporomandibular disorder . 1. Overview—surgical treatments of the TMJ include arthrocentesis, arthroscopy, disc repositioning, disc repair or removal, condylotomy, and total joint replacement. Arthroscopy involves the placement of two cannulas to allow access for intracapsular instrumentation of the superior joint space. Disc manipulation, disc release, posterior band cautery, and disc repositioning and stabilization techniques all have been described. Arthroscopy appears to be an effective modality in a select group of surgical patients and offers a potentially less morbid access to the joint.

21. Gardner syndrome - Supernumeary teeth, multiple polyp, intestinal polyp, osteomas. 22. Indirect rest instead amalgam to get better… For ideal contour, I think indirect restoration instead of amalgam 23. More common PSICHIATRIC pathology in older ( Mania, depression, ) 24. Vertically face divided (5, 3) 25. Penumbra (They gave the concept) - Penumbra is lack of sharpness of the film. It is a fuzzy, unclear area that surrounds a radiographic image and is affected by focal spot size (smaller the better), film composition (larger the size of crystals less sharp the image), and movement during the exposure. 26. Osteomyelitis Staph -- infection from in to out -- radiolucent--- pain after ext -- pen v I. V Inflammation of bone (and bone marrow) or osteomyelitis is common in the jaws. Most lesions are associated with extension of periodontal or periapical inflammation. Others are associated with trauma to the jaws. Pain, paresthesia, and exudation are typically present. 27. Patient said “I don’t have time to quick smoking (contemplation, precontempation…) 28. Down coding and unbundling 29. Most crucial in replantation after avulsion ( time, open or closed apex…) 30. Harder area to floss (mesial of first pm) 31. Bimaxillary protrusion (the gave me the concept) - Bimaxillary protrusion refers to a protrusive dentoalveolar position of maxillary and mandibular dental arches that produces a convex facial profile. 32. Pka has effect on… The lower the pKa (dissociation constant) of the local anesthetic, the faster the onset of action, Low pka --- more free base--- fast onset. 33. Which is not radiopaque (AOT, Ameloblastic fibroma, Odontoma) 34. Least likely to recur - AOT 35. Anticancer drug with effect in acid folic - Methotrexate 36. RPI I bar fracture what do you do? Soldering 37. Brown tumors – Hyperparathyroidism, brown tumor of hyperparathyroidism is called giant cells lesion too, histologically resembles to central giant cell granuloma. 38. Macroglosia is not common in …. Hyperthyroidism 39. Clean tongue to prevent… odor 40. Sulfas MOA - stimulate insulin release from beta pancreatic cells (sulfonylureas hypoglycemic drugd). About sulfonamides antibiotic (inhibit PABA folic acid synthesis). 41. Differents questions of neurofromatosis (2 or 3) (What do you not see? Café au lait, lisch nodules of the iris, super numerary are SEEN. 42. X ray to light and to dark 43. Geographic tongue (they gave me a short description) - Geographic tongue (benign migratory glossitis, erythema migrans) 1. Common (2% of population) benign condition of the tongue of unknown cause. 2. Appears as white annular lesions surrounding atrophic red central zones that migrate with time. 3. Occasionally symptomatic (mild pain or burning). 4. No treatment necessary. 44. Nicotinic stomatitis (they gave me description) Nicotine stomatitis (Figure 4-4). 1. White change in palate caused by smoking. 2. Red dots in the lesion are inflamed salivary duct orifices. 3. Not considered premalignant, unless related to “reverse smoking” (lighted end in mouth). 45. Retentive claps (suprabulge, infrabulge…) 46. Initial treatment of LAP (antibiotics, antibiotics+ SRP, SRP alone..) 47. Lefort 1 ( the answer was maxillary sinus) 48. Questions of apexogenesis and apexification (they gives you a short case and you have to decide) Apexogenesis --> Vital tooth / Apexification --> non-vital 49. Treatment of ranula - excise 50. Liquid in Glass ionomer - Polyacrylic acid 51. Source of epithelium for grafts - donor 52. Half erupted third molar in a 18 years old patient, WHY do you extract that molar? (to avoid chronic pericoronaritis, because thirds molars can produce crowding in anterior, to treat pocket on the distal of second molar and other option) 53. Tooth with crown best pulpal test - Thermal test 54. Nsaid who does not affect palettes – Celecoxib, selective cox 2 inhibitors. 55. Multiple Myeloma first sign – bone pain 56. Melanoma common localization - Palate and gingiva, no tx. 57. Reciprocal anchorage - used for closure of midline diastema, use of Crossbite elastics. 58. Pictures of Dentigerous cyst, leukemia, amelobrastic fibroma 59. PM with 3 canals – max 1PM 60. Outline shape of prep of mandibular first molar (triangular, ovoid, trapezoid…) 61. Slob rule - Same lingual opp buccal 62. Disadvantage of partial thickness flap - thin flap so tear easily, can get lost easily, Difficult to elevate. 63. Most stable in moisture - PVS 64. More common reason of amalgam failure - if ask for failure water contamination, if ask for fracture then cavity preparation. The contamination of the amalgam by moisture during trituration and condensation is unquestionably the principal cause of failures. 65. Antibiotic seen in GCF (no tetracycline in options) Gingival crevicular fluid: Azithromycin 66. Gingivectomy where to do incision - External incision, at the base of the pocket. 67. Perio maintenance interval (3months, 4, 6…) 68. True about Niti over stainless steel (options something like harder, more adaptable in curvatures (keep shape), flexibility….) Shape memory is the ability of nitinol to undergo deformation at one temperature, then recover its original, undeformed shape upon heating above its "transformation temperature". 69. Intrapulpal anesthesia – back pressure 70. Supernumerary teeth seen in, Anterior maxilla ... Occlusal radigraph best, Gardener, Down syndrome, Cledocranial Dysplasia. 71. Treatment without consent - Battary 72. Least probable canal ledges (short, small, large, curved) 73. Med who increase cardiac output (beta blockers, alpha, ace…) – beta agonists, digitalis - It increases the force of contraction of the heart by inhibiting Na+,K+-ATPase and indirectly increasing intracellular calcium. 74. Wheezing seen in ( asthma, COPD) 75. Common between Aspirin and acetaminophen) - Antipyretic and analgesic 76. Montelukast Moa - Leukotriene receptor antagonist 77. Culture of sensitivity used to… Bact resistant 78. That question of 3 years old with 5 mm intrusion (they did not give to much details) leave it 79. Most common emergency in dental office - Syncope! 90% - Hyperventilation 9% 80. Herpetic gingiva stomatitis (short description) - Acute herpetic gingivostomatitis diagnosed early (within 3 days of onset) is treated immediately with antiviral therapy (acyclovir, 15 mg/kg ve times daily for 7 days). All patients should receive pallia- tive care, including plaque removal, systemic NSAIDs, and topical anesthetics. Proper nutrition should be maintained. Patients should be made aware of the contagious nature of this disease when vesicles are present. 81. What do you look at Gardners (osteomas) 82. Ignoring a patient bad behavior - extinction 83. #8 with radiolucency increase in size for all of the following except (apical scar) 84. Occlusion (3 questions where is located the interference? Where to grind, balanced occlusion concept - Occlusion, in a dental context, means simply the contact between teeth. More technically, it is the relationship between the maxillary (upper) and mandibular (lower) teeth when they approach each other, as occurs during chewing or at rest. 85. More common cells in cellulitis – leukocytosis (white cells above the normal range in the blood) 86. Most common tooth lost due to periodontitis - Max 2nd molar, maxi first most affected by periodontitis and max 2nd most common lost. 87. Pathology with early teeth lost - Papillion-Lefevre syndrome, Chediak-Higashi syndrome, hypophosphatasia, neutropenia, leukemia and in some cases Langerhans cell histiocytosis 88. Epinephrine avoid in …. multiple sclerosis, Hypertension, thyroidism 89. Side effect of albuterol - Candida and xerostomia 90. Dental lamina ( 2, 6, 8, 12 weeks) 91. Best bone to implant located in - Mand ant, type 1 92. Implants: high torque low speed, low speed high torque…) 93. Repair of veneer - Micro etch, etch, silane, bonding 94. Cause of mucocele - trauma 95. Pulp necrosis what type of resorption (inflammatory, replacement, surface..) 96. K sparing drugs - Eplerenone (Inspra), Spironolactone (Aldactone), Triamterene (Dyrenium) 97. Patient with SCC and he said something like “ Are you saying that I have cancer? How do you respond? (Do you want to call someone to be with you now? This has better prognosis than others cancers. Others options) 98. Common salivary gland tumor - Pleomorphic Adenoma (mixed tumor) 99. Properties of Zinz polycarboxilate, and other with GIC 100. Staffne cyst - Depression of mandible on lingual side. Below IAN. Stafne (static) bone defect a. Diagnostic radiolucency of the mandible secondary to invagination of the lingual surface of the jaw. b. Located in the posterior mandible below the mandibular canal. (Static, Psyduo, Mand Fossa, No treatment, Round and RL) 101. Nystatin Moa - binds to ergosterol 102. Primary teeth who resemble mandibular first molar - Mand.2nd 103. What to use with disable kid (voice control, consistency) 104. Patient with bradycardia what to give him (atropine, epinephrine..) – Use atropine, and anticholinergic, if bradycardia is present (DD). Atropine has direct inhibitory effect on vagal mediated sympathetic stimulus resulting into reflex tachycardia, even though Epinephrine do also have effect but it causes Cardiac Arrthymia. 105. A question said something like more cost effective fluoride treatment - community water fluoridation. 106. During extraction more common (fracture, hemorrhage…) 107. Incisal guidance - Vertical and horizontal overlap 108. Nsaid preferred for kids - preferred nsaid for kids is Ibuprofen, if not NSAID then acetaminophen. 109. Nsaid who does not affect platelets – Acetaminophen, Naproxen, Celecoxib 110. Index of caries – DMFT (decayed missing filled tooth) 111. Question of periapical cemento osseos dysplasia which one is not true? 112. Allograft concept - this graft material is obtained from cadaver bone that is processed to ensure sterility and to decrease substances in the bone that can trigger host immune response. However, this process destroys the osteoinductive capability of the bone, whereas the osteoconductive property of the graft remains. Although allograft avoids the need for a second surgical site, a greater amount of the grafted material is resorbed compared with autografts. Allogra materials include undecalci ed, freeze- dried bone allogra (osteoconductive material) and decalci ed, freeze- dried bone allogra (osteo- genic material owing to the presence of bone mor- phogenetic proteins that are exposed during the demineralization process). 113. Which is not used in cast restauration - irreversible impresiom mater? 114. Anug (2 quesntions, onhe for treatment and one they gave me a short description) 115. Acute necrotizing ulcerative gingivitis. 1. Characteristics. a. Painful, bleeding gingival tissues. b. Blunting of interproximal papillae. c. Pseudomembrane on the marginal gingiva. d. Fetid breath. e. High fever. Caused by fusiform bacilli (spirochetes) and other anaerobes. Most common in teenagers and young adults. Responds well to debridement, oxidizing mouth rinses, and antibiotics. Treatment of acute necrotizing ulcerative gingivitis includes evaluation of the medical history, application of topical anesthetic followed by gently swabbing the necrotic lesions to remove the pseudo-membrane, and removal of local factors such as calculus (o en with ultrasonic instruments unless contraindicated by the medical history). Systemic antibiotics should be prescribed only if there is evidence of lymphadenopathy or fever. The patient should be instructed to avoid alcohol and tobacco, rinse with chlorhexidine, get adequate rest, remove bacterial plaque gently, and take an analgesic as needed for pain. Patients should return in 1 to 2 days for reevaluation and further debridement. Patient should be seen again approximately 5 days later for reevaluation; further counseling regarding diet, rest, and tobacco use; reinforcement of oral hygiene instruction (including chlorhexidine rinses); and periodontal evaluation. 116. Belladone alkaloids ( contraindications) - Belladonna alkaloids and phenobarbital combination is used to treat cramping and spasms in the stomach and intestines. Contraindications: acute edema of the lungs, mechanical stenoses of GI tract, megacolon, narrow angle glaucoma, prostate adenoma and tachycardic arrhythmias. 117. Stridor seen in - laryngospasm 118. Efficacy concept - intrinsic effect 119. More common in men (diabetes, hemophilia, hypertension…) 120. What is not a benefit of ¾ crown one a Full crown - time for placement 121. treatment – Analgesic, sedative dressing, no antibiotics. 122. Treatment contraindicated in trough trough furcation, - GTR. We do guided tissue regeneration for class 2 furcation... but through and through furcation either class 3 or 4 and we can't do GTR 123. Least effective reducing interproximal plaque – water pick 124. Morphine side effects - respiratory depression, constipation, dysphoria, toxic: miosis, coma, resp depr. 125. Most common reason of cardiac arrests in children: respiratory failure 126. Bur used to polish porcelain - diamond Tons of questions of pulpal pathology, flaps indications and contraindications, side effects of medsGuys thanks for your everything. Some extra advice, check flaps deeply. As you can see I can't remember to much of farma because I hadn't too much and I'm bad on it. But about farma read about cancer treatments, radiation and biophosphanates, what meds can cause dry mouth, smoking cessation meds. I got a few ones from mdb reworded but mostly the same. I found it a good resource to practice and learn. I did Kaplan q bank and I loved it. I got like 6 cases. One a kid with so many many meds!! He had kidney transplant. Was 11 years old. They ask me about medications he was taking and which one was causing him gingival hyperplasia. He had 170/110 of BP or something really close. They ask which of his pathologies demand send him to emergency. - cyclosporine for gingival hyperplasia and I pick BP for emergency Another case of and old men with many heart conditions A lot of meds too. They ask me about meds and prostho and ortho I think. Not sure because I got two cases of old men. One of them had a first molar missing and second and third molar tilted to the space. They asked me what type of coast I DO NOT use on the second premolar (20) options distal I bar, distal facial wrought wire, mesial circumferential, mesiofacial something else. Ortho question was about move molars and premolar and about them at space. What meds was causing him dry mouth. And alcoholic case young guy. Teeth in really bad shape. Many perio questions on him about what to do in this teeth, or what not to do. One case of 14 years old girl with canine out of arch. She was asthmatic. What angle case? What to do with canine or extract and appliance or expand or appliance? She had a first molar with deep groove and a little be dark. What to do on that molar? Sealant, amalgam, nothing... Every single case had patient management questions. Two sentences to decide if both true, both false, first true second false, first false second true. I used a lot the highlights with the mouse to keep in mind key words. The alcoholic patient was 18 month clean. They asked me if he was cured. They ask if for example the cardiac patient should be premeds due to.. something that doesn't need premed although he has something else to be premeditated for. I don't know if I'm explaning well. But they play with your mind. If you read the case and the patient has something that alerts you that ok this patient needs premedication but in the question the ask like this patient needs antibiotics due to his angina. So you have to read carefully. I got a case of a young guy 43 years old I think. He was not happy with his teeth. He has diastema and they ask why do not close this space. Because closing diastema is hard in adults, because he has deep bite or something like that and many others options I had that question of US population. There's in US 55% or more women than men. More than 75% of people who lives in home care are women X percent of women after 65 are marriage... I choose the 75% .... one because that is a big number, two because is hard to me to believe that more moms are in home care centers (dads used to behave worst ) im sorry guys I don't know the correct answer.

KARA PAL RQs (JULY 19)

1. K sparing drug? Spironolactone, amiloride, triamterene - K+-sparing diuretics qreduce the driving force for K+ movement into the lumen. 2. Virax in oral cavity? Virax (acyclovir) - Acyclovir is an antiviral agent with activity against Herpes simplex virus types 1 and 2, (HSV-1 and HSV-2) and varicella zoster virus (VZV). 3. What procedures you cant do in AIDS patient? No prophylaxsis, because candida 4. Opiods effects: Miosis, constipation, cns depression, also respiratory depression, and ONLY MEPERIDINE mydriasis 5. Papoose contraindication: Long - term restraining, Mentally or physical abnormal 6. Battery? - treat without informed consent 7. Lot of prostho occlusal interference questions 8. Which study doesn’t show cause and effect – cross sectional, observational study 9. Drugs those block prostaglandins has increased effect on gastric mucosa? - Non selective COX inhibitors 10. Bimaxiallry protrusion - Both arch procline. When u see facial profile lip incompetence lips strain n prominence lip profile 11. Combination syndrome: Kelly syndrome, maxillary CD opposing mand anterior natural teeth Cc:- flaby max ant bone resorption - extrusion of mand teeth. Loss of bone from the anterior part of the maxillary ridge, overgrowth of the tuberosities, papillary hyperplasia in the hard palate, extrusion of the lower anterior teeth, the loss of bone under the partial denture bases. 12. Characteristics of band and loop: -Doesnt prevent sobre-eruption of opposing tooth. - Its non functional 13. Least fracture resistant? lithium, feldpathic, zirconia - The order of chipping resistance (from least to greatest) was: feldspathic porcelain and a leucite glass ceramic (which were similar), followed by the lithium disilicate glass ceramic and the two resin composites (which were similar), and finally the zirconia which had the greatest resistance to chipping. 14. Pka has effect on what? Onset of Action. Low pKa = fast onset, High pKa = slow onset 15. 16 kg of 3 year old how many mgs LA to give? 16*4.4=70.4mg 16. What meds you give in osteomyelitis? Clindamicin 17. Patient with bizarre behavior and disorientation you give what? insulin, glucose, thyroid - disorientation and confusion most commonly indicate hypoglycemia which needs glucose 18. Initial stages of sedation what patient feels? Tingling of hands 19. Condensing osteitis? Excessive bone mineralization around the apex of an asymptomatic VITAL TOOTH. Radiopacity may be caused by low-grade pulp irritation. This process is asymptomatic and benign. It does not require endodontic therapy. 20. 2 questions on Periapical Cemento osseous dysplasia - Reactive process of unknown cause that requires no treatment. Commonly seen at the apices of one or more mandibular anterior teeth. No symptoms; teeth vital. Most frequently seen in middle- aged women. Starts as circumscribed lucency, which gradually becomes opaque. An exuberant form that may involve the entire jaw is known as florid osseous dysplasia. 21. What cyst in roots of mandibular premolar? Traumatic bone cyst 22. What lesions are not radiopaque? AOT, Ameloblastic fibro odontoma 23. Least likely to occur? AOT, odontogenic myxoma 24. Which anticancer drugs has effect on folic acid? Methotrexate 25. Radiographic pictures Rg ameloblastic fibro odontoma - Unilocular or multilocular lucency, well-defined radiolucency often associated with an unerupted tooth. 26. 9 year old kid swollen gingiva, recurrent skin infections: May be Impetigo, pemphigus? 27. What is complication of maxillary molar extractions: Sinus perforation, Fracture tubeocity 28. What is easily curable? macule, hematoma, ... 29. 2 questions on Incisal guidance 30. RPD I bar fractured what you do? - Construct a new one 31. What does conjugation do to a drug? – Phase II. Makes it more water soluble to enhace excretion. 32. Patient with flared maxillary incisors and diastemas, to improve esthetics what you evaluate first? Radiograph, Diagnose wax up, probing depth 33. Downcoding and upcoding – Down when insurance company bills your treatment to the cheaper procedure. Up - when you type your treatment to be more expensive than actual value to the insurance company. 34. Brown tumors: Hyperparathyroidism 35. Which conditions have macroglossia? Cretinism, acromegaly, beckwith - wiedeman syndrome, down syndrome, hypothyroidism. 36. You don’t do RCT with PA radiolucency in which patients? Diabetes? 37. Systemic antifungal drugs ? Fluconazole, keroconazole, amphotericin B. 38. Why do you clean tongue? esthetics, prevent odor (Halitosis. Bad breath) 39. What bacteria in chronic periodontitis? P gingivalis, intermediate, treponema denticolla, forsythia. 40. Nasal obstruction what sedation you cant give? Inhalational - True nitrous contraindicate 41. Sulfonamide MOA - COMPETE with PABA to inhibit PABAs actions, which prevents bacterial folic acid synthesis to inhibit cellular growth. 42. Traumatic neuroma – ORAL TRAUMATIC NEUROMA - a lesion (soft tissue tumor) caused by trauma to a peripheral nerve, usually appearing as a very small nodule/swelling (O.5 cm in diameter) of the mucosa near/over the mental foramen on the alveolar ridge in edentulous areas, lips, & tongue. MOST COMMON SITE IS OVER THE MENTAL FORAMEN IN EDENTULOUS PATIENTS, but they can occur wherever a tooth has been extracted. Extraction sites in the anterior maxilla & posterior mandible are common sites. In the oral cavity, the traumatic neuroma may be due to trauma from a surgical procedure (i.e. tooth extraction, from a local anesthetic injection, or accident). A nodule or swelling PAINFUL WHEN PALPATED, as applied digital pressure elicits a response described as an "electric shock". Multiple neuromas on the lips, tongue, or palate may indicate the patient may have MEN III (Multiple Endocrine Neoplasia Syndrome). Treatment: surgical excision of the nodule with small proximal portion of the involved nerve. Recurrence is uncommon. 43. Patient wore denture for 10 years and there is 6*3 white lesion on buccal of mandible? Biopsy, or observe - Any lesion red or white doesn't go away for 2 weeks we should do biopsy. 44. Treatment of concussion? Baseline vitality tests and radiographs. Occlusal adjustment. No immediate treatment is needed. Let the tooth “rest” (avoid bite), then follow-up. 45. Primary maxillary incisors intruded 5mm what you do? On 2-3mm we were waiting to reerupt, so probably in younger patients reposition, older like above 5y/o extraction. 46. Radiolucency in primary molar furcation area? Pulpectomy - If 1st molar extraction 47. Most common caries location? Pits and fissure 48. Features of arrested caries? Hard, dark, shiny 49. Composite characteristics? Rough, glass filler 50. Opaque porcelain features? Mask the metal and provide chemical bonding with metal 51. Phelobith location? nerve, vein (I assume phlebitis) - Phlebolith is a Calcification in a vein 52. Lefort 3 fracture what side gaze is limited to? Laterally, medial or downward - LeFORT III (Transverse Fracture or Craniofacial Oysfunction)-a fracture in which the entire maxilla and one or more facial bones are completely separated from the craniofacial skeleton . These patients have restricted mandibular movement. (nasoethmoidal) LeFORT II (Pyramidal Fracture)- a unilateral or bilateral fracture of the maxilla in which the body of the maxilla is separated from the facial skeleton and the separated portion is pyramidal- shaped. Signs: periorbital edema, ecchymosis, subconjunctival hemorrhage, and nose bleeding. A common finding is paresthesia over the distribution of the infraorbital nerve. (Zygomaticomaxillary) 53. Lefort 1? LeFORT I (Horizontal Fracture} – a horizontal segmented fracture of the alveolar process of the maxilla in which the teeth are usually contained in the detached portion of the bone, causing an OPEN BITE. A LeFort I osteotomy is most commonly used to correct maxillary retrognathia. (intramaxillary) 54. Axillary freckling seen in? Nerofibromatosis 55. Radiographic image of lateral wall of nasal fossa 56. radiographic is too light? Why? - underexpost. may be too old solution 57. If you take radiograph 10 Ma with 1 sec exposure and you take another x ray with 0.5 sec what mA will you give to get same density radiograph? 5, 10 or 20 – qmAs = mA X time (sec) 10= x. 0.5 x=10/0.5 x = 20 58. Serpentile something feature? migratory glossitis – (Geographic tongue) 59. HIV infection feature? I put Usually asymptomatic in initial stages (lymphocytes reduction) 60. Normal skeletal and class 3 dental what could be the reason? Loss of primary mandibular 1st molar or incisor crowding or thumb sucking 61. 3mm lack of mandibular arch how you treat? Disk distal of molar, I put lingual arch and observe 62. Max 2nd molar less keratinized tissue, which graft will you give? - lateral /pedicle flape? 63. Most common crown root fracture tooth – max anterior 64. Primary mandibular 2nd resembles to? Permanent first 65. If you have ledge while doing RCT what you do? Bypass with smaller instrument 66. Access opening of Mandibular molar? Trapezoidal 67. Modified widman flap? Full thickness 3 incisions used to expose bone defects 68. When we don’t do gingivectomy? (aka gingivectomy contraindication) Bone surgery needed, inadequate attached gingiva, apical to the mucoging junction 69. Mouth breather features? class 2 occlusion, adenoid facies, open bite, dry perioral tissues. 70. 7-8 questions on studies 71. 2 questions on OSHA 72. Which study FDA do to check drugs? Randomized clinical trials. 73. 4 questions desensitization kind of terms. 74. Behavior shaping question, patient say I don’t wanna quit smoking? precontemplation stage - The behavioral change process occurs in several stages. a. Precontemplation — an individual is not considering a behavior change. Contemplation — an individual begins to consider a behavior change. Preparation — preparing to take steps to change (o en expresses a desire to change a behavior). Action — an individual is engaged in taking action toward behavior change (o en requires support for his or her e orts). Maintenance — an individual attempts to maintain a changed behavior. 75. Eg of secondary prevention? Amalgam and composite restorations - Secondary prevention is the elimination or reduction of a disease after it occurs. An amalgam restoration is considered secondary prevention because is removed and a restoration is placed. 76. Where you give General A.? 2 year old kid needs lot of restoration 77. Most common periodontal failure? Max PM, Max molar, Mand PM, and Mand molars - max molars due to trifurcation 78. Twisted questions on Meperidine and MOA read it carefully - Meperidine. a. Can be used orally. More potent than codeine but less potent than morphine. A metabolite, normeperidine, is a CNS stimulant. Not recommended for long-term pain relief. Contraindicated with MAO inhibitors. t1/2 ~ 3 hours. - MEPERIDINE (DEMEROL) - a SYNTHETIC narcotic (opioid) agonist that is weaker than morphine, but equally addictive. It's duration of action is shorter than morphine, and it is the only narcotic agent that does NOT cause miosis (pupillary constriction). Demerol is used as an IV supplement during conscious sedation procedures, but is less potent than morphine and much less potent than fentanyl. Demerol is also used as an oral medication to control pain after dental surgery. Oemerol treats moderate- to-severe pain (acute dental pain) and may be used as a preoperative medication to relieve pain and allay anxiety. Length of Effectiveness: IV or 1M (3hrs) and not very effective given orally. When combined with promethazine it is know as Mepergan Fortis. Meperidine can cause seizures, tremors, and muscle spasms. Synthetic Meperidine Oerivatives: Fentanyl (Sublimaze) - a synthetic potent opioid analgesic used primarily as an IV sedative during conscious sedation procedures or procedures requiring general anesthesia. It is 80-1OOx more potent than morphine. 79. What pain killer drug is safe in pregnant patient? tylenol 80. When does dental lamina starts? 2 weeks of utero, 6 weeks, 12, 10 81. What stage supernumerary occurs? initiation, histo, apposition 82. What % of community water fluoridation? 74% - Mosby 83. Mechanism of fluoride? options weren’t the regular one we read, very twisted terms they used – Topical effect of constant infusion of a low fluoride concentration into the oral cavity = enamel remineralization. Fluoride converts hydroxyapatite fluorapatite by substituting OH' for Fl'. Fl decreases the solubility of hydroxyapatite crystal, while increasing the crystal size. Fluoride ion is easily exchanged with the hydroxyl ion in the lattice structure of enamel because FI is slightly smaller than OH- with a greater affinity for hydroxyapatite crystals than OH-. Fluoride inhibits glycolysis where sugar is converted into acid by bacteria (fluoride ion inhibits enzymatic production of glucosyltransferase). 84. Multiple myeloma feature? bone pain? Yes, first sign, Punch out lesion, Bence jhon proteins, 85. 2 question on best bone for implant? Type 1 86. What is implant abutment? it is connecting element b/w crown and implant fixture 87. You making FPD and you see 1 prosthesis in Radiograph is completely seating on implant what you do? Screw the implant, take another RG, section the FPD 88. Disadvantages of cemented implant restoration? need more occlusal space 89. Fearful patient how do you respond? gradual desensitisation 90. Leading question, open ended and reassurance ? leading question: you are not affraid, do you? (not recomendable to use) - open ended: how are you feeling? (let the pt to communicate and its the best choice - reassurance: what advantages or disadvantages do you see in tx plan? (making sure pt understood). 91. Modeling in fearful kid ? let another person to show him who to behave, like brother or stuuf 92. Buccal limitation of mandibular denture - masseter 93. Lingual limitation of Mandibular denture? Genioglossus, Frenum, Mylohyoid, Sup constrictor, Platoglossus 94. ANUG resembles what? I put herpetic stomatitis - recurrent herpes-stomatitis mimicking acute necrotizing ulcerative gingivitis (ANUG). 95. When do you do elective RCT? 96. Lefort 1 includes what structure? Maxilla, hard palate, there is a sign of bilateral hematoma on palate = guerin sign 97. Smokeless tobacco causes what? Leukoplakia - verrucus carcinoma 98. Max anterior teeth placed too superior and too anterior what sound will affect? F and v 99. When you make rest on molar you make rest deep enough in marginal ridge, buccal incline, lingual? - yes 1.5mm 100. What is not included in the consent? Cost 101. Epinephrine contraindicated in thyroid? in hyperthyroidsm yes 102. (Few questions on some recent study questions. Never heard and don't remember) 103. Dental hygienist and 3 doctors in practice, dental hygienist screwed the case who will be legally blamed? dental hygenist and the doctor "supervising" 104. Abcess include marginal and interproximal gingiva called? Gingival abscess, pericoronal, periapical 105. What is the indication of half erupted third molar in 18 years old? dental hygenist and the doctor "supervising" 106. Another question on complication of maxillary molar extraction: 1st molar root in sinus, 2nd molar related with maxillary tuberosity fracture. 107. You can avoid ledge if canal is: Small, large, short or curved - short less incidence of ledge and long more incidence 108. External bevel incision? gingivectomy 109. Ear pain refer to? Mandible molars 110. Lots of pharma questions 111. An athlete having pain and soreness on tmj when he wake up? MPDS - myofacial pain. Myofascial Pain Dysfunction (MPD) Syndrome-the most common cause of TMJ pain. It is a disease primarily involving the muscles of mastication. MPD often responds to an acrylic night guard (occlusal separator or occlusal appliance) along with a soft diet, limited talking, and elimination of gum chewing. Moist heat applied to the face and non-steroidal anti-inflammatory agents are also helpful during the acute phase. 112. Sublingual varicosities? Twisted options - age 113. Radiograph of dentigerous cyst - Dentigerous Cysts (Primordial or Follicular Cysts)-cysts associated with the crowns of unerupted teeth, probably the result of degenerative changes in the reduced enamel epithelium. Well-defined usually unilocular radiolucency. 114. Distance between two implants: 3 mm 115. # of granulocyte for surgery? 1500 - Any ANC < 1500 cells / mm3 is considered neutropenia, but <500 cells / mm3 is considered severe - For platelet 50.000 in dd surgery 116. Periodontitis common in? African American, males 117. Nursing bottle caries? Options were so new and confusing never heard before sorry don’t remember - Max incisors and molars, mandibular protected by tongue. Early childhood caries – ECC: definition by the American Academy of Pediatric Dentistry (AAPD)—the presence of more than one decayed (noncavitated or cavitated), missing (owing to decay), or filled tooth surface in any primary tooth in a child younger than 6. Severe ECC: a. Younger than 3 years—any sign of smooth surface decay. b. Ages 3 to 5. (1) One or more cavitated, missing (owing to caries), or filled smooth surface in primary maxillary anterior teeth. (2) A decayed, missing, or filled surface (DMFS) score of greater than 4 (age 3), greater than 5 (age 4), or greater than 6 (age 5). Previously termed “baby bottle syndrome” or “nursing bottle caries.” Typical presentation of “baby bottle syndrome.” a. Caries are present on maxillary anterior teeth and primary molars. The mandibular incisors are unaffected because of the tongue covering these teeth during feeding. History often reveals that the child is consistently put to bed with a nursing bottle containing milk or a sugar-containing drink. AAPD recommendations. a. Infants should not be put to sleep with a bottle. Ad libitum nocturnal breast-feeding should be avoided after the first primary tooth begins to erupt. Parents should be encouraged to have infants drink from a cup as they approach their first birthday. Infants should be weaned from the bottle at 12 to 18 months of age. Repetitive consumption of any liquid containing fermentable carbohydrates from a bottle or no-spill training cup should be avoided. Oral hygiene measures should be implemented by the time of eruption of the 1rst primary tooth. An oral health consultation visit within 6 months of eruption of the 1rst tooth and no later than 12 months of age is recommended to educate parents and provide anticipatory guidance for prevention of dental disease. An attempt should be made to assess and decrease the mother’s or primary caregiver’s Streptococcus mutans levels to decrease the transmission of car- iogenic bacteria and lessen the infant’s or child’s risk of developing ECC. In children younger than 2 with moderate to high caries risk, a smear of fluoridated toothpaste should be used. All children age 2 to 5 should use a pea sized amount of fluoridated toothpaste. 118. Oral infections in what leukemia: Chronic Monocytic Leukemia 119. Treatment of ranula? Enucleation, If recurrence removal of gland 120. What is mucocele? Stupid options extravasation of fluid, common in lower lip, due to trauma to ducts of minor salivary glands. 121. Too deep PPS (Posterior palatal seal)? I put Gag - also unseated of denture 122. Beta 1 selective actions - decrease heart rate 123. Some medicine name I don’t remember, it was something like, given in lung patients for better functions and improve asthmatic effects. Mechanism of it? theophilline? albutamol? salbuterol? montelukast? 124. LA mechanism include all except? Read everything about it 125. Exhale wheezing sign of what? wheezing on expiration is asthma 126. Montelukast drug moa? Block the action of leukotrienes - leukotriene receptor antagonist (LTRA) used for the maintenance treatment of asthma and to relieve symptoms of seasonal allergies. 127. Anti-depressants? 128. Dentures major connector function Where to place rest? Stability and rigidity, on marginal ridge 129. What happen if temp of developing solution is too high ? Dark image 130. Inc water:powder ratio in gypsum does what? Increases working/setting time, Decreased setting expansion, decreases strength – The increase in water:power ratio decreases the number of nuclei of crystallization per unit volume and increases the amount of space between crystalizing nuclei, thus increasing porosity when drying. 131. Which bur is not good for porcelain? 132. Most porcelain liable to fracture: a.PFM b.zirconia c. pressed leucite, d.silicate core 133. Common between gardener and peutz -- a polyps b odoronta c pigmentation 134. Epinephrine inc anxiety? yes, anxiety has elevated HR and epinephrine can make the feeling worst 135. Cevimeline - For treatment of xerostomia 136. Russell bodies - multiple myeloma 137. Sarcoidosis resemble - tb 138. Can we give chlorhexidine to ADHD (attention deficit hyperactivity disorder) child ? no 139. Pigeon chest seen in? Rickets - Pectus carinatum can also be caused by vitamin D deficiency in children (Rickets) due to deposition of unmineralized osteoid. 140. Bond strength and more lasting of resin is more in: Dentin Enamel Both 141. Rifampin is contradicted to pregnant women? yes or no 142. Sodium hypochlorite properties: No chelating, Removes organic material 143. Tooth most involved in vertical root fracture: Max. 1st premolar..for sure..Guys it vertical root fracture. 144. Part of facial nerve affected during TMJ pain: temporal, masseter zygomatic cervical 145. Pemphigoid and pemphigous difference: Pemphigus- nikolsky sign and suprabasilar cleft / Pephigoid - subbasilar cleft 146. What flow of saliva affects denture from seating – Thick 147. What would reverse LA: Phentolamine (non selective alpha blocker), its A alpha adr antagonist 148. Tricyclic antidepressants inhibits: drugs of FIRST choice to treat UNIPOLAR DISORDER (depression). Tricyclics inhibit neuronal reuptake of NE & Serotonin (5-HT) in the brain, which results in potentiation of their neurotransmitter actions at post- synaptic receptors. 149. Patient has PFM crowns and amalgam restoration, apply: 1% (not sure) neutral sodium fluoride - 2% of naf, ph= 9.2 150. What not to do in veneer: try in paste for shade apply silane to inner surface apply bonding agent etch enamel with hydroflouric acid 151. Recent research shows the relationship between periodontal disease and: cardiovascular disease 152. Xerostomia causes all of the following except: A.smooth surface caries B.candiosis C.leukoplakia D.salivary gland infection 153. Pulpal anesthesia - Back pressure 154. Primary teeth with occlusal convergence - Max first molar 155. Short crown resistance and retention - Buccal groove 156. Xerostomia drug receptor: muscarinic 157. HIV patient prophylaxis which we cant do: Candida prophylaxis 158. White lesion on 75 yr old pt, from 19 years, what to do? Biopsy or cytology? Relining and Observe 159. Filler disadvantage - Difficulty in polishing, decrease esthetics also 160. Bur for porcelin cutting: Diamond burs 161. Retentive arm where angaged - At the infrabulge area 162. Best material for cervical lesion – GIC 163. Occlusal rest: On marginal ridge for support 164. Orthodontic treatment consequences: Resorption 165. Systemic desensitization: teaches the person to replace the feelings of anxiety with feelings of relaxation when the object or behavior is present. 166. Precontempling: Pt not willing to quit the habit 167. Which changes in results in Benign Migrating Glossitis (aka: geographic tongue)? a) atrophy filiform b) hypertrophy filiformc) atrophy fungiform d) hypertrophy fungiform 168. Treatment for Benign Migratory Glossitis: a. Perform Biopsy b. Administer Topical Corticosteroids c. Reassure the patient that no treatment is required d. Administer antibiotic therapy with ciprofloxacin(Cipro) for 10 days. 169. An ameloblastoma is most likely to develop in the wall of a dentigerous cyst. 170. Sequence treatment: Endo, perio, operative, surgery and ortho

Radiographic appearances: 1-Ground Glass appearance--> Fibrous dysplasia 2-Punched out radiolucencies--> Multiple Myeloma 3-Cotton Wool Appearance--> Paget's Dz 4-Tooth Floating in Air--> Eiosinophilic Granuloma 5-Snow Appearance--> Calcifying Epithelial Odontogenic Tumor(CEOT) 6-Honey Comb Appearance--> Odontogentic Myxoma 7-Soap Bubble Appearance--> Aneurysmal Bone Cyst, Cherubism 8-Scooped out radiolucencies at mid root level--> Histiocytosis X 9-Scalloped radiolucencies around the roots of teeth--> Simple bone cyst aka traumatic bone cyst 10-Beaten Metal appearance on the skull--> Crouzon Syndrome 11-Enlarged marrow spaces--> Sickle cell Anemia 13-Widened PDL with dissolving bone--> Non-Hodgkin lymphoma 14-Moth-Eaten radiolucency--> external resorption 15. Salt and pepper appearance radio-graphically-COC 16.Gosth teeth appearance- Regional Odontodisplasia 17. Hair on end -Thalassemia 19.Cherry blossom- Sjogren syndrome 20.Sunburst pattern- osteosarcoma 21.Abnormal widening of PDL - scleroderma and osteosarcoma 22. Teeth floating in space - hand shuller chrestien

Sukh Singh’s RQs (JUNE 2015)

1. Lots of questions on pt management. 2. Fluoride supplement for 7 year old with no fluoridation in water = 1.0mg/day

3. 9 year old lost 2nd premolars. With space maintainer. = 9 yrs no premolars if so, band and loop 4. Antibiotics and their side effect. Which was not matched correctly. 5. Which of following antibiotic use is restricted due to its side effect: tetracyclin, chloroamphenicol, PNC, cephalexin = Chloroamphenicol, causes aplastic anemia 6. Pt allergic to both ester and Amine which LA would you use: Diphenhydramine can be used as an alternative to ester and amide local anesthetics in minor procedures of short duration. 7. Prilocaine causes methemoglobinemia = Oxidizing agents such as prilocaine are the most common cause of acquired toxic methemoglobinemia. 8. Epinepherin in 1.8, 2% 1:100k. 9. Now working and working incline interference. 10. What is the important factor when reducing a cusp. Outline form, retention form, resistance form or convinence 11. Medazolam overdose, which drug u give = Flumazenil - Flumazenil (Mazicon), a benzodiazepine antagonist, used to reverse effect of benzodiazepines in the event of an overdose. 12. Contraindication of nitrous sedation: Head injury, bowel obstruction, pneumothorax, middle ear and sinus infections, COPD (emphysema or bronchitis – NOT ASTHMA, there ARE NOT contraindications for the use of nitrous oxide sedation in asthmatic patients), first trimester of pregnancy, with whom communication is difficult (autistic patients), having a contagious disease since it is difficult to sterilize entire tubes. 13. Patient has Bisphosphonate-related osteonecrosis of the jaw (BRONJ) bronj and bone is exposed, what is treatment? A) hyperbaric oxygen, B) sc/rp C) chlorhexidine rinse and oral antibiotics D) ALL 14. Best test to determine a irreversible pulpitis – Thermal, cold test both reversible n irreversible will show response but the diff is that if the stimuli is removed pain is subsided within 5sec fr reversible. Cold test-lingering pain 15sec / Heat test to differentiate from reversible. 15. How would you differentiate between a reversible pulpitis and periodontal lesions = Pulp vitality test 16. How would you treat necrotizing ulcerative gingivitis with no obvious systemic symptoms – NUG: Tissue debridement with topical or local anesthetic, rinsing with chlorhexidine or diluted hydrogen peroxide (h2o2) and oral hygiene. If systemic: antibiotic therapy with metronidazole or penicillin. 17. Focal distance increased from 8 to 16 Howard that intensify = Intensity = 1/4 of distance. So when distance increases, intensity decreases. I1 = D2 I1 = (16)2 256/64=4 4 TIMES DECREASED I2 D1 I2 = (8)2 18. Side effects of corticosteroids: Clouding of the lens in one or both eyes (cataracts), High blood sugar, which can trigger or worsen diabetes, Increased risk of infections, Thinning bones (osteoporosis) and fractures, Suppressed adrenal gland hormone production. 19. Doing composite restoration you have 1 mm of remaining dentin thickness what would you use for lining if any = glass ionomer cement for greater than 0.5 mm

20. Best restorative material for 13-year-old with posterior small occlusal caries not involving interproximal areas. 21. Main cause of alveolar osteitis (dry socket): Thought to develop because of increased fibrinolytic activity causing accelerated lysis of the blood clot - Fibrinolysis of clot 22. Main sign or symptom associated with dry socket. The patient develops severe, dull, throbbing pain 2 to 4 days after a tooth extraction. The pain is often excruciating, may radiate to the ear, and is not relieved by oral analgesics. (Dull throbbing pain, foul smell, slough). 23. How convulsions are managed in dental setting: Secure the patient and avoid injuries, proper airway. 24. Difference between fear and anxiety. 25. Pt with folded arms and looking down what do u say. 26. Child gets their dexterity by what age to brush unassisted: This is an adult job. No child has adequate dexterity for brushing teeth until age 6-9(average 7) yrs and flossing 10 years. Tooth brush dexterity 7 yrs. 27. Systemic fluoride won't benefit which tissue- root, occlusal, inter-proximal, smooth surface. 28. Main features of achondroplasia: ACHONDROPLASIA-the most common type of DWARFISM. Clinically, the child appears very short (around 50 inches), fingers are stubby, bowed legs, bulging of the forehead, bossing of the frontal bones, saddle-like nose, and mandibular prognathism. 29. Which is not important about designing tx plan for frail woman - age, dexterity, previous dental tx, ability to remove. 30. 1.5-2.5 year child more prone to injuries due to: accidental prone, Overprotective parent. Abuse. No fully coordinated development. 31. pt in for #8 crown. Missing 1-4 and 12-16. How would u record occlusion: Old dental records 32. 5 year old cavity prep, Haitian accidentally exposures mesiobuccal I'll come on what is the best course tx. Options were pulotomy with stainless steel crown, Pulp With calcium hydroxide followed by stainless steel crown, root canal treatment – (DPC is a relative contraindication in primary tooth) 33. 12-year-old patient who had crown fracture with exposed pulp comes the following day. For treatment, what is the best treatment: In young patients with immature, still developing teeth it is advantageous to preserve pulp vitality by pulp capping or partial pulpotomy. Also, this treatment is the choice in young patients with completely formed teeth. Calcium hydroxide is a suitable material to be placed on the pulp wound in such procedures. 34. A nine-year-old child suffering from some spontaneous pain on primary molar, it is determined and this is non-vital what is the best treatment for that tooth: pulpectomy- keep it as a space maintainer 35. Two points of Frankford plane-Porion to orbitale 36. Minimum distance of the implant two vital tissue: Generally Implant is placed at least 2mm away from any vital structure and min 5 mm ANTERIOR to mental foramen because of anterior loop on inferior alveolar nerve 37. Order of treatment for mild to moderate chronic periodontitis: Mild: oral hygiene instructions, moderate: scaling and root planning 38. Minimal interval for Perio therapy. 1 month. 3 month 6 month 12 month. After periodontal treatment, the first recall visit should be scheduled at 3 months. With excellent plaque control and maintenance of periodontal health, the interval can be lengthened to 4-6 months. 39. Questions on OSHA and their standards for blood borne. 40. Questions on ethic principle, justice, non-malevolence, veracity, and beneficence. 41. Phases of periodontal therapy 42. Couple question on sensitivity and specificity of the disease: Sensitivity & Specificity are INVERSELY proportional. As the specificity of a test increases, the sensitivity decreases. Sensitivity: ability of the test to diagnose correctly a condition or disease that actually exists. Sensitivity measures the proportion of people with a disease who are correctly identified by a positive test. Sensitivity is defined as the number of true positives (TP) divided by total number of potential positive findings (true positives and false negatives) in the sample. Sensitivity = TP/TP + FN. Specificity: ability of the test to classify health. Specificity is defined by the number of true negative (TN) results divided by the total number of false positive (FP) + true negative (TN) results in a sample. Specificity = TN/FP + TN. 43. Question on type of studies for example case control randomized controlled trial. 44. Few questions on chroma, Hue, value 45. Which is not risk factors of oral cancer: HIV, smoking, alcohol, HPV. 46. If a patient has an adverse reaction to medication who do you report to CDC, FDA, OSHA, EPA. 47. Epstein-Barr virus is associated with what: BURKITT'S LYMPHOMA, Epstein-Barr Virus is also associated with infectious mononucleosis, & orally hairy leukoplakia. In Africa, the Epstein-Barr virus (EBV) has been linked to Burkitt lymphoma, as well as to a form of acute lymphocytic leukemia. In the United States, EBV most often causes infectious mononucleosis (“mono”). 48. Oral Hair leukoplakia is most commonly found which which disease: HIV 49. Malignant carcinoma is associated with which gland parotid, sublingual, submandibular, minor oral salivary gland: Most series report that about 80% of parotid neoplasms are benign, with the relative proportion of malignancy increasing in the smaller glands. 50. Frequency of cleft lip and palate in Caucasian 51. Which race is associated with occlusal caries white, blacks, Native Americans, Hispanic. 52. Length of manual toothbrush can penetrate sulcus close compared to floss 53. Which would be least effective and cleaning furcation, toothpick, soft brush, water pick, interdental brushes 54. Another few questions and furcation 55. Least favorable solvent to store avulsed tooth. Milk, saliva, water Hans solution. 56. Extraction order of maxillary posterior teeth: 3rd molar, 2nd molar, 1st molar 57. Extraction of manibular third molars in association with IAN canal 58. Which structure is least likely to show on intro oral radiographs Mundibular foramen, mental foramen, hamulus notch. 59. Which radiograph is best to evaluate bone loss: Bitewing - Periapical Film- film of choice to evaluate root surfaces, supporting bone, and PDL space (not for occlusal or proximal caries). 60. Supernumeraries occur at which stage- initation. 61. Best test for patient with warfarin. Inr, PT time etc. 62. Questions on adrenal crisis. Addison's Disease-caused by HYPOSECRETION of aldosterone & cortisol. For adrenal crisis, treat with 2ml of cortisol (hydrocortisone) . Corticosteroids represent replacement only in Addison's Disease. 63. Question bout space between palate and the metal frame try in but good fit on master cast. What's the reason. Shirnkage of alloy, distortion of master impression etc. 64. Space maintenance on pt missing mans lateral in overall spaced dentition 65. Indication for 3rd molar extraction. 66. Pain on biting and eating. Thermal and ept test normal. Sharp pain on MB cusp only and fine with biting on other cusps. - Fracture off mb cusp 67. Question bout crown lengthening. 68. Most etiological factor for progression it periodontitis. Calculus, bacteria/biofilm Etiological: plaque( bac) - Contributing :calculus 69. Max denture extended to far buccally will get interference from- coronid process. 70. Soft tissue transillumination in young child to see. Siolathasis, leukemia, herpetic gingivistomatitis one more option I forgot. 71. Caries start at. Pits fissure, interproximally. Above contact point. Below contact point. 72. Which is not important when determine caries rate. Oral hygiene, frequency of carbohydrate, quantity of carbohydrates. Amount of cariogenic bacteria. 73. Cervical cavity prep. Kidney shaped 74. How to test the root caries. Softness, dicolouration. Two more options. 75. Auriculotemporal nerve damage consequences. FREY'S SYNDROME (AURICULOTEMPORAL SYNDROME) - an unusual/uncommon phenomenon that arises due to damage of the auriculotemporal nerve and subsequent reinnervation of the sweat glands by parasympathetic salivary fibers. Frey's syndrome can occur after surgery (i.e. removal of a parotid tumor, ramus of the mandible, or infection of the parotid that has damaged the auriculotemporal nerve (branch of V3). Gustatory sweating (sudoracion gustative) is the chief complaint. Patient exhibits flushing and sweating of the involved side of the face during eating. 76. Which is the antibiotic prophylaxis for patients with allergic to penicillin- Clindamycin 77. Drug that not reduce saliva. Propanol, atropine, scopolamine and one more which is similar to atropine 78. Most common type of arthritis. Osteoarthritis, rheumatoid arthritis osteoporosis 79. Bout mandibular fracture and displacement: Fracture of angle of mandible displaced in which direction in edentuluous pt: Anterior and superior 80. Best test for pt on coumarin therapy. PT - INR 81. Facial height is divided into 3rds, 1/2, fifths.

Shreya's post

https://www.youtube.com/watch?v=u6UnG7gzscA&t=0s

https://www.youtube.com/watch?v=qnPz3Cvztvo&feature=youtu.be

INTERFERENCES CORRECTIONS WORKING BULL (INNER INCLINES) BULL (INNER INCLINES) LUBL (OUTER INCLINES) NON-WORKING LUBL (INNER INCLINES) LUBL (INNER INCLINES) PROTRUSIVE DU (DISTAL INCLINES DU (DISTAL INCLINES FACIAL CUSP UPPER) FACIAL CUSP UPPER) ML (MESIAL INCLINES ML (MESIAL INCLINES FACIAL CUSP LOWER) FACIAL CUSP LOWER) CENTRIC INTERFERENCE MU (MESIAL INCLINES UPPER) MU (MESIAL INCLINES UPPER) (FORWARD SLIDE) DL (DISTAL INCLINES LOWER) DL (DISTAL INCLINES LOWER)

1. Lingual cusp of upper molar hit lingual inclines of facial cusp of mandibular molars, which movement? Non working: Balancing side (non-working side) interferences generally occur on the inner aspect of the facial cusp of mandibular molars. 2. Contact on lingual portion of buccal cusp of mandibular molar, what kind of interference? Non working 3. Wear facets on lingual inclines of maxillary lingual cusp and facial inclines of mandibular facial cusp on left side? a) Left working interface b) Protrusive interface c) Right non working interface d) Left working interface 4. Wear on buccal of maxillary premolars due to, due to mandibular movement? working nonworking 5. The mesiobuccal incline on the mesiobuccal cusp of mand molar has wear: this is because of movement in which direction(s) !!! 1. Working and protrusive movement 2. Non working and protrusive movement 3. None of the above 6. Tooth 30 gold crown has wear located on the MB cusp of the MB incline, cause A. protrusive and working side movement B. protrusive and non-working side movement C. only protrusive D. Non-working side movement 7. Max molar on mesial slope of mesial lingual cusp where do you have wear on lower teeth? Mesio buccal cusp of lower molar 8. The mesial angle of the ML of max 2nd molar occludes with what on the man 2nd molar a. Mesial MB cusp b. Distal MB cusp c. Mesial DB cusp d. Distal DB cusp 9. Which periodontal procedure we cannot do in AIDS patient? Flap 10. Which bur not used in porcelain? Carbide 11. Sublingual varicoses -age or hypertension: Age or hypertension – Varicosities or varices, are abnormally dilated and tortuous veins. Age appears to be and important etiologic factor because varices are rare in children but common in older adults. The most common type of oral varicosity is the sublingual varix, which occurs in two thirds of people older than 60 years of age. 12. Vertical root fracture common in which tooth: Mand 1 molar 13. Which bacteria cause elastase and collengase: P gingivalis 14. From where caries start to progress? inner surface, outer surface, DEJ, dentin 15. Kenedy class 3, type1, which would provide best stability? rest, baseplate, major connector, retainer - major connector for stability and especially single palatal or anterior posterior platat strap is used for class 3. Primary function of rest is to provide vertical support and single palatal strap is indicated in Kennedy 3 for stabilization. 16. Radiographs for Paget's disease? Lateral (Cotton wool appearance) 17. Which is not good for a 25 year old patient in a behaviour modification.,?! A) operant conditioning B) carrotstix 18. Modified widman flap by primary or secondary? Primary when we approximate the ends, like flap and suture, secondary we leave it open with out approximation if ends , like tooth extraction, scaling and root plannig , gingivectomy. Two basic methods of wound healing (soft tissue): 1. Primary intention (also called primary closure): Involves minimal re-epithelialization and collagen formation, allowing the wound to be “sealed” within 24hrs. Healing occurs more rapidly with a lower risk of infection and with less scar formation and less tissue loss than wounds allowed to heal by secondary intention. Examples include: well-repaired and well-reduced bone fractures. 2. Secondary intention (also called secondary closure): involves re-epithelialization via migration from wound edges, collagen deposition in the connective tissue, contracture, and remodeling. The site fills in with granulation tissue. Healing is slower and results in scarring and wound depression. Examples include: extraction sockets, poorly reduced fractures and large ulcers. 19. External bevel which used in gingivectomy is apical to pocket (junctional epi) but coronal to mucogingival junction. Internal bevel runs apical to the crest of alveolar bone , below mucogingival. 20. Infection of premolars drains to submand or subling? , sublingual space and pterygomandibular space.

21. Worst cantilever? Worst cantilever is central to lateral (not lateral to central) we always cantilever pontic to abutment 22. How to differentiate chronic apical abscess and chronic apical periodontitis?! Both are necrotic, so pulp vitality is negative, but for chronic apical periodontitis there may be slight tenderness to percussion or palpation testing. 23. Initiallly caries bacteria? Stcococus, lactobacil, str salivarious, sangius?? For intial caries its streptococcus mutans, sanguis is initial bact in plaque 24. How to decrease penumbra? To decrease penumbra use smaller focal spot, less film to object distance, increase target to film distance. 25. Incidence of oral cancer more on? Black male White male 26. Thick cortical with dense trabecular bone? What type? D2, type two 27. Test with 2 continuous variable? Chi or T? 2 continuous variable: person correlation – 2 categorical variables: Chi square – 2 constant variable: linear regression. 28. Percentage of N2O and O2? In dd surgery 2 cards mention its 60%, in ada website its 70%, couple of friends had the exam and same q its either 60% in options or 70% , so it will not come both options (For nitrous maximum 70% , and oxygen 30%)- For kids nitrous not exceed 50% 29. Nitrous oxide is contraindicated for children with sickle cell anemia? T or F 30. Fluoride highest value? max 3 ppm in DD pedo 31. Fluoride average value: systemic: 0.7-1.2 ppm 32. Healing after scaling and root planning - regeration, long junctional epithium, repair / Periosurgery – regeneration / Flap, by repair and forming long junctional epi - Srp is by new connective tissure reattachment. 33. Bacteria in day 2: Gram + cocci and rod 34. Battle sign where? Mastoid echymosis, fracture of middle cranial fossa 35. What to do first in a patient with hearing problem.,?! Interpreter 36. Lymphoma in jaw name: burkitt's lymphoma 37. Operative considerations to be taken in Albright syndrome? Bisphosphonates are used to prevent recurrent fractures and they act as antiresorptive agents 38. In kidney disease-cretine increase or decrease? Increase 39. Withdrawal of odontoblast process mainly in response to chemical, mechanical injury 40. Pigmentation related syndrome.,?! 1. Albright syndrome 2. Neurofibromatosis 41. Which drugs Shouldn't be taken during renal failure.,??? Aspirin, nsaids 42. Which is not imp about design, for frail women (weak and debilitated)? age, dextrity,ability to remove 43. Most retained deciduous: Primary 2nd molar 44. Definition of ductility: Deform under tensile strength - Malleability under compressive strength 45. Green colour change in porcelain indicates: silver 46. Toxic dose of fluoride.,???? 5mg/kg 47. Conditions where we have to use anaesthesia without epinephrine.,???? hypertension, multiple sclerosis, hyperthyroidism 48. In a class 2 patient with 8mm over jet, which surgery has to be performed.,?! 1. Genioplasty 2.maxillary setback 3. Sagital back 4.maxillary expansion 49. Rigidity or support nd rigidity or stability? For major connector its rigidity and stability 50. Radiolucency in primary molar at furcation area: extr/ pulpectomy? 51. Interaction bw meperdine nd mao? seizures, coma 52. Which is called whn pt charge several procedure at onces? Upcoding. bundling, unbundling, downcoading 53. Numbeness on right lower molar ,where trauma- angle, symphysis, condyle, coronoid 54. 5mm intrusion in primary teeth what to do? No treatment and let the tooth to reerupt. 55. Bone drill temp -29, 36, 57, 70 56. Lingual flange recorded by whom: Superior pharyngeal constrictor 57. Trismus which muscle: Massser if massetric space infection - Medial prerygoid due to IAN block puncture 58. What anesthesia posterior soft palate? Lesser palatine 59. Bur use to polish porcelain? Diamond 60. Antibiotics given in impacted tooth? Before extraction, after extraction, healing delayed, acute suppression 61. Aspirin should stop dts 81 mg before extraction? True or false - yes as in dd, stopped 7 days before extraction 62. Frequency of cleft lip nd palate in causacian is high true or false 63. Bill out for a core build up and crown and insurance says build up is only covered, what is this? Bundeling Unbundling Upcoding 64. Downcoding 65. Dentist not reporting the waiver of copay to insurance overbilling downcoding upcoding bundling - unbundling 66. Downcoding is reimbursing less money than dentist deserves. Upcoding is charging more than total deserving. How it is different from bundling and unbundling? Bundling is terming multiple procedures as one and paying for that single one. Unbundling is separating / disintegrating single big procedure into several smaller ones and charging for each. 67. Multiple procedures cut down to increase reimbursement A. Unbundling B. Bundling C. Downcoding D. Upcoding 68. Unbunding: When dentist charge more than the actual benefits by charging a separate fee for each component 69. Upcoding: is fault practice where the doctor bill higher than what was done. Insurance company pays more than wat it has to pay 70. Downcoding- here the insurance company pays less by changing the code to a lesser cost procedure 71. Dentist has done two procedures but the third party pays only for one procedure what is it called a)underbilling b)overbilling c)upcoding d)downcoding 72. A study is designed to determine the relationship between emotional stress and ulcers. To do this, the researchers used hospital records of patients diagnosed with peptic ulcer disease and patient diagnosed with other disorders over the period of time from January 2014- January 2017. The amount of emotional stress each patient is exposed to was determined from these records. This study is : a Cross Sectional b Cohort c Historical Cohort d Clinical Trial e Case – Study 73. Which is true of intrapulpal anesthesia. 1. Produce anesthesia after 30 sec 2. It does not cause discomfort 3. Produce anesthesia by pressure 74. Deep bite most common in caucasians race T/F 75. Open bite most common in Blacks T/F 76. Is thyroglossal duct cyst congenital? T 77. Which development cyst in the neck would move when u swallow – epidermoid, dermoid, thyroglossal 78. Intraligamentary LA needs an Antibiotic Prophylaxis in patients with risk of Endocarditis. T/F 79. Which antibiotic okay to give in myasthenia gravis Penicillin Azithro Erythro Impenem 80. Which sterilization technique do not dull instruments? dry heat 81. Whats the most common tooth to erupt in a crossbite? Maxillary Lateral Incisor 82. Most common impacted anterior tooth--- maxillary canine 83. Most common impacted tooth --- lower 3rd molar then upper 3rd molar and maxillary canine then mnd 2nd pm 84. Most common supernumerary tooth — mesiodens Most common ectopically erupted tooth — maxillary permanent first molar followed by canines - Man: canine & 2 pm 85. Most common malignancy of oral cavity—squamous cell carcinoma 86. Most common benign tumour of oral cavity — fibroma 87. Most common retained tooth – primary mandibular second molar 88. Most common recurring cyst— odontogenic keratocyst 89. Most common cyst in oral cavity— periapical cyst 90. Most common lichen planus- reticular lichen planus 91. Most common dermatosis to affect oral cavity- lichen planus 92. Most common chemical burn in oral cavity –aspirin burn 93. Most common topical fluoride in adults – stannous fluoride 94. Most common topical fluoride in children—1.23 APF gel. 95. Most common burshing technique- scrub technique 96. Most common developments cyst- nasopalati ne cyst 97. Most common complication of GA (op)-nausea 98. Most common used drug for petitmal epilepsy- no treatment 99. Most common used drug for grand mal - phenytoil 100. Most common drug used for temporal epilepsy- carbomezepine 101. Most common treatment for cyst – enucleation 102. Most common used clasp-simple circlet clasp 103. Most common used face bow in fpd- kinematic 104. Most common complication of RA involves TMJ-fibrous ankylosis 105. Most common salivary malignancy in children – mucoepidermoid carcinoma. 106. Most common salivary malignancy in palate area- ACC 107. Most common type of haemophilia--- haemophilia A 108. Most common type of gingivitis in children--- eruption gingivitis 109. Most common type of cerebral palsy is – athetoid/ spastic 110. Most common nerve involved in C sinus thrombosis – abducent nerve 111. Most common type of impaction ---mesoangular 112. Most common benign epithelial tumour---- papilloma 113. Most common complication of surgical extraction of lower third molar — loss of blood clot 114. Most common used instrument grasp — pen grasp 115. Most common susceptible tooth for caries— mandibular first molar 116. Most common contrast media - iodine in oil 117. Most common cause of light radiographs — exhausted developer 118. Most common cause of failure of RCT— inadequate cleaning and shaping - debridement 119. Most common isolated yeast strain from RCT— Candida 120. Most common bacteria found in root canals --- gram positive 121. Most common part of oral cavity affected by L planus –buccal mucosa. 122. Amantadine for Parkinson D T/F (prevents dopamine reuptake) 123. Which type of study cannot be used to determine cause and effect? Cross sectional 124. Consent comes under the ethical principle? Autonomy 125. Pit and fissure sealant best retained on which teeth? max molars, mand molars, max pms, mand pms 126. Which is the most common non odontogenic cyst in the oral cavity? Nasopalatine 127. Clinically Lichen planus can be commonly confused with A.scc B.verruca vulgaris C.desquamative gingivitis D.histoplasmosis 128. Traumatic cyst treatment? Aspirational, Marsupialization, Curettage 129. For flexibility which clasp use- cast alloy, wroght wire, basemetal

Rima Gandhi's post

1. Adrenal crisis: Hypoglycemia, hypotension, syncope, confusion, slurred speech. Treatment is 100 mg hydrocortisone hemisuccinate 2. Allergic to LA: Allergic reactions to local anesthetic are usually caused by an antigen- antibody reaction (Type III). Administer dinphenhydramine (Benadryl) 3. Patient allergic to both Esters and amides - which Local Anesthetic would you use? A. No need of LA B. Ibuprofen IV C. NO sedation 4.Diphenhydramine 4. Hypertensive pt. Does of LA? Epinephrine? 0.04 mg epinephrine. 5. Hypertensive emergency - mecamylamine and nitroglycerin (parenteral) hypertensive urgency - nifedipine (oral) - sublingual captopril in hypertensive crisis 6. Anaphylactic crisis: Epinephrine 0.3 mg IM or 0.1mg IV Hypoglycemia: Conscious and unconscious pt: Conscious Orange juice - Unconscious IV 50% dextrose in water - Unconscious pt 1 mg glucagon IM Or 50 ml of 50% glucose by rapid IV infusion 7. Acute asthma attack: Albuterol/oxygen 8. Acute asthma unconscious pt: Epinephrine 9. Hyperglycemia – insulin 10. Cardiac arrest: 911, Cpr, Oxygen then epi 11. Laryngospasm: Succinylcholine 12. CHF treatment: Diuretic, Inotropic, Vasodilator 13. Angina or heart attack: Sublingual nitroglycerin 14. Syncope: Pallor first sign - Trendelenburg position - For pregnant pat- lay her on left side with right hip up 15. Lidocaine toxicity. Symptoms and management: CNS depression and acute allergic rxns - treated with diazepam 16. Diuretic pt. What to avoid Digoxin: People with heart failure who take digoxin are commonly given medicines called diuretics, which remove excess fluid from the body. Many diuretics can cause potassium loss. A low level of potassium in the body increase the risk of digitalis toxicity 17. Renal disease pt..what to avoid - Antibiotics of choice in such pt: Avoid mepridine aspirin Nsaids acetaminophen morphine 18. Myasthenia gravis pt - Antibiotics of choice - Treatment – Precaution: Penicillin, Contraindicated: mainly fluroqinolones. 19. Cerebral palsy or What to avoid: Antianxiety drugs 20. Parkinson disease: Levodopa carbidopa 21. Hearing impeared.. First to do: Get translator

Nbde Target’s Post

1. Small elevation around his anterior caries teeth? what it is ulcer or fistula ? Fistula 2. Posterior tongue in relation to occlusal plane while in rest? lower and retruded 3. Hepatitis which test: Hbs (surface) antigen 4. Circumferential clasp: generally used on a tooth-supported removable denture 5. Patient with Class II Kennedy PD, good oral hygiene and low caries index you would use: a. Circumferential clasp b. Back action clasp c. Cast clasp 6. Wrought wire: Wrought wire clasp have greater tensile strength than cast clasps and hence can be used in smaller diameters to provide greater flexibility without fatigue or fracture 7. White lesion who wore denture for 15 years? Old denture, adjust and check in one week. 8. Lesion with chronic inflammatory cells and epithelial lining filled with fibrous wall: Granuloma - Cyst if continuous epithelial lining 9. Voice control doesn’t include - option 1) raising voice 2) gaining child attention? 3) mild punishment 10. Sulfonylurease moa: Increase insulin production 11. Size of radiolucency increase with tooth 8 which cannot be the reason 1) apical scar 2) change in angulation 3) proximity to incised canal 3) canal leakage 12. Schedule 2 drug: vicodin, percocet, hydrocodiene + ibuprofen ? better Percocet than Vicodin. 13. Draining pus from mandibular region which muscle news to be dissected? Abscess is drained by a horizontal incision, made 2–3 cm below the angle of mandible. Blunt dissection along the inner surface of medial pterygoid muscle towards styloid process is carried out and abscess evacuated. A drain is inserted. 14. Coil spring for uprighting: open coil spring to upright a molar. Coil spring tends to "spin" premolars unless precautions are taken. 15. A coil spring used over an arch wire segment to regain space should deliver a force of: A 40 to 60gms B 90 to 120gms C 150 to 300gms D 375 to 450gms 16. A buccal coil spring is used to regain space between 1st premolar & 1st molar. The most common post treatment complication is: A. Pain B. Gingival irritation C. Tendency for the 1st molar to intrude D. Tendency for the 1st premolar to rotate 17. What determines epithelium of graft? Donor epithelium - type of epithelium is determined by donor epithelium (either keratinised or nonkeratinized), epithelium is formed by recipients epithelium or donor basal cells. 18. Tachycardia seen in: Side effect from anticholinergic drugs. 19. Dental abnormalities in down syndrome > class 3 ear infection macroglossia 20. Initial symptom of HIS – herpes: fever, blister on lips 21. Moa of benzodiazepines? Enhance GABA - it facilitates the action of GABA 22. Not an advantage of distraction osteogenesis over osteotomies? Long time, require 2 appointment 23. Advantage of partially covered crown over fully covered? Less reduction of tooth structure, we can do pulp vitality test 24. Tramadol/cyclobenzaprine adverse reaction with will be? Tramadol oral and cyclobenzaprine oral both increase affecting serotonin levels in the blood. Too much serotonin is a potentially life-threatening situation. Severe signs and symptoms include high blood pressure and increased heart rate that lead to shock. 25. International normalized ratio (INR) is closely related to prothrombin time (PT). PT and measures such as prothrombin ratio and IRN are measures of the extrinsic pathways of coagulation. The PT/INR are used to determine the clotting tendency of blood. The INR measures the effect of warfarin (Coumadin), a vitamin K antagonist, effects of vitamin K deficiency, on clotting. PT measures Dicoumarol (INN) or dicumarol (USAN) 26. Which antibiotic works on gingiva? I think question is asking about antibiotic bioavailability in gingiva: Doxycycline 27. Automated defibrillator? Can be given when required, Discharge when needed. The modern automated external defibrillator (AED) abolishes the need for the operator to have ECG interpretation skills. Analize the ECG signal. They evaluate the frequency, amplitude, and shape of the ECG waves. They are designed to be used by people with little training. Automated external defibrillators are available for adult and pediatric patients. 28. Automated defibrilator, how does it work? a- monophase function b. 2 shocks c. contraindicated below 12 year old d. discharge when needed 29. Which bacteria causes collangenase? “Clostridium perfringens” secretes collagenase, a proteinase of “Bacteroides gingivalis” has been reported to induce secretion of tissue collagenase, this is suggested to be involved in the etiology of periodontal disease. “Porphyromonas gingivalis” belongs to the phylum Bacteroidetes and is a nonmotile, Gram-negative, rod-shaped, anaerobic, pathogenic bacterium. 30. 15 year old has fever, malaise vesicle and lymphadenopathy? Primary herpetic gingivostomatitis: Initial infections of HSVI, in some patients, the initial infection with these viruses produces no noticeable clinical signs and can go undetected clinically. In other patients, however, the symptoms resulting from this initial infection can be quite severe, and it is these severe symptoms that are know as primary herpetic Gingivostomatitis. Primary herpetic gingivostomatitis is contagious and requires careful attention to prevent its spread. The initial infection with HSVI usually occurs in childen or in young adults, but it can occur at any age. Primary herpetic gingivostomatitis signs: Oral pain, difficulty in eating and drinking, swollen, red, bleeding gingiva, painful oral ulcers, in the more severe clinical manifestation, this infection is associated with symptoms such as pain, elevated temperature, a vague feeling of discomfort (malaise), headache, and swollen lymph nodes (lymphadenopathy). 31. Antiviral given oral for mucous and systemic diseases? Acyclovir 32. Gingival cord disadvantages? Technique sensitive as the instrument offers poor tactile sensation • It can potentially damage the periodontium. 33. Gingival retraction cord is placed ______crown prep is completed and is removed _____ final impression taken. A. After, after B. After, before – Before taking the final impression after the crown preparation is completed, retraction cord is placed into the gingival sulcus to enable retraction of the soft tissue. The impression will more adequately include the cervical margin of the preparation. 34. Why do we need to keep sulcus dry when placing gingival retraction cord? Dry tissue makes it easier to see the details of the gingival tissue and place the retraction cord. The use of chemomechanical tissue retraction involves a cord impregnated with a homeostatic agent such as epinephrine or aluminum chloride. The homeostatic agent will assist by shrinking the tissue temporarily and controlling bleeding, but the displacement of the tissue is the primary method of action. 35. Opioid causes constipation through: 1-brain receptor 2-stomach receptor 3-spinal cord receptor - GIT receptors: Opoid increase nonpropulsive contractions in the middle of the () and decrease longitudinal propulsive peristalsis - motions critical to moving food through the intestines. This results in food that fails to travel through the digestive tract thus constipation. Morphine and other opioids in GI: decreased peristalsis. 36. Somnolence: OPIOIDS 37. Sequestrum seen in: A sequestrum is usually a complication of osteomyelitis and represents devascularisation of a portion of bone with necrosis and resorption of surrounding bone leaving a 'floating' piece. The sequestrum acts as a reservoir for infection and as it is avascular is not penetrated by antibiotics. It usually requires excision if cure is to be achieved. This is not seen only in osteomyelits, but also in eosinophilic granuloma, fibrosarcoma and lymphoma. 38. 3mm crowding in year old: observe 39. If a child has 3mm crowding on the lower and permanent canines haven’t erupted, what do you do? Observe 40. Mand incisors crowding 3mm at 8 years: A.grind distal of primary mand canines B.extract primary mand canines C.Lingual arch and observe 41. Anterior teeth finish line? Subgingival 42. Tooth borne appliances: Bionator, activator, herbst etc 43. Tissue borne appliances: Functional appliance – Frankel 44. Saw palmetto is contraindiacted with Coumadin, aspirin, blood thinners, anticoagulants. For surgery should discontinue use of saw palmetto at least 2 weeks before surgery due to the herb's anticoagulant effects. Saw palmetto may interact with anticoagulants, antiplatelets, finasteride, hormonal contraceptives and hormone replacement therapy (HRT) 45. Ginseng is contraindicatied with: Ginseng is contraindicated in acute infections, especially those involving fever and during anticoagulant therapy. 46. According to ADA classification for alloy, a noble alloy has more than ---% nobel metal content? 1. - 60% 2. - 75% 3. -25% 47. Large inlays and onlays comes under which classification of ADA ? A= CLASS 2 48. Porcelain adheres to metal primarily by which bond ? chemical bond (COVALENT BOND) - The mechanical bond is a physical interlocking of porcelain and metal made possible by microscopic irregularities in the metal. How does porcelain bond to the alloy? Ceramic adheres to metal primarily by chemical bond. 49. Standard ¾ crown preserves which area? Buccal 50. Butt joint is the BEST or POOREST type of finish line? Poorest 51. The path of insertion of anterior ¾ crown should parallel the long axis of the tooth ? T/F A = FALSE The path of insertion of anterior ¾ crown should parallel the incisal ½ - 2/3 of labial tooth not tooth’s long axis; if parallel to long axis, will cause more gold to be displayed 52. Which finish line is preferred on the Cast gold restoration? A= Chamfer 53. Acrylic resins EXPANDS when immersed in water & become DISTORTED when dried out??? T/F A= True 54. Heat (accelerator) decomposes what into free radicals which initiate polymerization of MMA to PMMA? A = benzoyl peroxide (initiator) 55. Porcelain veneer can be given In High Caries patient???? T/F A= False 56. Gold or Porcelain, which one is more preferred for bruxit patient ? A= Gold 57. Portion of pontic approximating ridge should be as convex as possible? True 58. Which type of pontic is best for esthetics? A= modified Ridge Lap 59. Which pontic are used in case of the concave ridges? A= Ovate pontic 60. Is it fine if the pontic contacts during non- working movement ? No, it should not 61. Pontic should be convex MD and concave FL? T/F A= True 62. What displaces gases & removes corrosion products by combining w/ them or reducing them? A= Flux (Borax) 63. Whats is the Ideal C:R Ratio for FPD? A= 1:2 64. In Case of mobile teeth in an aged patient splint natural teeth & implants in a FPD? T/F A= No…… As the Implant has no PDL 65. Which is the most critical characteristic which is matched first in case of porcelain ? A= Value (Brightness) 66. Which stain is most often used to change the Hue (shade) A= Orange 67. Labial Bar should be 3 or 5 mm below the ginivigal margin ? A = 3 68. In case of lower RPD which requires lessere gigival height ??? Ligual Bar or Lingual Plate? A= Linual Plate 69. If vestibuler is less than 5 mm than which is more preferred Lingual plate of lingual bar ? A= Lingual Plate 70. Indirect Retainers prevernts the horizontal dislodgement of the Ditsal extentions base? T/F A= false= Prevents Horizontal displacement 71. Which retainer are the most esthetic in RPD? Intra or Extracoronal ? A= Intra 72. If patient has missing 4 teeth except mandibular incisor which one you will prefer? RPD or FPD ? = RPD 73. If denture falls when smiling, buccal notch& flange underextended ? False = Overextended 74. Most effective time to check phonetic ? Wax Try In 75. Average interocclusal space at rest ? = 3 mm 76. ExcessiveVDO= dec reeway space; DecreasedVDO=incr freewayspace??? T 77. Best impressurion technique for pt w/loose hyperplastic tissueis to register tissue in which position ??? Active or Passive ?? = Passive 78. What the role of Antifulx ? A =Restricts flow of solder soft graphite pencil. 79. Gypsum + Water= Endo or Exothermic ??? = Exothermic 80. MasseterMuscle–contracts or relaxes during swallowing? =Contracts 81. Which is not Determinants of Occlusion: 1)TMJ, 2)Occlusal Surface of teeth 3)Neuromuscular System 4) CO ?? = CO( Centric occlusion) 82. Who have more palatal tori ??? male or female?? = Female 83. Diabetes majorly impairs which cells ? RBC? WBC? PLATELT ? = WBC My test questions 072017 – UMAIR SEYAL 1. Which immunoglobulin is concentrated in gingival clevicular fluid: IgG 2. Which of the following does NOT present itself in the form of Macroglossia: Hypoparathyroidism / hyperparathyroidism – hyperthyroidism. Causes of Macroglossia: Congenital hyperplasia/hypertrophy. Tumors—lymphangioma, vascular malformation, neurofibroma, multiple granular cell tumors, salivary gland tumors . Endocrine abnormality. Acromegaly, cretinism. Infections obstructing lymphatics. Beckwith- Wiedemann syndrome. Macroglossia, exophthalmos, gigantism. Amyloidosis HYPOTHYROIDISM: orofacial findings include facial myxedema, an enlarged tongue (macroglossia), compromised periodontal health. Amyloidosis, down syndrome, Beckwith – Wiedeman syndrome, cretinism (hypothyroidism) 3. Which of the following is not associated with Melkersson-Rosenthal Syndrome? A. B. Granulematous cheilitis C. Macroglossia D. Facial paralysis *** Classical triad of syndrome include Fissured tongue, Granulematous cheilitis and Facial paralysis! 4. Working interference question: BULL rule – straight up asking about which cusps occlude where in working interference. 5. Mandibular canal is on lingual of mandibular 3rd molar, by moving the x-ray sensor inferiorly and x-ray direction superiorly which way will the canal appear to move: A) Apical, B) Mesial, C) Distal, D) Coronal…….I picked A because SLOB rule 6. Dentist makes an xray of pt 3 molar at 0 degree vertical angulation that looks like the superior border of mandibular canal contacts the apices of tooth. Dentist then makes another xray at -2 degree angulation that suggest that mandibular canal is separated from apices of 3 molar by several mm.Using information of these 2 xrays which is true about relation of mandibular canal to the root apices of 3 molar? Mandibular canal is: 1. superior and facial 2. superior and lingual 3. in contact with root apices 4. inferior and facial 5. inferior and lingual 7. Informed consent can have all of the following EXCEPT: A) Informed consent must be presented in advance of the treatment. B) Informed consent must contain treatment options. C) Informed consent must be in written form. D) Informed consent must contain risks and benefits of the treatment…..Weird right? I picked C – Consent may be given in either of two formats: express and implied. 8. Which of the following are required informational elements for informed consent? (Choose all that apply.) A. Explanation of the procedure in understandable terms B. Reasons for the procedure and the benefits and risks of the procedure and anticipated outcome C. Any alternatives and their risks and benefits, including no treatment at all D. The costs of the procedure and the alternatives 9. Which of the following shows the best way of active listening? A) Rephrasing the listener’s understanding of speaker’s communication. B) Active eye contact. C) By sounding listener’s concern 10. Patient complains, “Why do I have to stay here for so long for you to do this, why can’t you finish it already?” A) Because that’s how treatment works you idiot. B) That’s how long it takes to provide quality care. C) It seems like you’re upset, may be we can reschedule you for another day for longer appointment. D) It seems you are upset, what are your concerns about the procedure we’re doing today? 11. Question on Type – I error, gave the test result value of 0.01 and the researchers rejected the null hypothesis, what kind of error? Type – I / Provability of rejecting Ho when, in fact, it is true. Accepting when it’s false is Type II. The possible values of the probability of a type 1 error range from 1% to 0.1%. If the study is very concerned with making a type 1 error, a lower value is used (0.01) If the observed probability is less than or equal to .05 (5%), the null hypothesis is rejected (i.e., the observed outcome is judged to be incompatible with the notion of “no difference” or “no effect”), and the alternative hypothesis is adopted. In this case, the results are said to be “statistically significant.” If the observed probability is greater than 0.05 (5%), the decision is to accept the null hypothesis, and the results are called “not statistically significant” or simply NS—the notation often used in tables. 12. Patient says, “My teeth hurt when I brush them”, what is an appropriate response? don’t remember the options but it was easy one, something along the lines of oral hygiene instructions etc. 13. Adolescent have trouble following OHI at home after getting braces. What is the most effective way to make sure they follow the cleaning regimen? A) Ask the parents to supervise them. B) Educate them about oral hygiene. C) Give them limited praise with good progress at each appointment. 14. Porcelain porosity: because of Inadequate condensation 15. What is the most common process by which the porosity of porcelain is reduced? 1- condensation 2- sintering-sintered 16. Amalgam failure: Water contamination 17. What do we write the consult for: A) To gain certain information B) To gain clearance C) To have a better relationship with patient’s physician, of course , because why not -_- Much is written on communicating with the patient to discover why they have consulted and to gain information about their illness. 18. Wheelchair bullshit was there too –Determine the patient’s needs, prepare the dental operatory, prepare the wheelchair, perform the two-person transfer, position the patient after the transfer, transfer from the dental chair to the wheelchair (use sliding, it means every dental office should have this sliding passage for weel chair pt.) 19. What is the best method to transfer the dental "wheelchair" patient? sliding 20. Child starts throwing fits: Voice control 21. Mouth wash for disabled child? Naf or Chx or listerine? NaF 22. Disabled kid, best measure: Consistency 23. Unstimulated Salivary flow rate in an adult: A) 1L/min B) 0.1ml/ min C) 1L/min D) 10L/min Unstimulated 0.1 - 0.5 ml/min - Stimulated >1ml/min (DD) 24. Stimulated Salivary flow rate in an adult: A) 10L/day B) 1L/day C) 0.5ml/day 25. Fluoride ppm in community water, average value: 1.0 26. Fluoride next to tooth: 4 minutes 27. Implant to implant: 3mm 28. Best Amalgam: High copper admix & spherical – smaller particle size results in higher strength, lower flow, and better carvability. Spherical amalgams high in cooper usually have the best tensile and compressive characteristics. 29. Copper contents over 6% (high copper alloys) eliminate the gamma-two phase by forming a copper-tin phase resulting in superior properties. 30. Amalgam has a coefficient of thermal expansion approximately twice that of tooth structure. 31. The tensile strength of amalgam is about one-fifth (1/5) to one-eight (1/8) of its compressive strength. 32. Most common gland involved in salivary gland tumors: Parotid 33. Middle-aged male has a fluctuant mass in the midline of neck: A) Thyroglossal duct cyst B) Brachial cleft cyst. 34. Question about what do you need for caries: Bacteria, supporting carbs and a susceptible tooth 35. What else do S. mutans produce along with dextran after breaking down sucrose: A) mucopolysaccharides B) macros C) levans D) proteins 36. Most common TMJ ankyloses caused by: A) Trauma B) Rheumatic arthritis C) random shit 37. Histo differentiation involves which of the following: Amelogenesis and dentinogenesis 38. Blue sclera: Osteogenesis imperfecta 39. Cleidocranial dysplasia characteristic: supernumerary teeth 40. Ectodermal dysplasia: oligodontia – anodontia or oligodontia (partial anodontia) 41. Ectodermal dysplasia: scarce hair (Atrophic skin, defective hair, partial anodontia, & hypoplastic sweat glands.) 42. Patient smokes pipe and has red bumps on palate: Nicotine stomatitis 43. Pseudoepithilomatous hyperplasia a characteristic of: A) SCC B) Verrucous C) Pemphigus – Granular Cell Myoblastoma, The differential diagnosis of squamous cell carcinoma (usual type) mainly includes pseudoepitheliomatous hyperplasia. 44. 85yr old elderly patient’s son brings the consent form that has name of the legal guardian, what needs to be done before treatment? Contact the legal guardian who has the power of attorney to get consent of treatment 45. Nerve involved in Bell’s palsy: VII – facial nerve 46. 4mm implant, how much do you need buccolingually: 6mm (1mm on each side) 47. Mandibular 3rd molar root lost: submandibular space 48. IAN block needle infection where: Pharyngeal, Pterygoid, medial pterygoid muscle - pterygomandibular space, PSA its pterygoid plexsus. Masticator space infections are almost always of dental origin, especially from the mandibular molar region . *Needle tract infections after and IA block initially involve the pterygomandibular space. 49. Arch discrepancy after loss of which tooth: Mand 2nd molar The MOST RAPID LOSSES IN ARCH PERIMETER are usually due to a MESIAL TIPPING & ROTATION of the permanent first molar after removal of the primary second molar. When the primary second molar is lost, ALWAYS MAINTAIN SPACE until the second premolar arrives.

50. Which of the following is clinical sign of Leukemia: Bleeding from gums, pale conjunctiva, fever 51. The research concludes that patients who use chlorhexidine have better oral health than those who do not, however, other researchers say there is not much difference in oral environment of those who use chlorhexidine or not? Paraphrased the question but you get the jest of it……Double Blind 52. High school kids have: Marginal gingivitis 53. Cardiac arrest in children: Respiratory depression 54. Veneer facial reduction – 0.5mm 55. PFM buccal margin depth – 1.3-1.5mm was the only option that made sense 56. Patient comes back after 1 year of composite restoration with pain and sensitivity: I picked Microleakage 57. Kid has occlusal caries on posterior molar which material will you use to restore: I picked Amalgam because it’s better than Composite resin, however, Resin saves more tooth structure sooooooo whatever floats your boat I guess. 58. Endo and Perio diff: Pulp vitality test 59. Endo and Perio tx: Endo first followed by Perio 60. Acromegaly causes: Excessive growth of mandible 61. Radiograph to check integrity of Zygomatic arch: CT – Do not pick Lateral cephalometric (Submentovertex for fracture the best, but CT always better) 62. TMJ radiograph: MRI 63. MRI us used to view: disc of TMJ 64. Bechet’s syndrome associated with: Herpes simplex, Aphthous ulcers, Leukemia – Recurrent herpetiform consists of clusters and ulcers. Patients with frequent recurrences should be screened for diabetes mellitus or Bechet's Syndrome. 65. Mechanical damage to teeth: Abfraction 66. Chemical damage to teeth: Erosion 67. Bradycardia treatment: Atropine - Atropine is the first drug used to treat bradycardia in the bradycardia algorithm. 68. Which is NOT used to inhibit salivary secretion: Pilocarpine - In dentistry, cholinergics drug treat dry mouth (Xerostomia) by inducing salivation. Cholinergic drugs used are: Pilocarpine (Salagen)- a cholinergic agonist and alkaloid indicated to treat xerostomia caused by salivary gland hypofunction caused by radiotherapy for head and neck cancer by stimulating salivary flow. Common side effects: excess sweating, nausea, heartburn, and diarrhea due to the drug's cholinergic nature. Cevimeline (Evoxac) - a cholinergic agonist indicated to treat xerostomia in patients with Sjogren's Syndrome. Common side effects: increased sweating, nausea, heartburn, diarrhea due to the drug's cholinergic nature. Specific for the M3 receptor on the salivary glands. 69. Which is contraindicated in nitrous: nasal congestion 70. Target lesions: Erythema multiform 71. Radiograph to identify: the soft tissue tip of nose along the root tips of centrals 72. Radiograph to identify: external auditory meatus 73. Which of the following confirms the diagnosis of xerostomia: A) location of probing depths of >4mm. B) location of anterior restorations. (I picked this instead of A because chemotherapy causes xerostomia which leads to class V lesions) C) location of partial denture flange 74. Over titration of Amalgam leads to: reduced working time etc etc - the longer the trituration time the more streinght. 75. Buccal limit of mandibular denture: Masseter 76. Lingual limit of mandibular denture: mylohyoid, genioglossus, palatoglossal and superior constrictor. 77. Case of mandibular denture and question about the efficient way of increasing retention of the denture, this patient had partial denture with only molars and she was tight on money so I picked extending the buccal shelves into the vestibules a bit more because the denture flanges looked pretty short of the vestibule on both sides. There was an option for implant too but I did not go for it due to patient’s financial concern. Your choice really! 78. Cavernous thrombosis - infection via anterior triangle?? Canine space infections and deep temporal space infections can result in thrombosis via the ophthalmic veins. (Mosby) 79. To anesthetize anterior teeth, which other injection would you have to administer on top of nasopalatine nerve block? ASA nerve block 80. Tooth mostly involved in perio relapse: Max 1st molar due to trifurcation 81. Maxillary first molar is the tooth most likely to benefit from occlusal sealant placement. 82. One week after cementation of an MOD onlay on a maxillary molar adjacent to an existing amalgam, the patient reports sensitivity to cold and pressure of the tooth. The most likely cause is hyperocclusion. 83. Premolar with 3 roots: Max 1st 84. Which of the following is NOT true about vertical root fracture: a) It is common occurrence in post and core teeth only. b) Extraction of the tooth is usually the only treatment for it – there was a third option but from what I recall it was a true statement so I went with this. 85. What’s the status of pulp when the pain goes away quickly following the removal of stimulus: a) normal b) reversible pulpitis c) necrotic d) irreversible pulpitis 86. Fluoride form for kids under 3yrs of age: drops 87. Best place for implant: lower anterior 88. Warfarin and Coumadin test: a) PTT b) Prothrombin to thrombin c) Tissue factor 8 d) international normalized ratio 89. Oral hairy leukoplakia: filiform 90. Best solution to keep avulsed tooth in: Hank’s or whatever it’s called - HBSS - Hank's Balanced Salt Solution. 91. Treatment for flared out front teeth of 7yr old: No treatment because of ugly duckling phase 92. Picture of flared out anteriors asking Class: Class II division I 93. Opioids affect: Chemotactic center - Opiates have been demonstrated to reduce chemotaxis, phagocytosis, and the production of cytokines and chemokines. 94. Alkylating anticancer drug’s side effect: a) nephrotoxicity b) uric acid retention c) bone marrow suppression – I thought bone suppression was the side effect of non- alkylating anticancer drugs yet I ended up picking this just because I felt like it. - an alkylating agent irreversibly inactivates cellular nucleic acids (DNA) and proteins. A chemotherapeutic drug (Alkylating Agent) with adverse effects of nausea and vomiting (75%-100%; dose-related), alopecia, xerostomia, and changes within the oral cavity tissues (i .e. mucositis). • Mucosititis - a common reaction to cancer chemotherapy involving inflammation of the mucous membranes. During chemotherapy and radiation therapy, mucosal tissues begin to desquamate and ulcerate. The mucosal integrity is broken and is secondarily infected by oral flora. Palliative treatment is indicated for mucosititis. 95. Ginseng contraindicated for: salicyclic acid 96. Side effect of Zoloft: salivary hypofunction?? Selective Sserotonine Reuptake Inhibitors (SSRIS): Fluoxetine (Prozac), Paroxetine (Paxil), Sertraline (Zoloft), Fluvoxamine (Luvox), Citalopram (Celexa) & Escitalopram (Lexapro). Adverse Effects: nausea, headache, anxiety, agitation, insomnia, and sexual dysfunction. SSRls do not have secondary anti-cholinergic effects, thus do not cause any significant dry mouth. Selective Serotonin Reuptake Inhibitors (i.e. Prozac) do not have an effect on NE in tissues and interaction with a vasoconstrictor like EPI is not a problem. 97. Pseudomembranous colitis: Clindamycin usage 98. Biotransformation definition - The interaction between a drug and the living organism in which the body brings about a chemical change in the drug. Biotransformation (or drug metabolism) Process of converting a drug into one or more metabolites. 99. First Pass Effect - Enteral Administration (via the intestine or GI tract): Oral route is most known for its significant hepatic "FIRST PASS" metabolism. Oral Route Disadvantage: drugs must be absorbed (usually from the small intestine) before they can be transported to their site of action. Blood from the intestinal tract passes first to the liver (some drugs are metabolized in the liver "first-pass effect", while others may be stored there to be released slowly). This consideration makes it clear that oral administration is not suitable in emergencies or other occasions when a rapid effect is needed. Emotional stress decreases the rate of absorption of a drug when given orally. 100. Therapeutic effect: Safety 101. Which is False about medications taken parenterally (by injection)? They are very reliable since you know exactly how much effect gastrointestinal system has on the ingested meds. 102. Lack of indirect retainer: will result in denture being lifted away from tissue when pressed on one side 103. Tuberosity hitting retromolar pad: surgery on tuberosity 104. Why not use fovea as indicator for posterior limit: because you don’t want to block the minor salivary glands since fovea represents openings to those minor glands 105. When to remove the palatal torus: when it covers posterior palatal seal 106. Patient has a palatal torus that extends beyond posterior palatal seal into the soft palatal area. What will be the best course of action: Use horseshoe design to avoid the torus 107. Most common gland for sialolith: Submandibular 108. Most common salivary gland tumor: Pleomorphic 109. Gingival graft contraindicated when: a) pocket is below the alveolar crest b) pocket is below free gingival groove c) excessive keratinized tissue 110. Patient swallowed a crown where is it most likely for it to end up: a) left bronchus b) right bronchus c) trachea d) paranasal sinus 111. Picture to identify Fluorosis 112. Graft between same species but genetically different individual: Allograft 113. Anesthetic that is vasoconstrictor: Cocaine 114. Cross allergy for anesthetics: know esters and amides - For patients allergic to ester and amide local anesthetics, DIPHENHYDRAMINE is a safe and effective alternative. Lidocaine and Mepivacaine are most likely to show cross-allergy. 115. Which anesthetic is good without vasoconstrictor: Mepvicaine 116. Patient does not have tooth #11 and has all the premolars, which one has the Worst Prognosis: A) fixed bridge from #10-12. B) RPD with pontic for #11 – I picked this, however, I think I should have picked A, your call. C) Implants 117. Rifampin used for: tuberculosis 118. Penicillin moa: transpeptidase – Inhibits Transpeptidase, stage 3 in cell wall synthesis 119. Manic depression: lithium - Antimania drugs are used to treat manic-depressive illness. A. Drugs: 1. Lithium. 2. Carbamazepine. 3. Valproic acid. 4. Lamotrigine. B. Mechanisms of action. 1. Lithium works inside the cell to block conversion of inositol phosphate to inositol. 120. Parkinson’s: lack of dopamine – In Parkinson's disease, nerve cells in the BASAL GANGLIA DEGENERATE, causing decreased dopamine production. 121. Lidocaine: Ventricular arrhythmia - Ventricular arrhythmias can be treated by an intravenous injection of lidocaine 122. Lidocaine calculation question 123. Oral contraceptives failure: Rifampin - Rifampin lowers the effectiveness by decreasing the birth control hormone levels (ethinyl estradiol and progestin) in women taking oral contraceptives. 124. Aplastic Anemia: Chloramphenicol - CHLORAMPHENICOL (CHLOROMYCETIN) - a broad-spectrum antibiotic that can cause BONE MARROW DISTURBANCES (aplastic anemia) thus, has LIMITED USE due to its side effects. 125. Bone penetration, which drug: Clindamycin – For endodontic infections that do not respond to penicillin, clindamycin is recommended as it produces high bone levels, and is effective against anaerobic bacteria. 126. Grand mal seizure: Phenytoin - Phenytoin (Oilantin)-treats tonic-clonic (grand mal) seizures. Also, Carbamazepine (Tegretol) = Trigeminal neuralgia and tonic-clonic seizures (grand mal) 127. ADA recognizes as dental specialty: dental public health 128. Informed consent: Autonomy 129. Child has signs of abuse, you try to question the mother but she seems reluctant. What do you try to pay attention to while talking to her? Answer was something like pay attention to micro-expressions, you’ll know when you’ll see it. 130. Improvement after Endo treatment is most likey after: 1 year 131. Telling patient about risks and benefits of a treatment is a part of: Autonomy

Avneet G Aulakh’s RQs JULY 2017

3. Primary tooth has the most effect on space loss: Upper canine, Upper first molar, Upper second molar, Lower first molar, Lower second molar 4. Disadvantage of widman flap and know the procedure of widman flap

Horizontal incisions for full-thickness flaps— three horizontal incisions are usually associated with a full-thickness flap design. (1) The first is the internal bevel incision - depending on the goal, this incision can be made 0.5 to 1 mm from the free gingival margin (apically displaced flap), 1 to 2 mm from the free gingival margin (modifed Widman flap), or just coronal to the base of the pocket (undisplaced ap). It also is known as the reverse bevel incision. This incision removes the pocket lining, conserves the outer dimension of the gingiva, and produces a thin sharp flap margin that can be adapted to the bone - tooth junction. (2) The second is the crevicular incision — made from the base of the pocket to the crest of the alveolar bone. The combination of the internal bevel and crevicular incisions creates a collar of tissue around the teeth. (3) The third is the interdental incision —this inci- sion separates the collar of gingiva from the tooth. Reflection of the flap after placement of these three incisions allows for visualization of the alveolar bone. The modified Widman flap uses the three horizontal incisions described previously but is not reflected beyond the mucogingival line. This flap design allows for removal of the pocket lining and exposure of the tooth roots and alveolar bone but does not allow for apical repositioning of the flap. 5. Modified Widman flap is a replaced flap. T/F - A replaced flap (also called repositioned flap and modified Widman flap) is one that is repositioned in or near its original location 6. Which of the following is the best for alveolar bone and root debridement? 1) mucoperiosteal flaps 2) modified Widman flap 3) partial thickness flap Three incisions are made in the modified Widman flap — internal bevel, crevicular, and interdental. It is designed to provide exposure of the tooth roots and alveolar bone. 7. Modified widman flap is an undisplaced flap? T/F - THE UNDISPLACED FLAP Ø Most commonly performed type of periodontal surgery. Ø It differs from the modified Widman flap in that the soft tissue pocket wall is removed with the initial incision; thus it considered an internal bevel gingivectomy. 8. Which graft uses the three horizontal incisions but is not reflected beyond the mucogingival line? A)Undisplaced flap B)Apically displaced flap C)Modified Widman flap D)Pedicle graft 9. Which would eliminate pockets? a) modified widman flap b) apically positioned flap c) undisplaced flap 10. Which of the following flap techniques is used to eliminate or reduce pocket depth? select all that apply 1- modified widman flap 2- undisplaced flap 3- apically displaced flap - MWF - remove pocket lining not pocket depth, it says used in order to simplify instrumentation and removal of the lining, the reduction in pocket depth is due to the healing shrinkage. MWF: Facilitates instrumentation but does not attempt to reduce pocket depth. The modified Widman flap (MWF) facilitates instrumentation but does not attempt to reduce pocket depth. The reduction or elimination of pocket depth is the main purpose of two flap techniques: the undisplaced flap and the apically displaced flap. The decision of which to perform depends on two important anatomic landmarks: • Pocket depth • Location of the mucogingival junction *** These landmarks establish the presence and width of the attached gingiva, which is the basis for the decision. The modified widman flap has been described for exposing the root surfaces for meticulous instrumentation and for removal of the pocket lining. This flap uses the three horizontal incisions but is not reflected beyond the mucogingival line. Note: It is not intended to eliminate or reduce pocket depth, except for the reduction that occurs in healing by tissue shrinkage. The undisplaced (unrepositioned) flap, in addition to improving accessibility for instrumentation, removes the pocket wall, thereby reducing or eliminating the pocket. This is essentially an excisional procedure of the gingiva. Note: Currently, the undisplaced flap may be the most frequently performed type of periodontal surgery. It differs from the modified Widman flap in that the soft tissue pocket wall is removed with the initial incision; thus it may be considered an internal bevel gingivectomy." The undisplaced flap and the gingivectomy are the two techniques that surgically remove the pocket wall. The apically displaced flap also improves accessibility and eliminates the pocket, but it does the latter by apically positioning the soft tissue wall of the pocket. Therefore, it preserves or increases the width of the attached gingiva by transforming the previously unattached keratinized pocket wall into attached tissue. Remember reduction or elimination of the pocket DEPTH: apically and undisplaced, and the 2 techniques to remove the pocket WALL is undisplaced and gingivectomy. dont get confuse. 11. Bacteria in red complex: P. gingivalis, T. forsythensis, T. denticola 12. Predominant cells in GCF: GINGIVAL CREVICULAR FLUID (GCF) - in health, GCF is a transudate that emerges from the gingival sulcus. GCF may contain a variety of enzymes and cells, particularly desquamating epithelium & neutrophils (PMN) being shed through the sulcus. An increase in GCF flow is the first detectable sign of inflammation. Once inflammation has occurred, GCF is called inflammatory exudate which contains a higher level of serum proteins and leukocytes. 13. There were 3 ques regarding composite like composite is intact but discolored what will u do: In our study, 60.5% of the composite fillings with brown discolorations at the margin were caries free. That means, if there are discolorations of the margins of composite fillings without any evidence of decay, the filling does not need to be removed completely. 14. You have a class 3 composite that is fully functional, margins intact, just some discoloration at margins... what do you do? 1- replace it 2-cut 1mm and fill 3-just polishing 15. Which of the following causing discoloration along the gingival margin of a PFM crown? A) Copper B) silver - Gingival - copper - Anywhere else in the crown- silver 16. Whats the difference between , cemento-osseous dysplasia and ? Cementoma = cemento osseus dysplasia = periapical cemental dysplasia , the lesion is within the bone , osseus deposition , it is a reactive lesion rather than neoplastic, response of periapical bone to some local factors (trauma from occlusion), two or more teeth affected. - Cementoblastoma = true cementoma, is cemental deposition attached to the tooth most common in the mandible posterior, neoplasm, always radiopaque, one tooth. - Radiographically: cementoblastoma: mand posterior teeth , integerated with the root of the tooth , can not differentiate the root from the lesion . Single tooth. Always opaque. To distinguish cementoblastoma forom condensing osteitis (CO) – in CO you can distinguish the root outline - Periapical cemento dysplasia (cemento osseous dysplasia): mand anterior teeth, the lesion is close to the root. 2 or more lesions must be there. The lesion has 3 stages 1) radioleucent 2) radiopaque pits 3) completly radiopaque. 17. Cemento osseous dysplasia ... more in ant mand middle age African females ... You have to choose exception - Clinical Features: occurs at the apex of vital anterior teeth, affecting women over age 30yrs (especially BLACK women) more than men. Asymptomatic, usually multiple, small periapical areas of radiolucency in the mandibular incisor area. Depending on the stage, a cementoma may appear radiolucent, mixed radiolucent & radiopaque, or completely radiopaque. 18. Cleft palate, mand hypoplasia and tongue obstruction - PIERRE-ROBIN SYNDROME - an inherited disorder with the following findings in the NEONATE: Micrognathia- smallness of the jaws. Glossoptosis - downward displacement or retracted tongue. Breathing problems & Cleft Palate. PIERRE ROBIN SYNDROME - a hereditary disorder that presents micrognathia (smallness of the jaws), glossoptosis (downward displacement or retraction of the tongue), & a high-arched or cleft palate. Most children require orthodontics. 19. Conditions associated with multiple supernumerary teeth: Gardener's syndrome, Down's syndrome, Cleidocranial dysplasia, & Sturge-Weber Syndrome. 20. Polyps is seen is which of the following condition? A) Garden syndrome. B) Petuz jegher. C) Crohn’s. D) All of the above. 21. Which is related to taurodontism 1. cleidocranial dysplasia 2. gardener syndrome 3. downs syndrome 4. amelogenesis imperfect, ONLY TYPE IV 22. Multiple Osteomas of jaw are seen in: a. Gorlin Goltz syndrome b. Peutz Jeghers c. gardener syndrome d. cleidocranial dysplasia - Gardner's...in A we would see multiple OKCs 23. Cafe ul lait is seen in: A. gorlin syndrome B. gardeners syndrome C. PJS (peutz jeghers syndrome) D all 24. Gardener syndrome: Clinical - osteomas, odontomas, fibromas, epidermal cysts, supernumerary/ impacted teeth, intestinal polyposis (very serious complication) - R/g --- multiple osteomas which give COTTON WOOL APPEARANCE. 25. What is common in gardener syndrome and cleidocranial syndrome?? 1. intestinal polyps 2. intraoral pigmentation 3. impacted supernumerary teeth 4. osteoma of skull and jaw 26. Neurofibromatosus - for syndromes they gave u option u have to recognize - VON RECKLINGHAUSEN'S DISEASE (NEUROFIBROMATOSIS) - the most outstanding feature is NEUROFIBROMATOSIS (condition of multiple tumors of nerve tissue origin). VRD is a relatively common inherited autosomal dominant trait characterized by multiple neurofibromas, cutaneous cafe-au-Iait macules, bone abnormalities, & CNS changes. Clinical Signs: presence of 6 or more cafe-au-Iait macules > 1.5cm in diameter indicates VRD unless proven otherwise. Treatment: there is no satisfactory treatment. The lesions run a high-risk of transforming into a malignancy. A single neurofibroma presents at any age as a non-inflamed, asymptomatic nodule that occurs on the tongue, buccal mucosa, & vestibule. This single nodule is removed by surgical excision, and rarely occurs. Cafe au lait spots, axillary freckling, lisch spots (iris spots). 27. Pt (young child) w/nodules on right side of tongue that are fluid filled the rest of the mouth is WNL(within normal limits)no other systemic signs. A neurofibromatosis B Lymphangioma C Granular cell tumour 28. Café au lait is seen in all except which one? a) Fibrous Dysplasia b) Neurofibromatosis Type 1 c) Melkerrson-Rosenthal syndrome d) McCune Albright Syndrome 29. There was picture with multiple periapical radiolucencies - for answered cemebto osseous dysplasia may be the Florida one. Periapical Cemental Dysplasia (periapical cement – osseous dysplasia): The most common presentation is a middle age female with multiple periapical radiolucencies in relation to lower anterior teeth. 30. What make penicillin allergic: beta lactam ring, which causes the allergy. Sometimes, patients with penicillin allergy produce the IgE antibody to the side chain of the drug and not to the beta-lactam ring 31. Montelukast (Singulair) is a bronchodilator used for asthmatic attach by which mechanism of action: A. Histamine competitive antagonism B. Beta 2 adrenergic agonism C. Baroreceptor reflex D. Leukotrienes receptor antagonism – antagonizes leukotriene receptors thus dreasing bronchoconstriction and inflammation 32. Antidepressants serotonin: Tricyclic antidepressants are generally considered to be the drugs of first choice for treatment of depression. These drugs inhibit the neuronal reuptake of NE and SEROTONIN in the brain. It inhibits the reuptake that means antidepressants make serotonin stay in brain for a longer time. 33. Selective serotonin re-uptake inhibitor drug with the longest half-life (SSRI)? Fluoxetine (Prozac) 34. Which two groups of antidepressant drugs has the highest incidence of dry mouth? 1)Tricyclic antidepressant 2) selective serotonin reuptake inhibitors 3)serotonin and norepinephrine reuptake inhibitors 4)MAO inhibitors – DD#56 These two categories of antidepressant drugs induce significant dry mouth in up to 75% of patients taking these medications. These effects are due to the secondary anticholinergic nature of these agents. 35. Girl with gingival bleeding and recurrent infection - leukemia - Acute lymphocytic (lymphoblastic) leukemia-largely confined to children (it is the most common leukemia in children. Lymph node enlargement is common. In 75% of cases, the lymphocytes are neither B nor T cells, but are called "null" cells. Bone and joint pain are common in children. 36. Least recurrence options were AOT, odontogenic myxoma, ameloblastoma, okc - The AOT (ADENOMATOID ODONTOGENIC TUMOR) is a benign epithelial tumor with a dense fibrous connective tissue capsule, which does not recur once removed. 37. Which of the following isn’t developmental: a) OKC b)AOT c)dentigerous cyst d)residual cyst 38. Questiom regarding dentist and placebo I had no idea .. 39. Dentures major connector function: Major connectors: The function of the major connector is to connect all the RDP components of one side of the arch with the opposite side to unite them. Provides stability to resist displacement while in function. Major connector should be rigid and not be placed on movable tissue. Undercut areas and soft and bony prominences (e.g., tori, median palatal suture) should be avoided, removed, or relieved, depending on the severity. Relief should be provided to prevent tissue impingement secondary to distal extension denture rotation. 40. RESTS - the primary purpose of any rest (occlusal, cingulum, or incisal) is to provide VERTICAL SUPPORT for the RPD.

41. A minor connector is a rigid component that connects the major connector or base with other components of the partial denture such as rests, indirect retainers, and clasps. 42. Lingual Bar-more popular than a labial bar. A lingual bar is placed so its upper border is at least 4mm below the gingival margins. When severely tipped premolars and molars are present, an alternate framework design or crowns are recommended. Lingually inclined mandibular premolars interfere most frequently with mandibular major connectors. 43. Lingual Plate- a lingual bar that has been extended upward to cover the cingula and interproximal spaces between mandibular anterior teeth. It should be thin and follow the contours of the teeth and embrasures. The upper border should be located at the middle 1/3 of the lingual surface of the teeth and extend upward to cover interproximal spaces to the contact point. Severe anterior crowding is a contraindication for using a linguoplate. 44. ADA classifies alloys as follows: Type I: used for small inlays. Type II: larger inlays &onlays. Type III:onlays, crowns, and short-span FPDs. Type IV: thin veneer crowns, long-span FPDs &RPDs. 45. Where to place rest: occlusal, cingulum (lingual), or incisal 46. What happen if temp of developing solution is too high: reduces development time. Set timer—typically 5 minutes at 68° F. shows effect of temperature on development time. Developers (1) Phenidone is as the first electron donor that reduces silver ions to metallic silver at the latent image site. (2) Hydroquinone provides an electron to reduce oxidized phenidone back to its original active state so that it can continue to reduce silver halide grains to metallic silver. 47. 8yr old boy ..crowding in incisors ...canines r yet to erupt ..what to do ?? extraction of primary canines ..stripping ..place lingual Arch and observe Space maintenance (in cases where primary teeth have been lost and space is otherwise adequate). a. Band and loop. b.Distal shoe (before eruption of a permanent molar). c. Lingual arch. d. Nance appliance (maxillary arch). Space regaining (localized space loss)— indicated when space loss is minor (<3 mm): Removable appliance with finger springs to tip teeth distally, Headgear (for the maxillary arch), Activated lingual arch (for the mandibular arch), Lip bumper (for the mandibular arch), Limited fixed appliances: Followed by placement of a space maintainer after space is regained. Moderate crowding (<4mm): arch expansion (this is a controversial topic), extraction of primary canines: Borrows space until permanent teeth erupt. Lingual arch necessary if mandibular primary canines are extracted because the permanent incisors will upright lingually and space will be lost. Severe crowding (>4mm): serial extraction – usually reserved for large space discrepancies (>10mm per arch) 48. Sequence of extractions. (a) Extraction of primary incisors, if necessary. (b) Extraction of primary canines to allow permanent incisors to erupt and align. c) Extraction of primary 1rst molars to encourage eruption of the permanent 1rst premolar (ideally, before the permanent canine erupts). (d) Extraction of permanent 1rst premolars to allow the permanent canine to erupt and align. 49. Best way to gain max info about pain..options were tell me about ur past dental experience ....so this hurts u ....I answered 1 one on the basis v shouldn't ask direct ques 50. Spontaneous pain at night pt wake up..had lingering pain from cold from a week options were irreversible pulpits or pulp necrosia - NECROTIC PULP (PULP DEATH) - may have no painful symptoms and does not respond to EPT at any current level, but the tooth sometimes responds to heat, but will not respond to cold. A tooth affected with a necrotic pulp may have no painful symptoms and may appear discolored. EPT is valuable because there will be no response at any current level. Treatment: RCT or extraction. 51. Patient was having diastema ...in que they ask what is that radiolucencies between upper incisors .... Options were intermax suture inciaive canal and follicle of mesiodense: incisive foramen - A small ovoid or round radiolucent area located between the roots of the maxillary central incisors / superior foramina - Two small round radiolucencies located superior to the apices of the maxillary central incisors / median palatal suture - A thin radiolucent line between the maxillary central incisors 52. There was questions on impression material regarding their hydrophobic and phillic....I don't remember exactly 53. Inc water powder ratio in gypsum does what: Water-Powder Ratio: this is an important factor in determining physical properties. When a high proportion of water is used, the powder particles are farther apart, resulting in less expansion with a retarded setting time and weaker product. Dental plaster requires 2x more water has a higher setting expansion than dental stone. • When mixing gypsum products always SPRINKLE the powder into the water to produce better powder mixing and to reduce air bubbles. • When gypsum products are mixed with water, heat is given off (exothermic reaction)• Exposure of a stone cast to tap water should be minimized to prevent eroding of the cast. Water Temperature: colder the water, the LONGER setting time. 54. Simple ques regarding pulpectomy 55. I had couple questions to differentiate between irreversible pulpits and pulp necrosis ...so learn their sign and symptoms well 56. Porcelain adheres to metal primarily by a CHEMICAL BOND. A covalent bond is established by sharing 02 with elements present in the porcelain (silicon dioxide (Si02) and metal alloy (oxidizing elements like silicon, indium, &iridium). 57. Resistance form in a cavity preparation is achieved by pulpal and gingival walls perpendicular to occlusal forces and proper angulation of cavity walls. 58. Which bur is not good for porcelain????? 59. Distance between implants 3mm, 1.5 implant – tooth, implant 4mm width.

60. Adv of implant over fpd 61. Less recurrent caries propioception there were two more options I forgot 62. 5mm intrusion of primary tooth - Unless it can be determined that the primary tooth is impinging on the permanent successor, intruded primary teeth are left alone in the hopes that they will spontaneously reerupt. Intruded permanent teeth have a poorer prognosis. If there is an open apex, an intruded permanent tooth should be closely monitored for spontaneous eruption. An intruded permanent tooth with a closed apex should be repositioned orthodontically, and a calcium hydroxide pulpectomy should be per- formed 2 weeks after the injury. 63. A 4-year-old had traumatic intrusion of tooth E, and 50% of the crown is visible. What is the treatment? A. Reposition, splint, and endodontics /react-text B. Reposition, splint, and formocresol pulpotomy /react-text C. Reposition and splint /react-text D. None of the above – Wait for spontaneous reeruption. DD SAYS FOR NATIONAL BOARDS ANSWER NO TREATMENT AND LET IT REERUPT 64. Sodium hypochlorite properties: Sodium hypochlorite (NaOCl). a. Indications. (1) Disinfection of root canal. (2) Dissolving organic matter — proteolytic material. (3) Does not remove smear layer. (4) Concentrations vary from 0.5% to 6%. 65. After crown on mand molar mandible gets deflected to working side ..They ask u to mention cusp and inclines of mandible involved 66. Best way to decrease fear of child. I said sit down to child's level - When treating a child who is obviously afraid, the dentist should permit the child to express his fear (identify the fear). All behavioral patterns are motivated by anger and fear. The crying child is NOT an abnormal child. Anger is easier to treat than fear. Fear is most likely exhibited by a young child on his first visit to the dentist, and is related to anxiety over being separated from a parent. The PARENT (not the dentist, has the greatest influence on the child's reaction at this initial visit. Child Control Techniques: Angry Child: Separate the parent and child, and place the child in the chair abruptly and be firm. Use "hand-over-mouth" technique (HOME) after getting the parent's permission! Display authority and command the child's respect by continuing with treatment even if he/she is uncooperative. Comfort the parent at the end of the visit, and compliment the child at the end of the visit. Fearful Child: • Have the parent stand quietly behind the chair. • Dentist must be consistent in tonal quality. • Allow the child to express his fears (identify the fear). • Change the child's focus off fear. • Lastly, sedation. Tell- Show-Do. 67. A S2-year old women requests extraction of a painful mandibular 2nd molar. She tells you she has not rested for two days and nights because of the pain. Her medical history is unremarkable, except that she takes 20mg of Prednisone daily for Erythema Multiforme. To treat this patient, you would give steroid supplementation and remove the tooth with local anesthesia and sedation. • The fear here is that the patient may not have sufficient adrenal cortex secretion (adrenal insufficiency) to withstand the stress of an extraction without taking additional steroids (this hold true for any patient treated for any disease with steroid therapy). • Erythema Multiforme- hypersensitivity syndrome characterized by polymorphous eruption and mucous membranes. Macules, papules, nodules, vesicles, or bullae and target ("bulls-eye-shaped") lesions are seen. A severe form of this condition is "Stevens-Johnson Syndrome". These patients may be receiving moderate doses of systemic corticosteroids, thus may be unable to withstand the stress of an extraction. Consultation with their physician is absolutely necessary before treating these patients. 68. Dentist asks a child u want me to help you to sit on the chair ...how to define this situation one option was perceived. 69. Helplessness - rest I don't remember 70. Tooth most involved in vrf: incomplete fracture – mandibular first molar. Horizontal root fractures occur more commonly in maxillary central incisors and result from the direct application of traumatic force to the face, alveolar processes, or teeth. In contrast, vertical fractures usually involve the molar teeth in adults. Mandibular molars and maxillary premolars are the teeth that most frequently have occult vertical root fractures. Max/mand PMs, mesial root of mandibular molars, mandibular incisors, mesio buccal root of maxillary molars. Demographic studies have shown that the most common teeth implicated in vertical fracture are mandibular first and second molars (42% of all fractures). 71. A tooth with a vertical root fracture has a POOR prognosis. Studies show most vertical root fractures are caused by using too much condensation force during obturation with gutta-percha. Vertical root fracture has a poor prognosis. 1. Occurs along the long axis of the tooth. 2. Often associated with a severe periodontal pocket in an otherwise periodontally sound dentition. 3. Can be associated with a sinus tract. 4. Can be associated with a lateral radiolucency extending to the apical portion of the root fracture. 5. A fracture can be identified only with visualization, and surgery is often necessary to confirm the fracture. C. How vertical fractures occur. Can occur after the cementation of a post. 2. Can be the sequela of excessive condensation forces during obturation of an underprepared or overpre- pared canal. a. Prevent fracture via appropriate canal preparation. b. Prevent fracture via balanced pressure of condensation forces during obturation. D. Treatment of vertical root fractures. 1. Removal of the involved root in multirooted teeth or extraction. Results in extraction of single-rooted teeth. 72. Most important in prognosis of tooth ...clinical attachment - The MOST critical factor to determine if a tooth should be extracted or have surgery (prognosis) is AMOUNT OF ATTACHMENT LOSS (the amount of apical migration of the epithelium attachment). Amount of attachment loss is the most important factor in the determination of a prognosis of a tooth with periodontal disease (more accurate than probing depth, tooth mobility, and presence of furcation involvement). 73. Gingivectomy CI and technique: GINGIVECTOMY - surgical procedure that eliminates pocket depth by resecting the tissue coronal to the pocket base. The final aspect of the gingivectomy is to bevel (contour) the wound's coronal margin to provide the most physiological shape and marginal thickness so adequate oral hygiene techniques can be performed. Indications: treating pseudopockets, hereditary , suprabony pockets, and hyperplasia caused by Dilantin therapy. Contraindications: infrabony pockets (defects) and lack of attached tissue. Limitations include compromised esthetics with longer teeth, lack of access to bony defects, and having a broad, open wound post-surgically. Factors to consider when electing to perform a gingivectomy rather than a periodontal flap: pocket depth (if the base of the pocket is located at the mucogingival junction or apical to the alveolar crest, do not perform a gingivectomy); need for access to bone; and amount of existing attached gingiva. 74. There was a question regarding pain on biting ..I answered cracked tooth - Prolonged sensitivity to heat, cold, and pressure after cementing a crown or fixed bridge is usually related to OCCLUSAL TRAUMA. If CR occlusion is high, patient complain of cold sensitivity and pain on biting down hard. Endodontically treated teeth restored with a cast post, core, and crown are subjected to the high incidence of VERTICAL ROOT FRACTURE. The main symptom is almost always PAIN WHEN BITING, and the radiograph usually appears normal. 75. Adv of stainless steel over ni titanium: SS files r more fracture resistant than Ni-Ti files

76. Maxillary Incisors &Canines: ALL have 1 root, 1 canal, & a distal axial inclination. Thus, when penetrating along the long axis of the tooth, the bur must be slightly angled toward the distal surface to avoid perforation of the mesial portion of the root. (DD) 77. MOD amalgam ...fracture on occlusal surface but not at isthmus wat to do ...options were polished out of occlusio..n observe .. take mo re do piece separate and rest I forgot - Fracture of a Class II amalgam restoration at the junction between the occlusal and proximal portions is the result of inadequate resistance form. When placing a Class II amalgam in a primary tooth, with isthmus width should be 1/3 of the inter- cuspal width. If an amalgam fracture occurs, it is most likely to occur here. A successful amalgam restoration requires 90- degree amalgam margins. Amalgam margins less than 90 degrees result in increased potential for fracture of the amalgam. Greater than 90-degree amalgam margins are good for the amalgam, but the corresponding enamel margin is less than 90 degrees and potentially undermined and prone to fracture. Because the amalgam is not bonded to the tooth, it must be retained in the tooth with under- cuts, in either the primary or the secondary preparation. An amalgam restoration needs a minimum 1-mm thickness in nonstress areas and 1.5- to 2-mm thickness in areas that may be under load. The preparation must provide this dimension. Except for class V amalgams, the prepared walls generally converge to the exterior. 78. Shape of mand access prep: Mandibular central incisors and canines – OVAL, mandibular PMs – OVAL, mandibular molars – TRAPEZOIDAL. 79. If pt injects LA intravs wat will happen ..there was an option tachycardia I answered that - LIDOCAINE is the local anesthetic that may manifest its toxicity clinically by initial depression and drowsiness (rather than stimulation and convulsion). The initial effect on the brain for local anesthetics is usually stimulation, then depression. However, it is possible that the excitatory phase of the reaction may be extremely brief, or may not occur (this is true especially with lidocaine and mepivacaine) causing patients to feel drowsiness. *lidocaine & Mepivacaine can also show cross- allergenicity. Usually, the FIRST clinical sign of mild lidocaine toxicity is NERVOUSNESS. Mild toxicity can be caused by an intravascular injection, unusually rapid absorption, or too large a total dose of the local anesthetic. Clinical manifestations of a mild lidocaine toxicity related to CNS excitation. Nervousness (increased anxiety), talkativeness, muscular twitching, perioral numbness, increased HR, BP, RR. Lidocaine can skip the excitatory phase and go straight to the depression phase (drowsiness). If the clinical manifestations do not progress beyond these signs with retention of consciousness, no definitive therapy is needed. The lidocaine will undergo redistribution and biotransformation, and the blood level will fall below the toxic level in a short time. Treatment of a sustained convulsive reaction to a local anesthetic includes oxygen and Diazepam IV. If proper equipment and adequately trained staff are unavailable, do not attempt injections. Possible side effects of lidocaine systemic absorption (not necessarily toxic levels) are tonic-clonic convulsions, respiratory depression, and decreased CO. EPI treats bronchospasm associated with hypotension (i.e. anaphylaxis). EPI is the agent of choice to treat/reverse anaphylactic reactions (given sublingually or subcutaneously) because it has desirable vasopressor activity, bronchodilator properties, and has a rapid onset of action. EPI has stimulatory effects on a & b adrenergic receptors. EPI treats hypersensitivity reactions. Common EPI side effects: headaches, agitation (anxiety), and tachycardia. EPI is used with caution in patients with high BP and hyperthyroidism. These patients may have an increased sensitivity to EPI. 80. The major reason that EPI vasoconstrictor is added to local anesthetics is to prolong the activity/duration of the local anesthetic (anesthesia) by decreasing the rate of diffusion and absorption from the injection site. EPI is ineffective in treating hypotension because of its alpha receptor stimulatory actions on the vasculature which could cause an even further elevation of BP. EPI does not reduce anxiety, but increases anxiety because it has CNS stimulatory effects. 81. They give childs age and asked for max dosage of lidocaine ...multiply it with 4.4 * kg 82. How to differentiate between apical and perio abscess 83. Remember drugs that Dec salivation: Anticholinergic drugs block receptor sites for acetylcholine and decrease salivary flow and respiratory secretions during surgery. Scopolamine, Atropine, and Benztropine decrease saliva flow and secretion from respiratory glands during general anesthesia. Decrease saliva flow during dental procedures (anti-sialogue) and the secretion from respiratory glands during general anesthesia: Atropine is the most commonly used anti-cholinergic drug for these purposes. Scopolamine penetrates the CNS more readily than Atropine, but is rarely used. A patient pre-medicated with Atropine will exhibit mydriasis (dilated pupils). 84. Ledge wat to do - bypass it and continue with ...use larger file to get rid of it...use bur to get rid of it ... Prebend the tip of a small file, lubricate, and try to negotiate around the ledge. Ledges can sometimes be bypassed; the canal coronal to the ledge must be sufficiently straightened to allow a file to operate effectively. This straightening may be achieved by anticurvature filing (file away from the curve). The dentist pre- curves the lfie severely at the tip and uses it to probe gently past the ledge. Otherwise, the dentist cleans to the ledge and fills; the patient is warned of the poorer prognosis. 85. What band and loop does provide ..vertical stop - "Band &Loop" Space Maintainer- most often used when the PRIMARY FIRST MOLAR must be prematurely extracted. Used after a unilateral loss of a primary first molar. It consists of a band that is usually cemented to a primary second molar. Attached to the band is a loop that extends to the canine's distal surface.The loop prevents mesial migration of the primary second molar. Note: limited strength allows only single tooth-space maintenance. 86. Space Maintenance: A 9 year old patient had an extraction of the primary mandibular first molar. The ideal treatment at this time is PLACE ASPACE MAINTAINER. Although this can be done with ei- ther fixed or removable appliances, fixed appliances are preferred in most situations because they eliminate the factor of patient cooperation. If the space is unilateral, it can be managed by a unilat- eral fixed appliance ("band & loop" space maintainer). If molars on both sides have been lost, and the permanent incisors have erupted, it is usually better to place a lingual arch space maintainer. 87. Collagenase and elastase by which bacteria: Porphyromonas gingivalis - Plaque bacteria produces ENZYMES (hyaluronidase, collagenase, chondroitin sulfatase, elastase, andproteases) that may initiate periodontal disease. • Collagenase- enzyme produced by Bacteroides species that catalyzes collagen degradation (hydrolysis). • Hyaluronidase-enzyme produced by Streptococcus mitans & salivarius that destroys the amorphous ground substance. • Chondroitin Sulfatase-enzyme produced by Diptheroids that also destroys amorphous ground substance. 88. Bacteria in 2 day of plaque - Bacterial Colonization: Gram + rods and cocci - DD: Cocci still dominate but there are increasing numbers of filamentous forms and slender rods. Gradually the filamentous forms replace many of cocci. Within a short time after cleaning a tooth, the pellicle coated tooth surface is colonized by gram (+) bacteria (i.e. Streptococcus sanguis, Streptococcus mutans, & Actinomyces viscosus - rods). These are the primary colonizers. 1. Primary Plaque Colonizers: DAY 1- GRAM (+) facultative bacteria that are the constituents of EARLY (YOUNG) plaque, appearing within a short time after cleaning a tooth. Streptococcus sanquis, Streptococcus mutans, & Actinomyces viscosus. 2. Secondary Plaque Colonizers: are GRAM (-) bacteria (Fusobacterium nucleatum, Prevotella intermedia, & Capnocytophaga species). Tertiary Plaque Colonizers: consists of GRAM (-), ANAEROBIC RODS that appear 1 week after plaque accumulation (Porphyromonas gingivalis, Campylobacter rectus, Eikenella corrodens, Actinobacillus actinomycetemcomitans/AA, and oral spirochetes (Treponema species). 89. There we're 2 que regarding cervical burnout: CERVICAL BURNOUT (due to decreased X-ray absorption) - a phenomenon caused by relatively low x-ray absorption on the mesial or distal surfaces of teeth, between the edges of the enamel and adjacent crest of the alveolar ridge. Because of this diminished x-ray absorption, these areas appear relatively radiolucent will ill-defined margins. It is caused by the normal configuration of the affected teeth (CEJ) which results in decreased x-ray absorption in those areas. These radiolucencies should be anticipated when viewing x-rays of almost any tooth, and must not be mistaken for a carious lesion. 90. Pt had less caries risk .. having stains and catch ..wat to do ...observe or gic 91. 40 yr pt with all 32 teeth no cavities . Has stain and catch in pit of molar ? What u do? A. Watch and observe 92. They ask u to recognize plaque in radiographs 93. Most common area for caries ...pit and fissure or apical to contact .... 94. Angle of instruments in SRP (Scale & Root Planing): Angulation refers to the angle between the face of the bladed instrument and the tooth surface. During scaling and root planning, optimal angulation is between 45 and 90 degrees. For subgingival insertion of a bladed instrument such as a curette, angulation should be as close to 0 degree as possible. 95. The BEST CRITERION to evaluate the success of SRP is NO BLEEDING ON PROBING (since BOP indicates active inflammatory periodontal disease). 96. Sharpening: The angle between the blade face and stone is kept at 100-110°. When sharpening instruments with a flat stone, the angle between the instrument and stone is 100-110°. 97. When sharpening instruments, the ______of the blade should be placed at a ______angle to the sharpening stone. Choose Your Favorite: Working end, 90° - Bevel, 45° - Bevel, 90° - Working end, 45° / OPERATORY When sharpening an instrument, the bevel of the blade is placed at an angle of 45 degrees to the sharpening stone. This ensures maximum efficiency of the blade to sharpen the instrument. 98. MOST common reason for amalgam failure ...preparation - The most frequent cause of failure of dental amalgam restorations is improper cavity design. 99. Common reason for failure of composite in posterior: LOW WEAR RESISTANCE is the property of filled resins that is primarily to blame for the failure of Class II composite restorations. One of the most common causes of failure in posterior composite restorations is occlusal wear. 100. Venner break down ... How to fix back ...microetch, etch, silane, bonding 101. They asked u treatment plans on anterior teeth like PFM crown .... All ceramic ..composite 102. How water contamination affects amalgam: Amalgam failure >> water contamination DELAYED EXPANSION of amalgam restorations is associated with insufficient trituration & condensation, and amalgam contamination by moisture during trituration and condensation (MAIN cause of failures). If moisture is incorporated into an alloy that contains zinc, the water reacts with the zinc to produce hydrogen gas. The resulting pressure from the liberated gas produces severe expansion of the amalgam, causing amalgam to protrude from the cavity preparation, post-operative pain, and excessive corrosion. Amalgam's compressive strength is greatly reduced when contaminated with moisture. 103. There was case on asthmatic pt: Two types of asthma exist (allergic asthma) the most common, and idiosyncratic asthma. Avoid using aspirin, NSAlDs, aspirin, barbiturates, narcotics, and erythromycin (if the patient is taking Theophylline). Corticosteroids treat asthma, Albuterol: b2 selective agonist. Administered orally or by inhalation. Primarily used in long- term treatment of obstructive airway diseases, emergency treatment of bronchospasm, or to delay premature delivery. Only EPI combines with 2 receptors to vasodilate bronchioles (treats asthma), skeletal muscle, uterus (relaxation), increase blood glucose (gluconeogenesis, glycogenelysis). NE usually combines weakly or not at all to b2 receptors. Nitrous oxide is SAFE to administer in asthmatics. Inhalation of a selective beta2-agonist (Terbutaline, Albuterol) is the preferred treatment for an acute asthmatic attack. Management of an Acute Asthmatic Episode during oral surgery: Terminate all dental treatment and position the patient in an erect or semi-erect position. Patient should administer their own bronchodilator using an inhaler. In most severe asthma attacks or when the patient's bronchodilator is ineffective, EPI (O.3ml of a 1:1,000 dilution) can be injected 1M or SC. Administer oxygen and monitor vital signs. 104. Proximal retention of amalgam: Axial wall depth. A, If no retention grooves needed, axial depth 0.2 mm inside (internal to) dentinoenamel junction (DEJ). B, If retention grooves needed, axial depth 0.5 mm inside (internal to) DEJ. Retention form in class II amalgam preparation: Occlusal convergence of buccal and lingual walls of the occlusal and proximal extensions. Dovetail design of the occlusal outline, retention looks or grooves. 105. When placement of proximal retention locks in class II amalgam preparations is necessary, which of the following is not correct? A. One should not undermine the proximal enamel. B. One should not prepare locks entirely in the axial wall. C. Even if deeper than ideal, one should use the axial wall as a guide for proximal lock placement. D. One should place locks 0.2 mm inside the denti-noenamel junction (DEJ) to ensure that the proximal enamel is not undermined. Retention locks, when needed in class II amalgam preparations, should be placed entirely in dentin, not undermining the adjacent enamel. They are placed 0.2 mm internal to the DEJ, are deeper gingivally (0.4 mm) than occlusally (i.e., they fade out as they extend occlusally), and translate parallel to the DEJ. If the axial wall is deeper than normal, the retention lock is not placed at the axiofacial or axiolingual line angles, but rather is positioned 0.2 mm internal to the DEJ. If placed at the deeper location, it may result in pulp exposure, depending on the location of the axial wall depth. 106. There was a question regarding probing depth inc in inflammation: Bleeding: physiologically, bleeding from the gingival sulcus should not be caused by gentle provocation. Bleeding in the absence of local irritants may indicate a systemic disease. Bleeding is the most reliable indicator of gingival or periodontal inflammation. 107. The two MOST critical parameters for the prognosis of a periodontally involved tooth are ATTACHMENT LOSS (most critical) and tooth mobility. 108. Tooth #30, a probing pocket depth on the facial is 6mm, the free gingival margin is 2 mm apical to the CEJ. Attachment loss of this tooth? a. 6mm b. 2mm c. 8mm d. 4mm – To measure attachment loss use a periodontal probe from an established reference point (CEJ or restoration margin) for initial measurement. Probings are repeated after appropriate time intervals and any changes are noted (i.e. if a probing depth is 4mm and recession is 3mm, total attachment loss = 7mm). 109. Axial walls on onlay: For a cast onlay or inlay, ALL axial walls must be convergent or there will be an undercut (DD). For an MOD onlay preparation, axial walls must slightly converge from the gingival walls to the pulpal wall (for the same reason the onlay would not seat if the axial walls diverged). 110. From facial to lingual, the axiopulpal line angle of an onlay preparation is longer than the axiogingival line angle. 111. The function of the direct retainer is to retain the removable partial denture by means of the abutments. Stabilization is provided by the minor connector. Support is provided by the rest. The indirect retainers improve the efficiency of the direct retainers. Direct retainers do not add strength to the major connector. 112. What can't be used as retainers in FPD inlay, onlay, partial crown, full crown (ALL can be used!) - The use of inlays as bridge retainers has died out – mainly because a tooth that is suitable for an inlay would also be suitable for a resin-retained wing and therefore the conventional bridge would be contraindicated. 113. Potassium sparing diuretic - Spironolactone, a potassium-sparing diuretic useful in treating edema and heart failure, is a com- petitive antagonist at the aldosterone receptor. 114. What do you often need to supplement with diuretics? Potassium 115. Which of the following are Potassium sparing Diuretics: 1-Spirinolactone 2- Triamterene, 3-Amiloride 4- all 116. Spironolactone and eplerenone. 1. True antagonists of aldosterone. 2. Similar in effects to amiloride and triamterene. 117. Which of the following drugs blocks the aldosterone receptor? A. Amiloride B. Triamterene C. Losartan D. Spironolactone E. Furosemide - Aldosterone antagonists: spironolactone, eplerenone. Spironolactone, a potassium-sparing diuretic useful in treating edema and heart failure, is a competitive antagonist at the aldosterone receptor. 118. The most important toxic effect of potassium sparing diuretics is: Hyperkalemia 119. Primary tooth with shortest occlusal table options were between max and mand molar: Greater convergence of the buccal and lingual walls results in a proportionately narrower occlusal table. This is more pronounced in the MAXILLARY FIRST PRIMARY MOLAR than second primary molar. 120. Pt came back after a month with discolored margins what could be the reason? discoloration due to leakage. 121. What opaque porcelain doesn't do - Opaque porcelain must mask the dark oxide color and provide the porcelain-metal bond. Bond strength depends on good wetting of the metal surface. Masking must be accomplished with the minimum thickness of opaque—about 0.1 mm— leaving maximum space to develop a natural appearance with body and incisal porcelains. DD: The opaque is applied first to mask the metal and to give the restoration its basic shade. 122. Which layer of porcelain mask the dark oxide color? 1. Incisal 2. Body 3. Opaque - 123. Which layer of porcelain provides most Color or shade? 1. Opaque 2. Incisal 3. Body Body or dentin porcelain contains most of the color or shade and is used generally to build most of the crown. Incisal porcelain is the most translucent layer of porcelain. The restoration is bulked out (overcontoured) to compensate for the 20% shrinkage, which occurs during firing. 124. What is function of opaque porcelain EXCEPT a. mask metal framework b. to help come up with a base/stump shade c. for initial bond to metal d. to decrease contamination of additional porcelain with metal in ensuing firing and baking procedures. 125. Cusp involved in non working interference LUBL (inner inclines) 126. Function of post: the primary purpose of the post is to retain a core in a tooth with extensive loss of coronal structure. Posts do not reinforce the tooth, but rather weaken it further by additional removal of dentin and by creating stress that predisposes to root fracture. Post preparation. The primary purpose of the post is to retain a core in a tooth with extensive loss of coronal structure. The need for a post is dictated by the amount of remaining coronal tooth structure. Posts do not reinforce the tooth but further weaken it by additional removal of dentin and by creating stress that predisposes to root fracture. At least 5 to 7 mm of remaining gutta-percha is recommended. 127. Why v record protusive relationship: A protrusive record registers the anterior- inferior condyle path at one particular point in the translatory movement of the condyles. Some clinicians use this type of record to determine the amount of space between maxillary and mandibular teeth or occlusal rims to maintain balanced occlusion throughout the mandibular functional range of movement when articulating teeth. Christensen’s phenomenon refers to the distal space created between the maxillary and mandibular occlusal surfaces of the occlusion rims of dentures when the mandible is protruded. It is caused by the downward and forward movement of the condyles. BB - The purpose of making a record of protrusive relation is to register the condylar path and to adjust the condylar guides of the articulator so they equal the patient's condylar paths. The mandible can protrude -10 mm. The protrusive record is probably the LEAST reproducible maxillomandibular record. 128. What is the purpose of making a record of protrusive relation and what function does it serve after it is made: A. To register the condylar path and to adjust the inclination of the incisal guidance. B. To aid in determining the freeway space and to adjust the inclination of the incisal guidance. C. To register the condylar path and to adjust the condylar guides of the articulator so that they are equivalent to the condylar paths of the patient. D. To aid in establishing the occlusal vertical dimension and to adjust the condylar guides of the articulator so that they are equivalent to the condylar paths of the patient. 129. After dpc place gic liner over caoh 130. They ask u to recognize class 123 relationship 131. Where to place retentive arm: The terminal end of the retentive arm is optimally placed in the middle of the gingival 1/3 of the clinical crown. However, it is acceptable to place it at the junction of the gingival and middle 1/3 of the clinical crown. When the partial is completely seated, the retentive arm should be passive and applying no pressure on the teeth. 132. Undercut for co-cr retentive clasp arm is A.0.75 mm B.0.5mm C.0.25 mm 133. Flexibility of the retentive clasp arm does not relate to: A. Length B. Cross section C. Material D. Degree of taper E. Under cut area 134. In case history: mentioned pt is receiving fluoridated water ...but having plaque ....in treatment options were like add more floride ...floride gels ...oral hygiene reinforcement. 135. They gave picture of condyles on panoramic and asked if pt have osteoarthritis rheumatoid arthritis...2 were more I forgot - Degenerative Joint Disease (Osteoarthritis)-organic degeneration of the TMJ's articular surfaces. 136. Football player use mouth guards, now he is having pain ..and he is not able to open jaw in the morning ...I answered myofacial pain syndrome ...there was arthritis also but I choose mps 137. There was questions on rubber dam holes I don't remember exactly - The hole to be punched in the rubber dam for the tooth being restored should be facial to the normal alignment with the adjacent teeth. Highest chance of mikroleakage in rubber dam holes are too closed. 138. Abscess ... penicillin vk. Treatment of the acute periodontal abscess is determined initially by whether there is localization of the abscess (if there is, drainage is the treatment). If the abscess is not localized, the patient is placed on antibiotics (Penicillin V) and instructed to rinse with warm saline. Clindamycin can be used in penicillin allergic patients. 139. Composite with stains and pits ..but other wise intact ...wat to do 140. Marginal stain on class 3 composite, how would u treat? REPAIR ?? or replace? - Its Replace.. marginal stain caused by microleakage. 141. 40 y pt w/ all 32 teeth. No cavities. Has stain & catch in pit of molar. what do you do? a. watch and observe b. sealant c. composite 142. Liquid in gic: The powder (calcium- aluminofluorosilicate glass) reacts with a liquid (polyacrylic acid) to form a cement of glass particles surrounded by a matrix of fluoride elements. Glass ionomer and polycarboxylate both contain polyarcylic acid. 143. Color stability tegdma: tegdma inc color stability and dec viscosity produced by bisgma (BisGma has questionable color stability) 144. IRM (Zinc Oxide Eugenol Cement) with PMM - Type III ZOE (Reinforced ZOE): a temporary filling material (lRM) and thermal insulating base. The powder is composed of zinc oxide and finely divided polymer particles (polymethyl- methacrylate) in the amount of 20-40% by weight. The zinc oxide powder is surface treated by an aliphatic monocarboxylic acid like propionic. Eugenol is the liquid. This combination of surface treatment + polymer reinforcement results in a material that has good strength and toughness which markedly improves abrasion resistance. Good for basing large and complex cavities as it withstands the pressure of amalgam condensation, with minimal effect on the pulp. 145. Chelation-a chemical phenomena common to zinc oxide-eugenol and polycarboxylate cements. 146. The cement which has antibacterial property is: Glass informer cement. Copper oxide cement. Zinc Polycarboxylate cement. Zinc phosphate cement. Zinc oxide Eugenol cement. 147. Not used for cast rest: irreversible impression material 148. Irreversible or reversible impression material 149. Safety valve of no allow 70 percent of nitrous oxide and 30 oxygen 150. Characterstics of autistic child: repetitive questions, repetitive behavior. 151. Cardiac arrest in children: 152. Most common reason for cardiac arrest in children – respiratory depression 153. Minimum anc count for surgery 154. Minimum granulocyte count: Generally accepted reference range for absolute neutrophil count (ANC) in adults is 1500 to 8000 cells per microliter (µl) of blood. 155. Topical antifungal options fluconazole clotrimazole miconazole griseofulvin – Clotrimazole and miconazole used topically to treat candidiasis. 156. Does premedications required options were cardiac stent, heart murmurs, pt had knee replacement within 2years - High-Risk Patients with prosthetic joints for Hematogenous Total Joint Infection: ALL patients during the first 2 years after joint replacement require antibiotic premedication. If more than 2 years after prosthetic joint replacement pass with no complications, then premedication is NOT required. 157. MI within 6 months.:. Antibiotic prophylaxis given or not? DD: YES 158. Distance for lateral cephalometric: 4feets, 6 feets, 60 inches - The distance b/n x ray source and the mid-sagittal plane of patient is fixed at 5 feet(60 inches), 6 feet is the distance between operator and patient. 159. Tooth removed socket tissue is examined ..they found lymphocyte neutophills, (granulocytes), and hemophiillc precursors ...I answered leukemia 160. Tooth removed ..tissue is examined ...they found inflammatory cells granulocyte etc ...options were scar cyst granuloma 161. Pt had fracture and numbness on side on nose ...fracture site floor of maxillary sinus roof of orvit ..medial wall of maxillary sinus ... 162. A patient experiences numbness of the left upper lip, cheek, and the left side of the nose following a fracture of his midface. This symptom follows a fracture through the A. nasal bone. B. zygomatic arch. C. maxillary sinus. D. infraorbital rim. 163. Lower lip numbness is seen in what kind of mandibular fracture, ANGLE T/F (angle and body) 164. There were half radiolucencies line on ramis ..identify ..fracture , air space ,two options were more ... 165. They can ask if the quality of x-ray is good or poor 166. Bacteria present in gingiva in ANUG when tissue is not necrosed ..that confuses me rods cocci spirochetes two options were more. Gingival crevices in ANUG show a predominance of fusobacteria and spirochetes. Treponema denticola, prevotella intermedia. ACUTE NECROTIZING ULCERATIVE GINGIVITIS (ANUG) - also called "Vincent's infection" or "trench mouth" is a condition that presents pathognomic signs and symptoms. Fusiforms, spirochetes, & Prevotella intermedia are involved in ANUG's etiology. Treponema denticola is the intermediate-sized spirochete associated with ANUG. Histologically, deeper areas of the lesion contain a zone of spirochetal infection. 167. Pt is having asymptomatic brown macules on buccal mucosa – intradermal nevus 168. Common mole is - A- blue Nevus B- compound Nevus C- intradermal Nevus D- junctional Nevus 169. Most common nevus in oral cavity: intradermal, intramucosal, compound, blue. 170. Melanocyte proliferation, inc melanin deposition: Increased in melanin- pigmentation seen in: (a) Albright's syndrome (b) Addison's disease (c) peutz jegher's syndrome (d) All of the above 171. Heat sensitive instruments sterlised by wat ..they didn't had ethylene oxide gas option .... But they had glutaraldehyde option ...I'm not sure - Glutaraldehyde Advantages: most potent category of chemical germicide, capable of killing spores (after 10hrs), EPA registered as a chemical sterilant, and can be used on heat- sensitive materials. 172. If prostaglandin is not there what will not happen ...it confuses me alot ..antipyresis, dec blood flow to kidney , platelet inhibtion and last option I forgot – reduced platelet aggregation. 173. Antiepileptic is used in which condition: Carbamazepine (Tegretol) - TRIGEMINAL NEURALGIA, Diazepam (Valium) - agent of choice to reverse status epilepticus induced by a local anesthetic overdose, Ethosuximide (Zarontin) - the preferred drug for effectively treating absence seizures because it causes minimal sedation by BLOCKING CALCIUM CHANNELS. Phenytoin (Oilantin) - treats tonic-clonic (grand mal) seizures. The rate of gingival hyperplasia is diminished by proper oral hygiene. Phenytoin is the most extensively used of all anti-epileptics, Phenytoin-induced gingival hyperplasia is a common adverse effect, that may partially or completely obscure teeth crowns 174. From where v can get large piece of graft ilium , ribs two options were more – iliac crest 175. Least resistant to fracture ...I ans high leucite I saw this question in group I'm not sure 176. Least resistant to fracture: a-pressed leucite b-feldspathic porcelain Leucite- reinforced ceramic c-Castable glass d-Glass-infiltrated alumina e- Leucite-reinforced ceramic 177. They ask how the position of mand foramrn shift relative to occlusal plane in kids - infraocclusal 178. Antiviral for hsv cmv vzv = FOSCARNET (HSV, VZV, CMV) Penciclovir (Denavir) - Herpes Simplex Virus Type 1 (HSV- 1). It is a cream formulation indicated to treat recurrent herpes labialis (cold sores) in adults. This condition is caused by HSV-l. Penciclovir is not available for systemic dosing. Acyclovir - inhibits viral DNA polymerase/viral DNA synthesis. Available in tablets and creams to treat HSV-1, HSV-2, & varicella zoster (chicken pox/shingles). Drug of Choice for HSV Encephalitis, genital herpes, herpes labialis (cold sores) & varicella-zoster virus. Valacyclovir (VALTREX)-PRODRUG of acyclovir given orally that is converted by 1st pass metabolism into acyclovir. Treats HSV1&2, genital herpes, cold sores (herpes labialis), &herpes varicella-zoster virus (shingles). Ganciclovir - inhibits viral DNA polymerase/viral DNA synthesis. Treats Cytomegalic Retinitis (CMV retinitis) & CMV prophylaxis in transplant patients (crosses BBB). 179. Bisphosphonates is not used in mutliple myeloma, Osteomyelitis, Metastatic cancer to bone from prostate and breast 180. Kidney transplant How to check it's failure initially: Dec renin creatn increased uric acid 181. Patient with renal transplant Caine to you for treatment to prevent the dental caries. What treatment should be contraindicated in him A)application of pit and fissure sealent B)flouride topical C)gic rest D)ttt with chlorhexidine 182. Patient has renal transplant, white non scapable lesion on the side of the tongue, shaggy frayed: a-hyperplastic candida b-idiopathic leukoplakia c-lichen plan d-hairy leukoplakia (shaggy frayed) appearance 183. Patient who has undergone kidney transplantation is at an increased risk of developing A brown tumors B. C erosive lichen planus D squamous cell carcinoma - Brown tumor is a rare complication of secondary hyperparathyroidism. It is exceptionally encountered after kidney transplantation. 184. Patient with kidney transplant needs prophy: Yes / no - No prophy but you cover with antibiotic after procedure since they have reduced immunity. 185. What ASA classification: someone who has DM, hepatitis C, hypertension, renal transplant 2 years ago. A,ASA 3 B.ASA4 C.ASA5 D,ASA6 186. Drug conjugation - more ionic - Conjugation of drugs results in polar, water-soluble compounds that are rapidly excreted in urine. Thus, the parent drug is effectively rendered inactive and transported out of the body by this process. 187. Types of reactions involved in drug metabolism. a. Phase I reactions involve reactions such as oxidation, reduction, and hydrolysis. b. Phase II reactions involve conjugation, in which a chemical substituent is added to the drug. The most common type of conjugation reaction is glucuronide conjugation. 188. In phase-II reaction in drug meabolism, the drug molecule is conjugated with ---? 1- a base 2- an acid - acid (glucoronic acid) - decks What contributes the most to a successful pulp capping ? a) exposure less than 0.75 mm b) isolated field c) absence of bleeding d) use o

DAY 2:

1150. Middle aged guy with kidney failure due to Lithium overdose. What pain drug is less expected to be nephrotoxic? Aspirin, Ibuprophen, Oxycodone, one more 1151. Why do we need ruler in lateral cephalogram? For magnification – scale Calibration ruler for magnification correction. Ruler to standardize the magnification rate of radiographs. 1152. In removal of palatine tori which structure can be damaged? Greater palatine artery 1153. 10 y.o girl, with good OH, no caries but a child of divorced parents. How would you rate her caries risk? Low, Middle, High - I took middle, because social history is super important 1154. Q. about that 10 y.o child case, where upper canines were closely to errupt, but primary canines were still there. They asked about the radiolucency that surrounded the erupting teeth. Options were different kinds of cysts and tumors. - I took eruption cyst.. I don’t know 1155. Hispanic guy, no insurance, needs tx. If you extract tooth 14, what is the most expected complication? The tooth had RCT and a very big amalgam fllg: Sinus perforation, Ridge fracture, Tooth fracture, Bleeding 1156. Pat. Allergic to sulfa, which meds. Are contraindicated? Next antibiotics: Sulfamethoxazole-trimethoprim (Septra, Bactrim), Erythromycin-sulfisoxazole, Sulfasalazine (Azulfidine), used to treat Crohn's disease, ulcerative colitis and rheumatoid arthritis. Dapsone, used to treat leprosy, dermatitis and certain types of pneumonia 1157. Middle aged Pat. With an one-tooth gap. If you want to close it orthodontically, what will you expect? No bodily movement of the teeth, Rotation mesio-facially, Rotation mesio-lingually, one more – I chose b 1158. A Q. about two small radioopacities in the bone where MD M1 was missing. There were options about Odontogenic tumors, Root rests, Focal idiopathic osteosclerosis. 1159. Some simple easy questions on identifying structures on x-ray. 1160. Also Qs on diagnostics of teeth on x-ray, if it is a proximal caries or burnout. - If it is on every tooth, it is most likely to be burnout! 1161. Qs about restorations on xray. 1162. Q on that child case which bite relationship it has. Distal, Mesial, End-to-end 1163. Test for kidney failure - creatinine 1164. Q on except for the 3molar incisor. 1165. How many teeth was the child missing. 1166. Q on RCT of a central maxillary incisor. What is true? I chose it will be difficult compared to a normal case, because canal was really obliterated. 1167. Which tooth would most likely need a RCT when observing the xray? Easy! 1168. Pat. Bites down on his maxillary M1, which already had a super big amalgam filling, and breaks off one of the cusps supragingivally. What tx.? – PFM 1169. On xray, opacity apical of a RC treated tooth. What could it be? Looked like sealer. 1170. Pat. Has very strange bite, posterior crossbite, Class 2 relationship on the molars, but almost perfect overbite. What is the most likely cause? I chose something with the Canines. I forgot :-D 1171. How to treat his posterior cross bite? Hyrax appliance (banded type)—for skeletal expansion, this is the most commonly used type of rapid palatal expansion/rapid maxillary expansion appliance. Haas appliance: However, difficulty in maintaining hygiene and possible inflammation of the palate are considered disadvantages by some clinicians. Hawley-type removable appliance with a jackscrew— for skeletal or dental expansion, this appliance may be used to correct mild posterior crossbites in children and young adolescents. Quad- helix and W-arch—generally for dental expansion, these appliances consist of heavy stainless steel wire with four (quad-helix) or three (W-arch) helices that are incorporated to increase the range and flexibility. 1172. Female pat. Wants all her teeth extracted, although they don't look as bad. Qs. About what tx. Is appropriate. 1173. That 10 y.o girl had a very strange bite, with a skeletal midline deviation to the left. They ask about the cause. 1174. On a lateral cephalogramm of the 10 y.o girl what is the radiolopacity that crosses her posterior teeth? Maxillary sinus, Orbita, Zygomatic arch, Palatine process of maxilla. 1175. Hispanic Pat, without insurance. Missing teeth, needs tx. What is the least tx indicated? I chose sinus lift 1176. Qs about that 10 y.o girl who was asmathic. Meds. 1177. Old pat., heavy smoker with white patch on lip? Hyperkeratosis due to smoking. 1178. How to biopsy that hyperkeratosis? incisional 1179. Pat. With leukoplakia on lower side of the tongue. How to biopsy that? - In all cases, leukoplakia must be completely excised since diagnosis cannot be made clinically (DD) 1180. What structure can be damaged while biopsying the lower side of the tongue? Medial to the hyoglossus: lingual artery, lateral to hyoglossus: submandibular duct, lingual nerve, lingual vein, hypoglossal nerve. 1181. Pat. With HTN. Taking various meds. Long list. Lisinopril, Beta-bloker among those. Which ones lowers his BP. - Lisinopril: ACE inhibitors, "inhibit" the conversion of inactive Angiotensin I Angiotensin II (a vasoconstrictor). This causes peripheral vasodilation and secondarily increases urinary volume excretion. Both actions cause reduced BP. 1182. Easy Q on tx. Of a RCT tooth with a big amalgam fllg. 1183. Elderly pat. With a super old bridge on lower back teeth. Has sensitivity when drinking cold drinks. Which tooth most likely to be the reason? The bridge had 3 abutment teeth. On the x-ray one tooth he had a post, another one had a big radiolucency and to me it looked dead! :-D So, it was the only molar, which looked normal to me with a little subgingival calculus. So, I picked that! 1184. Pat. Taking antidepresants. Having heavy bruxism. What is least likely to do to treat him. All options were correct, except the one saying to lower his medication! :-D 1185. Pat. With pigmentation above maxillary lateral incisor. What could that be and how would you biopsy that? - I took excisional, because it was small! Probably 3:3mm 1186. A Q. about that elderly apt. taking various med. Including Aspirin 81mg. Would you stop Aspirin before SRP? NO 1187. Middle aged woman who wanted all her teeth pulled out, had a radiolucency in the canal of an endo treated upper canine. What could that be? Radiolucency in the canal, guys! It was something with endo access and too much removal of guttapercha 1188. Another Q about her, showing a lateral upper incisor with post and crown, asking how that post looks on the x-ray. Too narrow, too long, too wide, too short 1189. Another Q about her. She had multiple fractured teeth, which were all previously endo treated. Why does her oral situation looks like that. All except question. I took external tooth trauma. 1190. Q. about that depressed guy, and his Amitriptyline med. Tricyclic antidepressants are very likely to cause xerostomia. Amitriptyline is especially potent in this regard. (Mosby) The most common CNS adverse reaction is DROWSINESS. Anti-cholinergic adverse effects are dry mouth (xerostomia), constipation, blurred vision, and tachycardia. Drug-induced xerostomia must be treated palliatively with artificial salivary substitutes. – BB EPI (vasoconstrictor) in local anesthetic injections must be used cautiously in patients taking tricyclic antidepressants (Le. Elavil), Serotonin & NE reuptake inhibitors (Le. Effexor) to avoid transient and significant increases in blood pressure. These antidepressants greatly increase NE levels in tissues. In the presence of a vasoconstrictor administered via a local anesthetic injection, the patient can experience a significant elevation of blood pressure due to the vasopressor actions of the combination. - BB 1191. Case Q, on picture it is obvious Class I relationship of 1st molars (MB cusp of upper 1st molar occludes mesial buccal groove of 1st mand molar)...answered Class I, but later there was another Q saying that on cephalometrics pt has ANB=6...so what are we supposed to do? Go back and change previous answer to Class II instead of Class I? - SKELETAL CLASS 2 with dental class 1 (teeth may compensate for dental disharmony) - SKELETAL CLASS 2, DENTAL CLASS 1 1192. Case Q, on picture there is a very deep overbite (upper incisors covered 2/3 of lower incisors), but canines position looked Class I (cusp of upper canine occludes in between lower canine and premolar) , and Q asked looking at anterior teeth what Class this pt has? Class I, Class II - Class ll.....if anterior looks like cls 2 div 2 along with deep bite +class 1 molar or canine relation termed as DECKBISS 1193. Case Q: On x-ray, there is max lat incisor with RCT done and crown, periapical RL (looks like old RL 4 mm) and within RL RO in the middle...Q asked what was that: hypercementosis? – need options 1194. Same Pt had this max upper lat inc and 1 st molar only left on that side. Pt Decided to do RPD. 1) Because of the absence of canine it affects this lat inc long prognosis. T/F 2) Because this upper denture has no vertical stop with mandible on that side it affects this upper lat inc long prognosis. T/F 1195. Another pt has mild crowding in lower ant teeth, narrow arch, no crowding in maxilla, but narrow arch, very little overbite (like 1mm), long face...Q asked she expected to have all, except: dolicho face, deviated septum, insufficient lips, obtuse nasolabial angle, one more option....(maybe related to vertical occlusion ) - It can’t be obtuse angle because of the flarring of teeth 1196. Pt has mand premolar and edentulous posterior to it. What you won’t place on it restoring the space: distal rest with a buccal retentive clasp, mesial rest with bar placed on facial mesially, mesial rest with bar placed on center of tooth, one more option not significant I ruled out. 1197. Pt is scared, nervous, delays your appointments, comes to the office but does not open up to you, pt is very dissatisfied by previous treatments, several ways of describing a number of scenarios and a number of way pt talks to you, how will you respond to him/her? How will you bring his/her concerns to you? What is the first thing to do? Etc, almost 20-25 questions only on this basis 1198. You are doing a wheel chair transfer, which of the following will you not do? Sliding method or scrolling method something I forgot exactly what it was, but other options were obvious to do, I chose not to use the belt of the pt. 1199. Pregnant pt, 2-5 questions. When is the best time to treat her? Best time 2 nd trimester during pregnancy. What meds can u give her or not? acetaminophen you can. What is most likely to lead her present with a syncope? pressure on IVC 1200. Old pt, 84 yo, what is your primary concern? Talk to him as politely and simple language as u can, involve him in his decisions for tx plan, involve a guardian in his tx plans, etc 1201. Stubborn pt, comes in coz of dissatisfaction of his previous several dentists. Despite of so many changes he still seeks for better prosthesis, what makes u determine if he will be satisfied by your tx or not? Several verbal communications in options, I chose the most affirmative and agreeing to what the pt was saying, to build rapport and form of trust is most imp (somewhere around that). 1202. Pt on anti hypertensive drugs, anti diabetic drugs, some numerical readings, looked normal to me so I chose to proceed with the tx 1203. Pt on no significant med history, past history of some surgeries, blood pressure comes out to be a little high, what will you do next? Call his physician, ask him if he is okay, schedule him for a recall, measure BP after 15 mins and see what comes up 1204. Pt has AIDS, his viral load is 1,000, T-cell count is 30, what will be your inference? His viral load is too high and he is on no condition to get tx, he has no problems taking any tx, his T-cell count is too low and that might put him at a risk of infection 1205. AIDS pt in first stage. Which of the following will you notice? He is highly infectious, he is moderately infectious, he is symptomless in this stage, he will show up with opportunistic infections 1206. Pt on coumarin, what lab test will you look for? PT 1207. Malignancies to the oral cavity from the rest of the body are most likey to end up in which region? I said floor of the mouth, others were bony landmarks and side of tongue and corner of lip. 1208. Which equipments in the dental health clinic will need a signed something? UV light, halogen light, lasers, high speed hand pieces 1209. Pt comes in with signs of physical assault, whom shall u contact first? Police or concerned local committees or something like that, talk to pts guardian or do nothing 1210. Denture wearing pt has pink raised lesions on the palate, what can it be? Epulis, fibroma, papillary hyperplasia, etc 1211. Tori on upper palate, extending to the post palatal seal, what decides whether you remove it or not? Tori extension affecting the stability of the denture, undercuts affecting the retention of the denture, size of tori - If it extend to posterior palatal seal means affects retention, answer b 1212. Ortho forces applied, what is least likely to happen? Differentiation of cells due to chemical influences in the pdl, zones of oxygen tension and oxygen deficit are created, changes in pdl blood supply, changes in pulpal response or something like that. (Application of orthodontic forces will provoke a haemodynamic response in the pulp). 1213. Pt comes to you with a catheter, which fo the following will facilitate your tx? Ask the pt to remove the cath, you remove the cath, leave it as it is and take detailed history again, some other options I don’t remember 1214. Drugs not to give to a pregnant lady, NO N2O and tetra or BDZ in option, so know all

1215. Pt only visit dental office for emergency treatment, all will help him to change his behaviour into routine check ups instead of emergency visits only by dentist Except- positive reinforcement, operant conditioning, educate the patient, give him knowledge regarding oral hygiene maintaince ( something like that) – Reinforcement means behavior increases, we don’t want that. 1216. Pedo, 1 was routine examination but found caries and what was tx for each tooth, about his behavior and patient management, simple stuff, space maintenance, number of permanent teeth seen on pano. She had a shunt placed some years ago, but no questions in that significance I saw. His pano had a oval radiolucency near the condyle on both the sides , asked for what it was and options were all anatomical landmarks like external auditory meatus or transverse canal etc 1217. Pedo, 2 was a girl with class 3 in primary teeth, although intraoral pics didn’t show primary 2Ms, but anterior were edge to edge so look for stuff like that to answer the questions. She lost a lot of space, they asked me the cause of space loss, and space management, not space maintenance, so look for small words in the questions to answer wisely, asked me her facial profile, her oral hyhient practices were prro, how to motivate her? Voice control, negative or positive reinforcement, etc. again simple 1218. Adult-1 was a man with mand tori identification on pano, with no significant med history but takes bisphosphonates, so everything went in that direction, for how would you modify your plans, not much hard or any new questions 1219. Adult-2 lady, had ortho done when she was teenager, now has upper front teeth lost, she is about 40’s now, reason for spaces, she had chelitis angularis, reason to that, and she had facia palsy, what would you tell the pt about the prognosis of this long term disease? Simple prostho management, placement of clasps, materials to be used, some teeth look likt their restorations are old, what will you treat these teeth with? This one was a little confusing coz her radiographs and no. of teeth seen on xrays didn’t match her clinical teeth, but was manageable coz didn’t ask me anything about that doubtful quadrant. 1220. Adult-3, 50 up pt, she had trauma some time ago and lower 3 teeth were discplored, upper right CI was RCt, apicectomy treated and she also had tori, but almost all teeth present, what will you do about the tori, what about the fractured crown, redo or repair, the upper CI periapical lesion did not heal in 2 years what can it be? And how to treat it? And other were simple RCT bleaching and crowns questions 1221. Adult-4, young lady with regular dental tx, on OCP, what meds not to prescribe, and she had a palatal lesion, they asked me differentials, she had unknown swellings in mand right post, vital teeth, differentials asked, extracted the third molar but cant resolve the lesion, was OKC, coz microscopy said they found epithelial cells and inflammatory cells, other regular questions on pdl management phases, she had Hep A treated previously, what should u keep in mind, I said it is not a blood borne disease, other things looked irrelevant, and if any special care or precautions needed etc 1222. Adult-5, 90 something year old man comes with his son as guardian, he has had tube ligation done, some anti hypertensive tx, several teeth missing, mand psot ridge knife edge, he thinks his dentures doesn’t fit any more, stuff about his prostho tx, but he had this one radiopacity between two teeth, asked for dx I probably wrote idiopathic osteosclerosis or something, but check this one! 1223. Adult-6, pt with very very poot OH, and retained root pieces, 3M present but no first or second molars in some places, lower both 3M were semi-impacted and mesioangulated, he had ameloblastoma, he had this drug for depression, for allergies, etc, and his treatment was based on early, and late treatment plans, kinds of prostho tx, clasps, crowns, materials of choice, etc 1224. Case 1 A lady of 50s..black pigment on palate. 1) which black pigment is not present in oral caivty a) Lentigo b) other IDK LENTIGO MALIGNA à usually occurs in the elderly. It is most common in sun damaged skin on the face, neck, and arms (Hutchinson freckle).

2) Pt has some non- painful, hard, movable swelling in the floor of the mouth (pic shown) pt is not aware about it: a) Sialolith b) Lymphoepithelial cyst c) Ranula 3) Pt has some non-painful lesion in 2nd molar region. Pt is not aware of the lesion. I could have which of the following D/D any but NOT a) Radicular cyst b) Lateral periodontal cyst c) Periapical cyst (tooth non vital, may be sensitive to percussion) d) O Keratocyst - Multiple lesions found in children may be a component of the nevoid basal cell carcinoma syndrome (Gorlin syndrome). THE CHIEF SITE OF INVOLVEMENT IS THE MANDIBLE IN THE POSTERIOR BODY AND ASCENDING RAMUS. Often associated with impacted tooth. Tendency to grow in an anterior-posterior direction without bony expansion. 1225. CASE 2: A child with missing lower right 2nd primary molar... 1) Space loss is due to mesial & distal drifting of both ant & post teeth 2) what kind of occlusion option a) class 1 on left class 2 right b) class 1 on left class 3 on right c) class 3 one left class 1 on right 3) Where does the chronic abscess seen in primary teeth a) Furcation b) Periapical c) other options 4) How to maintain the space for the missing 2nd primary molar with drifting of two adjacent teeth – My ans was we cannot as space is lost, we need space regainer its an ASDA ques 1226. CASE 3: Another child case I don't remember finding 1) pt has multiple class 1 caries what filling – I picked Amalgam over other as amalgam is preferred by the boards other reason composite has C factor and GiC has less strength 1227. CASE 4: elderly male 40 pack year cigarette smoking history with multiple drugs like gastric bypass, hypertension, other options 1) What can change this pt to stop the habbit (it had some rubbish options indirectly prompting us to say that he will not quit habbit without dentist motivation) a) Self motivation through behavior education 2) Behaviour of the society can be modified by a) Surveying b) Study conduction 1228. CASE 5: pt with upper and lower few teeth. Pt has financial restriction 1) How to raise his occlusion a) by fabrication of upper complete denture 1229. Case on Management of transient ischemic attack- read the drugs – antiplatelet agents are recommended over anticoagulants to reduce risk. Combining aspirin with dipyridamole is suggested over aspirin alone. Clopidogrel is a reasonable substitute for people allergic to aspirin. A transient ischemic attack (TIA) is a brief episode of neurologic dysfunction caused by ischemia (loss of blood flow) – either focal brain, spinal cord, or retinal – without infarction (tissue death). TIAs have the same underlying cause as strokes: a disruption of cerebral blood flow (CBF). Symptoms caused by a TIA resolve in 24 hours or less. Antiplatelet medications such as aspirin are generally recommended. They reduce the overall risk of recurrence by 13% with greater benefit early on. The initial treatment is aspirin, second-line is clopidogrel (Plavix), third-line is ticlopidine. If TIAs recur after aspirin treatment, the combination of aspirin and dipyridamole may be recommended. Some people may also be given modified- release dipyridamole or clopidogrel. An electrocardiogram (ECG) may show atrial fibrillation, a common cause of TIAs, or other abnormal heart rhythms that may cause embolization to the brain. An echocardiogram is useful in detecting a blood clot within the heart chambers. Such people may benefit from anticoagulation medications such as heparin and warfarin. 1230. Case 1) 11 years old, kidney dialysis for 10 years and got transplant 1 year ago. He had Hodgkin lymphoma 5 years ago, mitral valve and regurgitation. He is taking lot of complex medicines I don’t rem the name. RG and clinical pictures show he has amelogenisis imperfecta 1) All are immunocompromised drugs except: know all immunocompromised names and corticosteroids: Glucocorticoid, hydrocortisone, methylprednisolone, prednisone, (triamcinolone, beclomethasone, budesonide, flunisolide) these are inhaled corticosetetiod for astham treat. Other immunosuppresive, cyclosporin, azathioprine, methotrexate, cyclophosamide 2) What drug can cause amelogenisis imperfecta? tetracycline 3) why his third molars are missing? he is 11 year still third molar not erupted 4) Bilateral radioopacity in mandible whats the dx? 5)in a Rg canine was short in length whats the dx? AI, DI, Dentin dysplasia 6) does he need Ab before procedures? no need to antibiotic 7) why he has gingival enlargement? He was taking cyclosporine too, cyclisporine lead to gingival enlargment

1231. Case2) 14 years old, all 4 canines erupted buccally and has pigmented macules on her cheek, asthmatic taking albuterol 1) albuterol can cause all except? I put increased salivary secretion 2) small white lesions on palate? Cause of inhaler its candidiasis 3) is nitrous oxide is contraindicated? NO 4) Will you explain the whole ortho tx to her parents and post complications like she may need gingival grafts? YES 5) The reason of pigmentation on her cheek? Proliferation of melanocites, proli of basement cells, deposition of melanin or foreign body 6) will ectopically canine resorb #7 roots - YES 7) Anb 6, class 1, 2, 3? 8) Clinicall picture what class, it was class 1 9) Features of her face has everything except? I put incompetent lips, her lips looked fine to me 1232. Pt has all canines erupted ectopic, but the rest of occlusion looks fine, what should be done: Expansion of both jaws to place canines, extract canines and leave premolars on place , extract premolars and with fixed ortho move canines on their place 1233. If she decided to extract premolars what forceps not to used? -151 -150 -23 -286 1234. There was a q about the profile? It was convex depending in the photograph 1235. Another q asking what is the preventive treatment for this patient -sealant for #3,14, 19, 30 -encourage the patient to use brushing and dental floss -use mouth wash -one more option 1236. What is the most costly to do to prevent declassification around the braces -varnish every 6 month -scaling every 3 month -daily systemic supplement -using mouthwash 1237. Another q asking what is the treatment of choice for tooth #19 if it is already catch the prob during examination -compsite filling -sealant -no treatment

1238. Case3, 45 years male, 2 pack smoke a day, dry mouth, lot of carious teeth, went successful rehab for bad alcohol habits, seems he doesn’t drink now Q1) will you prescribe Acetaminophen/oxycodone in this patient? I put no Q2) missing canine will make max rpd compromised? YES 3) If you use #7 in rpd will it compromise the tooth? Yes cause no posterior teeth and no canine 4) Rg picture shows tori in maxilla and mandible both 5) 2*3 radioopacity on LI which has RCT on it, is it hypercementosis and will you biopsy it There was a photo here showing the patient has preparation about 0.5 from facial and incisal and a q ask about what type of restoration the patient lost -crown -Veneer -composite 1239. Case 4) middle age female, smokes daily and she is fed up from falling restorations evrytime and she wants to extract her all teeth, psoriasis in hands and feet Q 1) by doing what patient want, is conflict bw what two, autonomy, justice, nonm, bene? autonomy, nonmalficiency Q2) treatment options for her? Q3) what clasp will you give in max RPD if you class II kennedy - RPI Q4) why you can see condyles in PAN, bilateral fracture, osteoarthritis, rheumatoid arthritis? Q5) radiolunceny in bw 8 and 9 it was incisive foramen? Q6) if you want her to quit smoking the day of extraction would be the quit date and you give Chantix 1 week beore the quit date? T 7) some consent Qs 1240. An Old woman with Parkinson Disease came to the clinic with her hus-band. She had distal caries on maxillary molar. 1. Out of all the symptoms of Parkinson's disease which symptom is not important to dental treatment? A. Rapid Eye blinking B. Tremor C. Muscle rigidity D. Loss of Automatic Movements 2. While working, the patient moved, and the dentist injured her near the cheek. Which artery was injured and caused bleeding? A. Buccal artery of maxillary artery B. Labial artery of ECA C. Facial artery of ECA 3. What is the first important thing to do after the patient starts to bleed? A. Stop bleeding B. Call an oral surgeon C. Call her husband. D. Inform patient

1241. A 32 years old lady with cervical neoplasia comes to your clinic for ulcers on one side of her palate. Drug history of taking oral contraceptives. 1. Action of Oral contraceptives? Oral Contraceptives: Ovulation is inhibited by suppression of FSH and LH. A. Dec. Lh B. Inc Lh C. Inc Fsh D. Dec. Fsh 2.What could be the cause of the ulcer A. CMV B. EBV C. HPV 3. Patient is most likely to have which neoplasia A. HIV B. Cervical cancer C. Rubeola 4. What can be done for diagnosis of this viral disease except? A. Saliva examination B. Examination of fluid from vesicles C. Oral examination 5. Epithelium of this ulcer 1242. Cases with asthma women. She takes albuterol. What can she has: xerostomia. She had an attack what can you give her, choose three a.epinephrine b.oxygen c.steroids d. beta2 agonist Reason of asthma- read about that – constriction of brionchole. And inflammation of brionchole. Read it – it was q about true or false. What happens in asthmatic bronchiole – constrict. One q was a patient broke his tooth – max 1 pm. He had excellent hygiene. Tooth broke 2 mm under the gingiva. It has big amalgam filling. What you do: a.extraction and prosthesis b. RCT and post and crown. c.temporary filling and observe d. splint this crown to the tooth Many q about cases like: what you will do in this patient with his tooth 14 – decay – tret, don’t tret, observe. Pic of decay and you should recognazie it in the rvg or pic. A. Orthokeratinised B. Parakeratinised C. Nonkeratinised 1243. Case with the man with allergy to penicyline and clindamycine..He had a joint replacement . He doesn’t need prophy for that. Q was if he need prophy for treatment what you give him - Azitomycin He has terrible pain in the face. During the night too. It lasts 5 sec. It comes and go away. What is it a.acute sinuses sinues b.tooth ache c. Neuralgia – ans 1244. Case with 5 years old girl She had all first molars and erupting max central. Second central incisors – had accident and was avulsed. What you do with concussion a. extract b. observe and control max central c. splint Q about her age. Is her dental age; a. the same as bone age b. elder tooth age than bone age c. younger ……………… d. it can’t be connected She had dark changes in her gingiva; a. racial changes sth like this - ans b. Peutz jegher syndrome Q about sth on her gingiva above her central incisor. It was sinus tract. Because of her accident 3 mouth earlier. She has lateral crossbite – how to treat it. – maxillary expansion Can you give her for daily use CHX – T/F You should show her and her parent how to brush T/F 1245. Man with hyperlipidemia. Triglyceride is high. What do you suspect: metabolic syndrome. Metabolic syndrome can cause cancer of all except one: a. thyroid b. colon c. renal d. sth else. The same q as in group. 1246. Treatment of this patient: Initial therapy (OHIs – SRP – Caries control – crown lengthening) 1247. Man smoking. He had also rheumatoid disease. He has hypertension. Tekes Propranolol. Is taking Chantix. What he can have: xerostomia, dysguesia. Which drug causes dry mouth - Chantix 1248. You give him LA with epi – what happened – blood pressure elevated 1249. It doesn’t work – what else can you give him: a. bupropion Zyban b. nasal spray c. nicotine gum 1250. Which ASA classification he is – class II. RA (ASA classification) II Picture of mucosa in lower lip – hyperthrofia 1251. Sequence of tx plan (emergency – caries control – reevalute – definitive treatment – maintenance ) ECRDM 1252. Pic of amalgam filling: a. corrosion b. bad condensation c. bad trituration 1253. Gingival tattoo 1254. Children with cleft palte – class III malocclusion. Pano and cephalo. Given SNB - 73 and ANB -2 what angle class is it. 1255. There is an arrow in the pano of hyoid bone and q is show where is hyoid bone on the cephalo. There where 4 arrows and you should mark it. Was B ;) 1256. Picture of this child 11 years old, Maxilary central ok but lateral in crossbite. Sth like this. How to treat – maxillary expander? Patient had lateral crossbite – why - maxilla to small and lateral shift to the right side. What to do with teeth S – teeth was ok, let it don’t treat but observe Profile of this child in cephalo How many teeth is missing – you shoul count it. Had also some supranumerary – count them. Can’t implant in cleft palate. Tooth is painful – mand first molar. What to do: a. extract - ans b. pulpotomy c. don’t do anything What you will d 1257. Obese man with hypertension - 190/110 Didn’t treat it. Haven’t been in doctor 10 years. No medicaments. He is going to the restroom 2 per night. 1258. What do you suspect - diabetes type II 1259. MOA of sulforynoureas. 1260. What is the first think you will do to treat this patient: A. caries B. perio C. Extractions D. Refer to the physicans 1261. What contribute to the future risk of caries in this patient, exept one, which one: a. bad oral hygiene b. many sweets and bad diet (AMOUNT) c. not having dental appointment d. past caries and current caries e. changes in saliva 1262. Many q the same as in day 1. I have cases about: 1.Child with ADHD, What is he taking – amphetamine. He can’t sleep and eat because of this treatment. He had insomnia, why – amphetamine (indirect acting symphatomimetic) Not to many things during one appointment. Don’t use tell show do. – F, we use it. ADHD is most common with: males not female Read patient management about ADHD children Q about if you can give child with ampfo NO. Or should you lower the dose of amphetamine. No need to adjust dose 1263. Day 2 is tougher than day 1. Read a lot about drug interactions with epi. Bisphosphonates anti hypertensive drugs. 1264. One case i got about 8yr 7 months child with supernumerary anterior teeth. Patient had class 2 skeletal and class dental malocclusion due to missing canine teeth. 1265. Questions were. Extracting the supernumerary tooth when 1266. When to start ortho treatment for anterior cross bite 1267. How to correct patients molar relationship which were in dental class 2 malocclusion. 1268. SNA was 87 and SNB 82- what will we call 1) mandibular protrusion 2) maxillary protrusiom 3) mandibular retrusion 4) maxillary retrusion. 1269. This case really frustrated me up. 1270. patient had Hep A 20 years ago what lab test you need? Correct answer : nothing ! We don't care about his hep A 20 years ago !

EXTRA INFO REGARDING TIME AND BREAKS : Day 1 exam has 2 session of 3:30 min..100 ques 15 min break 100 ques 30 min break...100 ques 15 min break ...100 ques and done Day 2 exam has 57-67 ques 15 min break.....47 ques done... You cannot go back to any of ques once you finish those ques...Good luck to every one will post more if I remember anything else... I had 9 case 5 case before break with about 52 ques....4 case after break with about 48 ques